Plus Two Chemistry Model Question Paper 2

Kerala Plus Two Chemistry Model Question Paper 2

Time: 2 Hours
Cool off time: 15 Minutes
Maximum: 60 Scores

General Instructions to candidates

  • There is a ‘cool off time’ of 15 minutes in addition to the writing time of 2 hrs.
  • Use the ‘cool off time’ to get familiar with the questions and to plan your answers.
  • Read questions carefully before you answering.
  • Read the instructions carefully.
  • Calculations, figures, and graphs should be shown in the answer sheet itself.
  • Malayalam version of the questions is also provided.
  • Give equations wherever necessary.
  • Electronic devices except non-programmable calculators are not allowed in the Examination Hall.

Plus Two Chemistry Model Question Papers Paper 1 33

(Questions 1 to 7): Carry one score each. Answer all questions.
Plus Two Chemistry Model Question Papers Paper 2 1

Question 1.
Which type of stoichiometric defect am I shown by AgCI?
Plus Two Chemistry Model Question Papers Paper 2 2

Question 2.
What is the two-dimensional coordination; a number of a molecule in a square close-packed later?
Plus Two Chemistry Model Question Papers Paper 2 3

Question 3.
Mercury is a ……… cell.
Plus Two Chemistry Model Question Papers Paper 2 4

Question 4.
For the reaction ; Cl2 (g) + 2NO(g) → 2NOCI(g) ; the rate law is expressed as rate = K[CI2][NO]2 ! What is the overall order of this reaction?
Plus Two Chemistry Model Question Papers Paper 2 5

Question 5.
Name the method used for removing gangue I from sulfide ore.
Plus Two Chemistry Model Question Papers Paper 2 6

Question 6.
Which one of the following compound reacts, with chlorobenzene to produce DDT?
Plus Two Chemistry Model Question Papers Paper 2 7

Question 7.
Biotin is an organic compound present in yeast deficiency in the diet causes dermatitis 1 and paralysis. It is also known as
Plus Two Chemistry Model Question Papers Paper 2 8
a. Vitamin H
b. Vitamin B3
c. Vitamin B12
d. VitaminD

(Questions 8 to 20) : Answer any ten. Each question carries two scores.
Plus Two Chemistry Model Question Papers Paper 2 9

Question 8.
i. Which of the following is a molecular solid?
a. Diamond
b. Graphite
c. Ice
d. Quartz
ii. Unit cells can be classified into primitive and centered unit cells. Differentiate between primitive and centered unit cells
Plus Two Chemistry Model Question Papers Paper 2 10

Question 9.
i. Which of the following is a secondary cell?
a. Dry cell
b. Leclanche cell
c. Mercury cell
d. None of these
ii. What is the relationship between resistance and conductance?
Plus Two Chemistry Model Question Papers Paper 2 11
Plus Two Chemistry Model Question Papers Paper 2 12

Question 10.
Complete the following.
Plus Two Chemistry Model Question Papers Paper 2 13

Question 11.
Match the items of Column I with items of Column II.
Plus Two Chemistry Model Question Papers Paper 2 02
Plus Two Chemistry Model Question Papers Paper 2 01
Question 12.
How will you convert propanoic acid into the following compounds?
i. Ethane 0
ii. Butane
Plus Two Chemistry Model Question Papers Paper 2 15

Question 13.
Haloalkanes and haloarenes react with metals to give hydrocarbons or products from which hydrocarbons are obtained easily.
Plus Two Chemistry Model Question Papers Paper 2 16
Identify the product and name the reaction.
Plus Two Chemistry Model Question Papers Paper 2 17
Identify the product and name the reaction.
Plus Two Chemistry Model Question Papers Paper 2 18

Question 14.
Most of the organic chlorides, bromides and iodides react with certain metals to give compounds containing carbon-metal bonds.
i. Give one example of such a compound.
ii. How will you prepare the above compound?
Plus Two Chemistry Model Question Papers Paper 2 19

Question 15.
Schematic alignment of magnetic moments of ferromagnetic, antiferromagnetic and ferrimagnetic substances are given below. Identify each of them.
Plus Two Chemistry Model Question Papers Paper 2 20
Plus Two Chemistry Model Question Papers Paper 2 21
Plus Two Chemistry Model Question Papers Paper 2 22

Question 16.
Aldehydes and ketones are organic compounds containing a carbonyl group.
i. Write a chemical reaction to distinguish between aldehydes and ketones.
ii. Aldehydes and ketones can be subjected to Clemmensen reduction and wolfishness reduction. Name the reagents used in both cases.
Plus Two Chemistry Model Question Papers Paper 2 23

Question 17.
Write any two electrophilic substitution reactions of chlorobenzene.
Plus Two Chemistry Model Question Papers Paper 2 24

Question 18.
a. Which of the ones mentioned in the above table can be concentrated by magnetic separation. justify your answer.
b. Identify the ones that can be concentrated by leaching.
Plus Two Chemistry Model Question Papers Paper 2 25
Plus Two Chemistry Model Question Papers Paper 2 26

Question 19.
The temperature dependence of the rate of a chemical reaction can be accurately explained by Arrhenius equation. With the help of the Arrhenius equation calculate the rate constant for the first order reaction, C2 H5 I(g) → C2 H4(g) + HI(g) at 700 K. Energy of activation (Ea) for the reaction is 209 KJmof1 and rate constant at 600 K is 1.60 x 105 S’1. [Universal gas constant R= 8.314K1mol’1].
Plus Two Chemistry Model Question Papers Paper 2 27

Question 20.
Chemotherapy is a term found in medical terminology. What is chemotherapy?
Plus Two Chemistry Model Question Papers Paper 2 28

(Questions 21 to 29): Answer any seven. Each questions carries three scores.
Plus Two Chemistry Model Question Papers Paper 2 29

Question 21.
a. The accumulation of molecular species at the surface rather than in the bulk of a solid or liquid is termed adsorption.
i. What is adsorption isotherm?
ii. Write the mathematical expression of Freundlich adsorption isotherm.
b. Enzymes are known as biochemical catalysts. Write any two important characteristics of enzyme catalysis.
Plus Two Chemistry Model Question Papers Paper 2 30

Question 22.
Biomolecules are formed by certain specific linkages between simple monomeric units. Write the names of linkages and monomeric units in the following class of biomolecules.
i. Starch
ii. Protein
iii. Nucleic acid
Plus Two Chemistry Model Question Papers Paper 2 31

Question 23.
Polymers are macromolecules formed by a union of monomers.
a. Name natural polymer and synthetic polymer.
b. Distinguish between thermoplastic and thermosetting polymers with an example.
Plus Two Chemistry Model Question Papers Paper 2 32
Plus Two Chemistry Model Question Papers Paper 2 33

Question 24.
Carbohydrates are broadly divided into monosaccharides, oligosaccharides, and polysaccharides.
a. Write one example each of monosaccharide and oligosaccharide.
b. i. Write any one method for the preparation of glucose.
ii. What is peptide linkage?
Plus Two Chemistry Model Question Papers Paper 2 34

Question 25.
a. Amines are basic in nature. Arrange the following compounds in the increasing order of their basic strength.
NH2, C6H5NH2, CH3 NH2, (CH3) 2NH, (CH3)3N.
b. How will you convert Toluene (C6H5CH3) to Benzaldehyde
Plus Two Chemistry Model Question Papers Paper 2 35

Question 26.
Consider the coordination compound [CO(NH3)SO4]Br.
a. Write the IUPAC name of the above compound.
b. What is the primary valence and secondary valence of the central metal, Cobalt, in the above coordination compound?
c. Which type of structural isomerism is exhibited by the above coordination compound?
Plus Two Chemistry Model Question Papers Paper 2 36

Question 27.
Different drugs have different therapeutic action in our body. Write the therapeutic action of the following drugs in our body.
a. Tranquilizers
b. Analgesics
c. Antibiotics
Plus Two Chemistry Model Question Papers Paper 2 37

Question 28.
The concept of AG° of coupled reactions f is used to explain reductions in metallurgy.
a. Explain the above statement?
b. In the blast furnace for manufacturing iron, most of the reduction is carried out by CO rather than C (coke). How can you account for this?
Plus Two Chemistry Model Question Papers Paper 2 38

Question 29.
a. LDPE is a homopolymer, while Nylon66 is a copolymer. Explain?
b. Classify the following into homopolymer or copolymers. Nylon-66 and HDPE
Plus Two Chemistry Model Question Papers Paper 2 39

(Questions 30 to 33): Answer any three. Each questions carries four scores.
Plus Two Chemistry Model Question Papers Paper 2 40

Question 30.
Detergents are used to remove oil and dirt from surfaces.
a. What are synthetic detergents? Give an example.
b. What are anionic, cationic and nonionic detergents? Illustrate with examples?
c. A synthetic detergent is used as a dishwasher. To which of the above type would it belong?
Plus Two Chemistry Model Question Papers Paper 2 41

Question 31.
Crystal defects give rise to certain special properties in the solids.
a. What is meant by Frenkel defect?
b. VWiy does Li Cl not exhibit Frenkel defect?
c. Explain the pink color of LiCI when heated in the vapors of Li.
Plus Two Chemistry Model Question Papers Paper 2 42

Question 32.
The hydrolysis of an ester in acid medium is a first order reaction.
a. What do you mean by an Ist order reaction?
b. What is the relation between the rate of the instant and half-life period of a reaction?
c. Half-life period of a first-order reaction is 20 seconds. How much time will it take to complete 90% of the reaction?
Plus Two Chemistry Model Question Papers Paper 2 43

Question 33.
The limiting molar conductivity of an electrolyte is obtained by adding the limiting molar conductivities of cation and anion of the. electrolyte.
a. Name the above law.
b. What is meant by limiting molar conductivity?
c. Explain how conductivity measurements help to determine the ionization constant of a weak electrolyte like acetic add.
d. Explain the change of conductivity and molar conductivity of a solution with dilution?
Plus Two Chemistry Model Question Papers Paper 2 44

Answers

Answer 1.
Frenkel defect

Answer 2.
4

Answer 3.
Primary cell

Answer 4.
3

Answer 5.
Froth flotation process

Answer 6.
d

Answer 7.
a

Answer 8.
i. c. Ice I
ii. In primitive unit cells, constituent particles are present only on the corner position I of unit cell whereas in centered unit cell, one or more constituent particles are present at a position other than corners in addition to those at corners.

Answer 9.
i. d. None of these
ii. Conductance (C) is the reciprocal of the resistance (R). C = 1/R

Answer 10.
Plus Two Chemistry Model Question Papers Paper 2 45

Answer 11.
Plus Two Chemistry Model Question Papers Paper 2 46

Answer 12.
Plus Two Chemistry Model Question Papers Paper 2 47
ii. Kolbe’s electrolysis: By the electrolysis of aqueous solution of sodium salt of the propanoic acid.
Plus Two Chemistry Model Question Papers Paper 2 48
Butane is formed at the anode.

Answer 13.
Plus Two Chemistry Model Question Papers Paper 2 49

Answer 14.
i. Grignard reagent -R MgX
ii. The alkyl halide is treated with magnesium in dry ether to get Grignard reagents.

Answer 15.
i. Antiferromagnetic.
ii. Ferrimagnetic.
iii. Ferromagnetic.

Answer 16.
i. Oxidation: Aldehydes on oxidation give carboxylic acid containing same no. of C atoms.
Plus Two Chemistry Model Question Papers Paper 2 50
Ketones on oxidation give a mixture of carboxylic acids containing lesser no.of carbon atoms.
Plus Two Chemistry Model Question Papers Paper 2 51
ii. Zn amalgam and HCI for Clemmensen reduction. Hydrazine heated with KOH in ethylene glycol for WolfKishner reduction.

Answer 17.
Plus Two Chemistry Model Question Papers Paper 2 52

Answer 18.
a. Haematite, magnetite , Iron pyrites,
b. Bauxite.

Answer 19.
K = ln A – \(\frac { { E }_{ a } }{ RT }\);
where K = Rate constant,
A = Arrhenius factor, Ea = Activation energy, T= Temperature, R = Gas constant
Plus Two Chemistry Model Question Papers Paper 2 53

Answer 20.
i. Use of chemicals for therapeutic effect is called chemotherapy.

Answer 21.
a. i. The variation in the amount of gas adsorbed by the adsorbent with pressure at constant temperature is expressed using a curve called adsorption isotherm.
ii. \(\frac { X }{ m } =k.{ p }^{ 1/2 },n>1;\)
where x/m is the extent of adsorption, P is the pressure, K and n are constants which depend on the nature of the adsorbent and the gas at a particular temperature.
b. i. Most highly efficient. One molecule of an enzyme may transform one million molecules of the reactant per minute.
ii. Highly specific in nature. Each enzyme is specific for a given reaction.

Answer 22.
i. α – D- glucose, the bond is glycosidic linkage.
ii. An amino acid, a peptide bond is a linkage.
iii. Nucleotide, Bond is a hydrogen bond.

Answer 23.
a. Natural polymer Cellulose, Starch Synthetic polymer PVC, Nylon,
b. Thermoplastics are linear or slightly branched long chain molecules, which can be remolded again and again by heating and cooling. The intermolecular force is intermediate between elastomers and fibers.

Thermosetting are cross-linked or heavily branched molecules. On heating, they undergo extensive cross-links and becomes infusible. They cannot be remolded.

Answer 24.
a. Monosaccharide Glucose, Fructose-Oligosaccharide Maltose, Lactose, Sucrose Polysaccharide Cellulose
b.
i. Starch.
Plus Two Chemistry Model Question Papers Paper 2 54
ii. The linkage between units of amino acids in a polypeptide linkage is called peptide linkage.
Plus Two Chemistry Model Question Papers Paper 2 55

Answer 25.
Plus Two Chemistry Model Question Papers Paper 2 56

Answer 26.
a. Pentaamminesulphatocobait (III) bromide.
b. Primary valence is 3 and secondary valence is 6.
c. Ionization isomerism.

Answer 27.
a. Tranquilizers are used for the treatment of stress and mental diseases.
b. Analgesics reduce pain without causing any disturbance to the nervous system.
c. Antibiotics are drugs used to treat infections in our body. It inhibits the growth of microorganisms.

Answer 28.
a. For the feasibility of a reaction, ΔrG° must be negative. The positive value of ArG° means that the reaction is not spontaneous. This reaction can be made spontaneous by coupling with a reaction having very large ve Gibb’s energy value so that Gibb’s energy for the two combined reactions become ve.
b. A better reducing agent should have a larger ve value for AG°. If coke is used ArG° = 13 KJ/mol and for CO, ArG° = 56.2 KJ/mol Comparing the two values, it is clear that CO is a better reducing agent than C.

Answer 29.
a. If the polymer is formed only one type of monomer units, then it is a homopolymer. LDPE is low-density polyethylene whose monomer is ethylene. Hence it is a homopolymer.
Nylon66 is the polymer containing two different monomer units hexamethylene diamine and adipic acid. Hence it is a copolymer.
b. Nylon6: Homopolymer HDPE: Homopolymer

Answer 30.
a. Synthetic detergents are cleansing agents which have all the properties of soap, but actually, do not contain any soap. They can be used both in soft and hard water.
b.
i. Cationic detergents are quarternary ammonium salts of amines with acetates, Cl or Bras anions. Cationic part posses a long hydrocarbon chain. They are expensive and have germicidal properties used in hair conditioners, eg., Cetyl trimethyl ammonium bromide.
ii. Anionic detergents are sodium salts of sulfonated long chain alcohols or hydrocarbons. The anionic part of the molecule is involved in cleaning action. Used in toothpaste, household work, eg., sodium dodecyl benzene sulphonate.
iii. Nonionic detergents do not contain any ion. eg., Detergent formed by stearic acid and polyethylene glycol. Liquid dishwashing detergents are nonionic.
c. It belongs to a nonionic type.

Answer 31.
a. Frenkel defect arises due to the dislocation of a cation from its normal site to an interstitial site.
b. Frenkel defect is shown by crystals with a large difference in size between the anion and cation. In LiCI, the cation and anion have comparable (similar) size. So it will not exhibit Frenkel defect.
c. This is due to metal excess defect due to anionic vacancies. When LiCI is heated in vapors of Li, the Cf ions diffuse to the surface of the crystal creating vacant sites. Instead, the electrons diffuse into the crystal and occupy these anionic sites, forming F centers. These electrons absorb energy from white light and show a pink color.

Answer 32.
a. If the sum of powers of the concentration terms in the rate expression is one, then the reaction is a first order reaction.
Plus Two Chemistry Model Question Papers Paper 2 57

Answer 33.
a. Kohlrausch’s law.
b. It is the molar conductivity of electrolytes at zero concentration or at infinite dilution.
c. The degree of dissociation can be determined by the equation, \(\alpha =\frac { { \lambda }_{ m } }{ { \lambda }_{ 0 }^{ m } }\) To obtain ionisation constant, Ka substitute α in the following equation, \({ K }_{ a }=\frac { { C\alpha }^{ 2 } }{ (1-\alpha ) }\) where C is the concentration a of acetic acid in moles/liter.
d. The conductivity of a solution decreases with a decrease in concentration as the number of ions presents per unit volume of the solution actually decreases. The molar conductivity of electrolytes increases with a decrease in concentration.

Plus Two Chemistry Previous Year Question Papers and Answers

Plus Two Chemistry Model Question Paper 1

Kerala State Board New Syllabus Plus Two Chemistry Previous Year Question Papers and Answers.

Kerala Plus Two Chemistry Model Question Paper 1 with Answers

Board SCERT
Class Plus Two
Subject Chemistry
Category Plus Two Previous Year Question Papers

Time: 2 Hours
Cool off time: 15 Minutes
Maximum: 60 Score

General Instructions to candidates:

  • There is a ‘cool off time’ of 15 minutes in addition to the writing time of 2 hrs.
  • Use the ‘cool off time’ to get familiar with the questions and to plan your answers.
  • Read questions carefully before you answering.
  • Read the instructions carefully.
  • Calculations, figures, and graphs should be shown in the answer sheet itself.
  • Malayalam version of the questions is also provided.
  • Give equations wherever necessary.
  • Electronic devices except non-programmable calculators are not allowed in the Examination Hall.

Questions from 1 – 7 Carry 1 Score each. Answer all the questions. (7 × 1 = 7)

Question 1.
The edge length and axial angles of a unit cell are a = b = c and α = β = γ = 90° respectively. Identify the type of Crystal System.
Answer:
Cubic

Question 2.
Name the polymer used for non-stick surface coated utensils.
Answer:
Polytetrafluoroethene/Teflon

Question 3.
Which of the following forms coloured compounds
i) Sc3+
ii) Cr2+
iii) Zn2+
iv) Cu+
[Atomic number Sc-21, Cr-24, Zn-30, Cu-29]
Answer:
ii) Cr2+

Question 4.
The rate expression of a reaction is Rate = K [A]1/2 [B]1/2. Calculate its overall order.
Answer:
Overall order = \(\frac{1}{2}+\frac{3}{2}\) = 2

Question 5.
Among the following amines, identify the one that does not reach with Hinsberg reagent.
i) C2H5NH2
ii) (CH3)2NH
iii) (C2H5)2NH
iv) (CH3)3N
Answer:
iv) (CH3)3N

Question 6.
The Carbohydrate, that is known as ‘animal starch’ is …………
Answer:
Glycogen

Question 7.
Name the class of drug used for the treatment of stress and mental diseases.
Answer:
Tranquilizers

Questions from 8 – 20 Carry 2 Score each. Answer any 10 questions. (10 × 2 = 20)

Question 8.
Write the names and structures of the monomers of the polymer Buna-S.
Answer:
1, 3-Butadiene CH2=CH-CH=CH2
Styrene/Vinyl benzene C6H5-CH=CH2

Question 9.
Calculate the packing efficiency in body centred cubic structure.
[Given that r = \(\frac{\sqrt{3}}{4}\)a, where ‘r’ is the radius of sphere and ‘a’ is the edge length of unit cell]
Answer:
In body centred cubic (bcc) structure, total number of atoms per unit cell is 2.
Volume of atoms in the unit cell = 2 × (\(\frac{4}{3}\))πr3
Volume of the cube = a= (\(\frac{4}{\sqrt{3}}\)r)3
Plus Two Chemistry Model Question Paper 1, 1

Question 10.
Write one method for the preparation of sulphur dioxide. How is its presence detected?
Answer:
Sulphur dioxide is obtained when sulphur is burnt in air or oxygen.
S(s) + O2(g) → SO2(g)
OR
In the laboratory, it is prepared by treating a sulphite with dilute sulphuric acid.
SO32-(aq) + 2H+(aq) → H2O(l) + SO2(g)
OR
Industrially, it is produced as a by-product of the roasting of sulphide ores.
4FeS2(s) + 11O2(g) → 2Fe2O3(s) + 8SO2(g)
The sulphur dioxide gas decolourises acidified potassium permanganate solution. This is a convenient test for the gas.
5SO2 + 3MnO4 + 2H2O → 5SO42- + 4H+ + 2Mn2+

Question 11.
Write the IUPAC names of following compounds.
a) K3[Fe(CN)6]
b) Co[Cl2(en)2]Cl
Answer:
a) Potassium hexacyanoferrate(III)
b) Dichloridobis(ethane-1, 2-diamine)cobalt(III) chloride

Question 12.
Define van’t Hoff factor ‘i’. What would be the value of ‘i’ for a dilute solution of K2SO4 in water?
Answer:
The van’t Hoff factor, ‘i’ is defined as:
Plus Two Chemistry Model Question Paper 1, 2
Plus Two Chemistry Model Question Paper 1, 3
K2SO4 ionises in water as,
K2SO4 → 2K+ + SO42-
Therefore, for a dilute solution of K2S04 in water the value of i = 2 + 1 = 3

Question 13.
By taking ethyl bromide as starting material, prepare ethyl iodide and ethyl fluoride.
Answer:
Ethyl bromide is treated with Nal in dry acetone to get ethyl iodide. (Finkelstein reaction)
CH3CH2Br + Nal → CH3CH2l + NaBr
Ethyl bromide is heated with metallic fluorides such as AgF, Hg2F2, CoF2 or SbF3 to get ethyl fluoride. (Swarts reaction)
CH3CH2Br + AgF → CH3CH2F + AgBr

Question 14.
Write the complete reaction representing the disproportionation of benzaldehyde.
Answer:
Plus Two Chemistry Model Question Paper 1, 4
This reaction is called Cannizzaro reaction.

Question 15.
Vapour pressure of water at 293 K is 17.535 mm Hg. Calculate the vapour pressure of water at 293 K when 25g of glucose is dissolved in 450 g of water.
Answer:
\(p_{1}^{0}\) = 17.535 mm Hg, w2 = 25g, M2 = 180g mol-1, w1 = 450 g, M1 = 18 g mol-1, p, = ?
Plus Two Chemistry Model Question Paper 1, 5
Therefore, vapour pressure of glucose solution,
p1 = \(p_{1}^{0}\) – 0.09742
= 17.535 – 0.09742 = 17.4376 mm Hg

Question 16.
Name the products of hydrolysis of sucrose and explain why sucrose is not a reducing sugar.
Answer:
Sucrose on hydrolysis gives equimolar mixture of D-(+)-glucose and D-(-)-fructose.
Plus Two Chemistry Model Question Paper 1, 6
In sucrose, the two monosaccharide units (glucose and fructose) are held together by a glycosidic linkage between C1 of α – glucose and C2 of β – fructose. Since the reducing groups of glucose and fructose are involved in glycosidic bond formation, sucrose is a non-reducing sugar.

Question 17.
Tertiary butyl bromide react with aqueous NaOH to give tertiary butyl alcohol proceeds via SN1 reaction. Write the mechanism of the reaction.
Answer:
Plus Two Chemistry Model Question Paper 1, 7
Mechanism: This reaction occurs in two steps:
Step I: The polarised C-Br bond undergoes slow cleavage to produce tert-butyl carbocation and a bromide ion.
Plus Two Chemistry Model Question Paper 1, 8
Step II: The carbocation is attacked by the nucleophile OH to form the product, tert-butyl alcohol.
Plus Two Chemistry Model Question Paper 1, 9

Question 18.
Give the structures of A and B in the following reaction.
Plus Two Chemistry Model Question Paper 1, 10
Answer:
Plus Two Chemistry Model Question Paper 1, 11

Question 19.
Match the following
Plus Two Chemistry Model Question Paper 1, 12
Answer:
Plus Two Chemistry Model Question Paper 1, 13

Question 20.
Write a simple chemical test to distinguish ethanal from propanone.
Answer:
Tollens’ Test – On warming with Tollens’ reagent ethanal gives a bright silver mirror while propanone will not give silver mirror.
OR
Fehling’s Test – On heating with Fehling’s reagent ethanal gives a reddish brown precipitate while propanone do not respond to this test.

Questions from 21 – 29 Carry 3 Score each. Answer any 7 questions. (7 × 3 = 21)

Question 21.
The resistance of a 0.5 M solution of an electrolyte enclosed between two platinum electrodes 1.5 cm apart and having an area of 2.0 cm2 was found to be 30Ω. Calculate the molar conductivity of the solution.
Answer:
M = 0.5 M, l = 1.5 cm, A = 2 cm2, R = 30Ω, \(\Lambda_{m}\) = ?
Plus Two Chemistry Model Question Paper 1, 14

Question 22.
Write notes on different allotropic forms of phosphorous.
Answer:
The important allotropic forms of phosphorus are white, red and black phosphorus.
White phosphorus: white waxy solid, poisonous, insoluble in water but soluble in CS2, glows in dark, dissolves in boiling NaOH solution in an inert atmosphere giving PH3.
P4 + 3NaOH + 3H2O → PH3 + 3NaH2PO2
It readily catches fire in air to give dense white fumes Of P4O10.
P4 + 5O2 → P4O10
It consists of discrete tetrahedral P4 molecules. It is less stable because of angular strain in P4 molecule where the angles are only 60°.
Plus Two Chemistry Model Question Paper 1, 15
Red phosphorus: obtained by heating white P at 573K in an inert atmosphere for several days. This when heated under high pressure, a series of phases of black P is formed. It possesses iron grey lustre. It is odourless, non-poisonous, insoluble in water as well as in CS2, much less reactive than white P, does not glow in dark. It is polymeric, consisting of chains of P4 tetrahedra linked together as shown:
Plus Two Chemistry Model Question Paper 1, 16
Black phosphorus: It has two forms, α -Black P and β -Black P.
α -Black P: It is formed when red P is heated in a sealed tube at 803 K, can be sublimed in air, has opaque monoclinic or rhombohedral crystals, does not oxidise in air.
β -Black P: It is prepared by heating white P at 473K under high pressure. It does not burn in air upto 673 K.

Question 23.
Integrated rate equation for a first order reaction.
K = \(\frac{2.303}{t} \log \frac{\left[R_{0}\right]}{[R]}\)
a) Derive an expression for half life of first order reaction.
b) Find the half life of a first order reaction having rate constant K = 5.5 × 10-14 S-1.
Answer:
a) For a first order reaction R → P, the rate constant
Plus Two Chemistry Model Question Paper 1, 17
Plus Two Chemistry Model Question Paper 1, 18

Question 24.
What are adsorption isotherms? Write the Freundlich adsorption isotherm equation and plot a graph based on it.
Answer:
Adsorption isotherms are curves used to express the variation of the amount of gas adsorbed by the adsorbent with pressure at constant temperature.

The Freundlich adsorption isotherm gives an empirical relationship between the quantity of gas adsorbed by unit mass of solid adsorbent and pressure at a particular temperature. The relationship can be expressed by the equation:
\(\frac{x}{m}=k \cdot P^{1 / n}\)
where x is the mass of the gas adsorbed on mass m of the adsorbent at pressure P, k and n are constants which depend on the nature of the adsorbent and the gas at a particular temperature.

The relationship can be represented in the form of a curve where mass of the gas adsorbed per gram of the adsorbent (x/m ) is plotted against pressure(P).
Plus Two Chemistry Model Question Paper 1, 19

Question 25.
Describe the steps involved in the manufacture of sulphuric acid by contact process.
Answer:
The Contact process for the manufacture of sulphuric acid involves three steps:
Step 1: Burning of sulphur or sulphide ores in air to generate SO2.
S(s) + O2(g) → SO2(g)
Step 2: Conversion of SO2 to SO3 by the reaction with oxygen in the presence of V2O5 catalyst at a pressure of 2 bar and a temperature of 720 K.
Plus Two Chemistry Model Question Paper 1, 20
Step 3: Absorption of SO3 in H2SO4 to give oleum (H2S2O7).
SO3 + H2SO4 → H2S2O7
Dilution of oleum with water gives H2SO4 of the desired concentration.

Question 26.
Differentiate ferrimagnetism from paramagnetism and predict the change that takes place to ferrimagnetic substances on heating.
Answer:
Paramagnetism: Magnetic property which arises due the presence of one or more unpaired electrons. Such substances are weakly attracted by the magnetic field. They are magnetised in a magnetic field in the same direction and lose their magnetism in the absence of magnetic field.
e.g. O2, Cu2+, Fe3+, Cr3+.

Ferrimagnetism: Magnetic property observed when the magnetic moments of the domains in the substance are aligned in parallel and anti-parallel directions in unequal numbers as shown:
Plus Two Chemistry Model Question Paper 1, 21

They are weakly attracted by magnetic field as compared to ferromagnetic substances.
e.g. Fe3O4 (magnetite) and ferrites like MgFe2O4 and ZnFe2O4.
Ferrimagnetic substances on heating lose ferrimagnetism and become paramagnetic.

Question 27.
a) Draw the structure of dichromate ion.
b) How can dichromate ion be converted to chromate ion?
c) Write examples for the oxidising action of potassium dichromate in acidic medium.
Answer:
a)
Plus Two Chemistry Model Question Paper 1, 22

b) When an aqueous solution of dichromate is treated with alkali (i.e., by increasing the pH of the medium) it changes to chromate.
Cr2O72- + 2OH → 2CrO42- + H2O

c) Acidified potassium dichromate oxides iodides to iodine, sulphides to sulphur, tin(II) to tin(IV) and iron(II) to iron(III).
Plus Two Chemistry Model Question Paper 1, 23

Question 28.
Explain the following
a) Hydraulic washing
b) Zone refining
c) Roasting of ores
Answer:
a) Hydraulic washing: This is a method used for the concentration of ores. It is based on the differences in gravities of the ore and the gangue particles. It is type of gravity separation. Here an upward stream of running water is used to wash the powdered ore. The lighter gangue particles are washed away and the heavier ores are left behind.

b) Zone refining: It is a method used for obtaining extra pure germanium, silicon, boron, gallium and indium. It is based on the principle that the impurities are more soluble in the melt than in the solid state of the metal. Here a circular mobile heater is fixed at one end of a rod of the impure metal.

The molten zone moves along with the heater which is moved forward, the pure metal crystallises out of the melt and the impurities pass on into the adjacent molten zone. This process is repeated several times and the heater is moved in the same direction. At one end, impurities get concentrated. This end is cut off.

c) Roasting of ores: Roasting is used for converting ore to its oxide. It is mainly applied to sulphide ores. Here the ore is heated in a regular supply of air in a furnace at a temperature below the melting point of the metal, e.g.
2ZnS + 3O2 → 2ZnO + 2SO2 OR
2PbS + 3O2 → 2PbO + 2SO2 OR
2CU2S + 3O2 → 2Cu2O + 2SO2

Question 29.
Describe the following reactions.
a) Aldol condensation
b) Etard reaction
c) Rosenmund reduction
Answer:
a) Aldol condensation: Aldehydes and ketones having at least one α – hydrogen atom undergo a reaction in the presence of dilute alkali as catalyst to form β – hydroxy aldehydes (aldol) or β – hydroxy ketones (ketol) respectively. The aldol and ketol on heating readily lose water to give α, β – unsaturated aldehydes or ketones respectively.

e.g. Ethanal on treating with dil NaOH gives 3-hydroxy butanal which on heating loses water to give but-2-enal.
Plus Two Chemistry Model Question Paper 1, 24

b) Etard reaction: When toluene is treated with chromyl chloride (CrO2Cl2) in CS2 a chromium complex is formed which on hydrolysis gives benzaldehyde.
Plus Two Chemistry Model Question Paper 1, 25

C) Rosenmund reduction: When an acyl chloride(acid chloride) is hydrogenated over catalyst, palladium on barium sulphate the corresponding aldehyde is formed.
e.g. Benzoyl chloride on hydrogenation over catalyst, palladium on barium sulphate gives
benzaldehyde.
Plus Two Chemistry Model Question Paper 1, 26

Questions from 30 – 33 Carry 4 Score each. Answer any 3 questions. (3 × 4 = 12)

Question 30.
Write chemical equations for the conversion of phenol to acetylsalicylic acid (aspirin).
Answer:
Plus Two Chemistry Model Question Paper 1, 27

Question 31.
With the help of diagram describe the construction and working of a fuel cell using H2 and O2. What are the advantages of fuel cell over conventional cells.
Answer:
In the H2 – O2 fuel cell, hydrogen and oxygen are bubbled through porous carbon electrodes into concentrated aqueous NaOH solution. Catalysts like finely divided platinum or palladium metal are incorporated into the electrodes for increasing the rate of electrode reactions.
Plus Two Chemistry Model Question Paper 1, 28
This cell uses the reaction of hydrogen with oxygen to form water. The electrode reactions are:
Anode reaction: 2H2(g) + 4OH(aq) → 4H2O(l) + 4e
Cathode reaction: O2(g) + 2H2O(l) + 4e → 40H(aq)
The overall reaction is, 2H2(g) + O2(g) → 2H2O(l)

Advantages:

  • The cell runs continuously as long as the reactants are supplied.
  • Fuel cells produce electricity with an efficiency of about 70%, which is much higher than that of conventional cells.
  • Fuel cells are pollution free.

Question 32.
Sketch the Crystal field splitting of d-orbitals of the Central metal ion in [CoF6]3-. Also write the electronic configuration of central metal ion and predict its magnetic behaviour based on Crystal field theory.
Answer:
Plus Two Chemistry Model Question Paper 1, 29
The electronic configuration of the central metal ion. Co3+ is [Ar]3d6 4s0.
Here F- is a weak field ligand. Therefore, Δ0 < P and no pairing occurs (\(t_{2 g}^{4} e_{g}^{2}\)). Thus, there are four unpaired electrons and hence [CoF6]3- is an outer orbital/high spin/spin free paramagnetic octahedral complex.

Question 33.
Give the structures and IUPAC names of the products formed on the following reactions.
a) Hydration of propene in the presence of dilute sulphuric acid.
b) Reaction of methanal with methyl magnesium bromide followed by hydrolysis.
Answer:
Plus Two Chemistry Model Question Paper 1, 30

Plus Two Chemistry Previous Year Question Paper 2017

Kerala Plus Two Chemistry Previous Year Question Paper 2017

Time: 2 Hours
Cool off time: 15 Minutes
Maximum: 60 Scores

General Instructions to Candidates

  • There is a ‘cool off time’ of 15 minutes in addition to the writing time of 2 hrs.
  • You are not allowed to write your answers nor to discuss anything with others during the ‘cool off time’.
  • Use the ‘cool off time’ to get familiar with the questions and to plan your answers.
  • Read questions carefully before you answering.
  • All questions are compulsory and the only internal choice is allowed.
  • When you select a question, all the subquestions must be answered from the same question itself.
  • Calculations, figures, and graphs should be shown in the answer sheet itself.
  • Malayalam version of the questions is also provided.
  • Give equations wherever necessary.
  • Electronic devices except nonprogrammable calculators are not allowed in the Examination Hall.

Plus Two Chemistry Previous Year Question Papers and Answers 2017 1

Question 1.
a. Identify the nonstoichiometric defect
i. Schottky defect
ii. Frenkel defect
iii. interstitial defect
iv. Metal deficiency defect
b. What type of substance could make better permanent magnets ferromagnetic or ferrimagnetic? Justify your answer.
c. In terms of Band theory write the differences between conductors and insulators.
Plus Two Chemistry Previous Year Question Papers and Answers 2017 2
Plus Two Chemistry Previous Year Question Papers and Answers 2017 3

Question 2.
a. Henry’slaw is related to the solubility of a gas in a liquid.
i. State Henry’s law
ii. Write any two applications of Henry’s law.
b. 1000 cm3 of an aqueous solution of a protein contains 1.26 gm of the protein. The osmotic pressure of such a solution at 300 K is found to be 2.57 × 10-3 bar. Calculate the molar mass of the protein. (R = 0.083 L bar mol1K1)
Plus Two Chemistry Previous Year Question Papers and Answers 2017 4

Question 3.
a. Represent the galvanic cell based on the cell reaction given below:
Plus Two Chemistry Previous Year Question Papers and Answers 2017 5
b. Write the half-cell reactions of the above cell.
c. \(\wedge _{ m }^{ 0 }\) for NaCl, HCl and NaAc are 126.4, 425.9 and 91.0 S cm2 mol-1 respectively. Calculate \(\wedge _{ m }^{ 0 }\) for HAc.
Plus Two Chemistry Previous Year Question Papers and Answers 2017 6

Question 4.
a. Plot a graph showing variation in the concentration of reactants against time for a zero order reaction.
b. What do you mean by zero order reaction?
c. The initial concentration of the first order reaction, N2O5(g) → 2NO2(g) + 1/2O2(g) was 1.24 × 10-2 mol L-1 at 300 K. The concentration of N2O5 after ‘ 1 ’ hour was 0.20 × 10-10 mol L-1. Calculate the rate constant of the reaction of 300 k.
Plus Two Chemistry Previous Year Question Papers and Answers 2017 7

Question 5.
There are mainly two types of adsorption. They are physisorption and chemisorption.
a. Differentiate between physisorption and chemisorption.
b. Write any two applications of adsorption.
Plus Two Chemistry Previous Year Question Papers and Answers 2017 8

Question 6.
Leaching is a process of concentration of ores. Explain the leaching of alumina from bauxite.
Plus Two Chemistry Previous Year Question Papers and Answers 2017 9

Question 7.
Nitrogen forms a number of oxides and oxoacids.
a. Which of the following is a neutral oxide of nitrogen?
i. N2O
ii. N2O5
iii. NO2
iv.N2O4
b. Prepare a short write-up on Nitric acid highlighting its laboratory preparation, chemical properties and uses.
OR
Phosphorous forms a number of compounds, a. The gas liberated when calcium phosphide is treated with dil.HC/ is
i. Cl2
ii.H2
iii. PH3
iv. All of the above
b. Prepare a short write up on PCl3 and PCl5 highlighting the preparation and chemical properties of PCl3 and structure of PClr
Plus Two Chemistry Previous Year Question Papers and Answers 2017 11

Question 8.
a. Transition elements are d block elements.
i. Write any four characteristic properties of transition elements.
ii Cr2+ and Mn3+ have d4 configuration. But Cr2+ is reducing and Mn3 is oxidizing. Why?
b. Which of the following is not a lanthanoid element?
i. Cerium
ii. Europium
iii Lutetium
iv. Thorium
Plus Two Chemistry Previous Year Question Papers and Answers 2017 12
Plus Two Chemistry Previous Year Question Papers and Answers 2017 13

Question 9.
[CO(NH3)5S04]Cl and [Co(NH3)Cl]SO4 are coordination compounds.
a. Identify the isomerism shown by the above compounds.
b. Write the IUPAC names of the above compounds.
c. Identify the ligands in each of the above compounds.
Plus Two Chemistry Previous Year Question Papers and Answers 2017 14

Question 10.
a. An ambident nucleophile is
i. Ammonia
ii Ammonium ion
iii Chloride ion
iv. Nitrite ion
b.Haloalkanes and Haloarenes are organohalogen compounds.
i. Suggest a method for the preparation of alkyl chloride.
ii. Aryl halides are less reactive towards Nucleophilic substitution reactions. Give reasons.
Plus Two Chemistry Previous Year Question Papers and Answers 2017 15
Plus Two Chemistry Previous Year Question Papers and Answers 2017 16

Question 11.
a. Arrange the following compounds in the order of increasing boiling points: Ethanol, Propan1 o 1, Butan1 o 1, Butan2ol
b. In the lab, students were asked to carry out the reaction between phenol and cone. HN03. But one student, ‘A’ carried out the reaction between phenol and dil. HNOr Do you think that the student ‘A’ got the same result as others. Substantiate with suitable explanations. [Also write the chemical equations wherever necessary]
Plus Two Chemistry Previous Year Question Papers and Answers 2017 17

Question 12.
a. The product obtained when benzene is treated with carbon monoxide and hydrogen chloride in presence of anhydrous AlCl3 is
i. Chlorobenzene
ii. Phenol
iii. Benzaldehyde
iv. Benzoic acid
b. How will you carry out the following conversions?
Plus Two Chemistry Previous Year Question Papers and Answers 2017 18
OR

b. Explain the following:
i. Esterification
ii. Tollen’s test
iii. HVZ reaction
iv. Dicarboxylic acid
Plus Two Chemistry Previous Year Question Papers and Answers 2017 19
Plus Two Chemistry Previous Year Question Papers and Answers 2017 20

Question 13.
a. Classify the following amines as primary, secondary and tertiary
Plus Two Chemistry Previous Year Question Papers and Answers 2017 21
Identify the products B and C and write their formulae.
Plus Two Chemistry Previous Year Question Papers and Answers 2017 22

Question 14.
a. Which of the following is a polysaccharide?
i. Maltose
ii. Sucrose
iii. Fructose
iv. Cellulose
b. Explain the amphoteric behavior of amino acid.
Plus Two Chemistry Previous Year Question Papers and Answers 2017 23
Plus Two Chemistry Previous Year Question Papers and Answers 2017 24

Question 15.
a. Which of the following is not applicable to Nylon 6, 6?
i. Synthetic polymer
ii. Fiber
iii. Addition polymer
iv. Condensation polymer.
b. Differentiate between thermoplastics and thermosetting plastics. Write one example for each.
Plus Two Chemistry Previous Year Question Papers and Answers 2017 25

Question 16.
‘Antibiotics, antiseptics, and disinfectants are antimicrobial drugs. Explain any one of the above-mentioned drugs with examples.
Plus Two Chemistry Previous Year Question Papers and Answers 2017 26

Answers

Answer 1:
a. Metal deficiency defect
b. Ferromagnetic substances make better permanent magnets because, in the solid state, the metal ions of ferromagnetic substances are grouped together into small regions called domains. Thus, each domain acts as a tiny magnet. In an unmagnetized piece of a ferromagnetic substance, the domains are randomly oriented and their magnetic moments get canceled. When the substance is placed in the magnetic field, a strong magnetic effect is produced. This ordering of domains persists even when the magnetic field is removed and the ferromagnetic substance becomes a permanent magnet. However, this is not possible in ferrimagnetic substances,

c. In conductors, the valence band and conduction band overlap. Hence electrons can easily flow. In insulators the energy gap between the valence band and the conduction band is large, hence electrons cannot jump from one band to the other.

Answer 2.
a. i. It states that “the solubility of a gas in a liquid is directly proportional to the partial pressure of the gas present above the surface of liquid or solution”. The most commonly used form of Henry’s law states that “the partial pressure(p) of gas in the vapor phase is proportional to the mole fraction of the gas(x) in the solution” and it is expressed as, P = KH.X, where KH is Henry’s law constant and ‘x’, is mole fraction of gas in solution.
ii.
1. To increase the solubility of CO2 in soda water and soft drinks, the bottle is sealed under high pressure.
2. To avoid the toxic effects of a high concentration of nitrogen in blood, the tanks used by scuba divers are filled with air diluted with helium.
b. p = 2.57 × 10-3 bar
V = 1000 cm3 = 1L
T = 300k
R = 0.083 L bar mol-1K-1
Plus Two Chemistry Previous Year Question Papers and Answers 2017 27

Answer 3.
Plus Two Chemistry Previous Year Question Papers and Answers 2017 28

Answer 4.
a.
Plus Two Chemistry Previous Year Question Papers and Answers 2017 29
b. In zero order reactions, the rate remains constant throughout the course of reaction i.e., the rate does not change with a change in concentration of reactants.
Plus Two Chemistry Previous Year Question Papers and Answers 2017 30

Answer 5.
a. Physical adsorption (Physisorption):

  • Force of attraction between adsorbate and adsorbent are Van der Waals forces.
  • Lacks specificity.
  • Reversible.
  • Enthalpy of ads 40kJmol-1)
  • Occurs at low temperature.

Chemical adsorption (chemisorption):

  • Force of attraction I and adsorbent I bonds.
  • Highly specific.
  • Irreversible.
  • Enthalpy of advisor 240kJ/mol).
  • Occurs at high terr

b.

  • Production of high
  • Gas masks
  • Heterogenous cat?
  • Froth floatation pr
  • Control of humidity;
  • Chromatographic analysis

Answer 6.
Bauxite ore is treated with aqueous NaOH at 473-523K and 35-36 bar pressure. This way Al2O3 is leached out as sodium aluminate leaving the impurities behind.
Plus Two Chemistry Previous Year Question Papers and Answers 2017 31
The aluminate in solution is neutralized by passing C02 gas and hydrated Al2O3 is precipitated.
Plus Two Chemistry Previous Year Question Papers and Answers 2017 32
The sodium silicate remains in the solution and hydrated alumina is filtered, dried and heated to give back pure Al2O3
Plus Two Chemistry Previous Year Question Papers and Answers 2017 33

Another example: Gold or silver ore is leached with a dilute solution of NaCN or KCN in the presence of air from which the metal is obtained by replacement (using Zn as a reducing agent).

Answer 7.
a. N2O
b. In the laboratory, nitric acid is prepared by heating KNO3 or NaNO3 and concentrated H2SO4 in a glass retort.
Plus Two Chemistry Previous Year Question Papers and Answers 2017 34
On large scale, it is prepared mainly by Ostwald’s process. This method is based upon catalytic oxidation of NH3 by atmospheric oxygen.
Plus Two Chemistry Previous Year Question Papers and Answers 2017 35
Nitric oxide thus formed combines with oxygen giving NO2,
Plus Two Chemistry Previous Year Question Papers and Answers 2017 36
Nitrogen dioxide so formed dissolves in water to give HN03.
Plus Two Chemistry Previous Year Question Papers and Answers 2017 37
NO thus formed is recycled and the aqueous HNO3 can be concentrated by distillation up to 68% by mass. Further concentration to 98% can be achieved by dehydration with concentrated H2SO4. Chemical properties
1. Acidic nature
Plus Two Chemistry Previous Year Question Papers and Answers 2017 38
2. Oxidizing agent behaves as a strong oxidizing agent because of its tendency to give nascent oxygen.
Plus Two Chemistry Previous Year Question Papers and Answers 2017 39
Uses :
Used in the manufacture of various fertilizers, explosives, plastics, fibers, dyestuffs etc. It is used as a laboratory reagent and as an oxidizing agent. It is also used in the purification of silver an gold.
OR
a. (iii) pH3
b. Phosphorus forms two types of halides, PX3 (X = F, Cl, Br, I) and PX5 (X = F, Cl, Br). Preparation
It is obtained by passing dry chlorine overheated white phosphorus. P4 + 6CI2 → 4PCIIt is also obtained by the action of thionyl chloride with white phosphorus. P4 + 8SOCI2 → 4PCI3 + 4SO2 + 2S2CI2 Properties It is a colorless oily liquid and hydrolyzes in the presence of moisture.
PCI3 + 3H2O → H3PO3 + 3HCI It reacts with organic compounds containing OH group such as CH3COOH, C2H5OH.
3CH3COOH + PCI3 → 3CH3COCI + H3PO3 3C2H5OH + PCI3 → 3C2H5CI + H3PO3 It has a pyramidal shape as shown, in which phosphorus is sp3 hybridized. Preparation
Phosphorus pentachloride is prepared by the reaction of white phosphorus with an excess of dry chlorine. P4 + 10Cl2 → 4PCL5
It can also be prepared by the action of SO2CI2 on phosphorus.
P4 + 10SO2CI2 → 4PCI3 + 10SO5Properties PCI5 is a yellowish white powder and in moist air, it hydrolyzes to POCI3 and finally gets converted to phosphoric acid. PCI5 + H2O → POCI3 + 2HCI POCI3 + 3HO → HPO4 + 3HCI In gaseous and liquid phases, it has a trigonal bipyramidal structure as shown. The three equatorial P_CI bonds are equivalent, while the two axial bonds are longer than equatorial bonds.
Plus Two Chemistry Previous Year Question Papers and Answers 2017 41
This is due to the fact that the axial bond pairs suffer more repulsion as compared to equatorial bond pairs. In the solid state, it exists as an ionic solid, [PCI4]+ [PCI6] in which the cation, [PCI4]+ is tetrahedral and the anion, [PCI8] cathedral.

Answer 8.
a. i. The transition elements are those elements which have partially filled subshells in their elementary form or in any one of its oxidation states. They have low ionization energy. They are malleable and ductile. They are good conductors of heat and electricity. High melting and boiling point.
ii. Cr2+ is reducing as its configuration changes from d4 to d3, the latter having a half-filled t2g level. On the other hand, the change from Mn2+ results in the half-filled (d5) configuration which has extra stability,
b. Thorium

Answer 9.
a. Ionization isomerism
b. [C0(NH3)5S04]CI- Pentaamminesulphatocobalt (III) chloride [CO(NH3)5Cl]SO– Pentamminechloridocobalt (III) sulfate
c. Counterion in a complex salt itself a potential ligand. ligands are Ct and SO42-

Answer 10.
a.iv. Nitrite ion
b.i. The preparation of alkyl chloride is carried out either by passing dry hydrogen chloride gas through a solution of alcohol or by heating a solution of alcohol in concentrated aqueous acid.
Plus Two Chemistry Previous Year Question Papers and Answers 2017 42
ii.
1. Aryl halides are electron rich.
2. In aryl halides, the halogen is attached to an sp2 hybridized C, which is highly stable.
3. Because of the possible repulsion, it is less likely for the electron-rich arenes.
4. In the case of haloarenes, the phenyl cation formed as a result of self-ionization will not be stabilized by resonance and therefore, SN, a mechanism is ruled out.

Answer 11.
a. Ethanol < Propane -l-ol < Butan 2-ol < Butan -1-ol.
b. They don’t get the same result. Phenol with dilute nitric acid at low temperature (298 K) phenol yields a mixture of ortho and para nitrophenols.
Plus Two Chemistry Previous Year Question Papers and Answers 2017 43
Phenol with concentrated nitric acid, phenol is converted to 2, 4, 6 trinitrophenol. The product is commonly known as picric acid. The yield of the reaction product is poor.
Plus Two Chemistry Previous Year Question Papers and Answers 2017 44

Answer 12.
a. (iii) Benzaldehyde
Plus Two Chemistry Previous Year Question Papers and Answers 2017 45
Plus Two Chemistry Previous Year Question Papers and Answers 2017 46
OR
b.
i. Carboxylic acids react with alcohols in the presence of a strong acid catalyst like H2SO4 to form esters. The reaction is reversible and the forward reaction is called esterification,
Plus Two Chemistry Previous Year Question Papers and Answers 2017 47
ii. On warming an aldehyde with freshly prepared ammoniacal silver nitrate solution (Tollen’s reagent) a bright silver mirror is produced due to the formation of silver metal. The aldehydes are oxidized to corresponding carboxylate anion. The reaction occurs in alkaline medium.
Plus Two Chemistry Previous Year Question Papers and Answers 2017 48
iii. HVZ reaction
Carboxylic acids having an α – hydrogen is halogenated at the α – position on treatment with chlorine or bromine in the presence of small amount of red phosphorous to give α – halocarboxylic acids. The reaction is known as Hell Volhard Zelinsky reaction.
iv. Carboxylic acids lose carbon dioxide to form hydrocarbons when their
Plus Two Chemistry Previous Year Question Papers and Answers 2017 49
Sodium salts are heated with soda lime (NaOH and CaO in the ratio of 3:1). The reaction is known as decarboxylation.

Answer 13.
Plus Two Chemistry Previous Year Question Papers and Answers 2017 50

Answer 14.
a.iv. Cellulose
b. In aqueous solution, the carboxyl group can lose a proton and amino group can accept a proton, giving rise to a dipolar ion known as twitter ion. This is neutral but contains both positive and negative charges. In winter ionic form amino acids show amphoteric behavior as they react both with acids and bases.

Answer 15.
a.iii. Addition polymer
b. Thermoplastic polymers These are the linear or slightly branched long chain molecules capable of repeatedly softening on heating and hardening on cooling. These polymers possess intermolecular forces of attraction intermediate between elastomers and fibers. Some common thermoplastics are polyethylene, polystyrene, polyvinyls, etc.

Thermosetting polymers These polymers are cross-linked or heavily branched molecules, which on heating undergo extensive cross-linking in molds and again become infusible. These cannot be reused. Some common examples are bakelite, urea formaldehyde resins, etc

Answer 16.
Antibiotics: These are chemical substances which are produced wholly or partly by chemical synthesis, which in low concentration inhibits the growth or destroys microorganisms by intervening in their metabolic processes, eg., penicillin, streptomycin, ampicillin, tetracycline. Antibiotics are of two types:
Bactericidal antibiotics: They have killing (cidal) effect on microbes, eg., pencil in, ofloxacin, etc.
Bacteriostatic antibiotics: They have an inhibitory (static) effect on microbes, eg., Tetracycline, chloramphenicol, etc.
Antibiotic spectrum: The range of microorganisms attacked by an antibiotic is called its spectrum.

  • Antibiotics which kill or inhibit a wide range of Gram-positive and Gram-negative bacteria are called broad-spectrum antibiotics.
    eg., Ampicillin, tetracycline, ofloxacin.
  • Those which are effective mainly against Gram-positive or Gram-negative bacteria are narrow spectrum antibiotics.
    eg., penicillin G.

Plus Two Chemistry Previous Year Question Papers and Answers

Plus Two Chemistry Previous Year Question Paper March 2018

Kerala State Board New Syllabus Plus Two Chemistry Previous Year Question Papers and Answers.

Kerala Plus Two Chemistry Previous Year Question Paper March 2018 with Answers

Board SCERT
Class Plus Two
Subject Chemistry
Category Plus Two Previous Year Question Papers

Time: 2 Hours
Cool off time: 15 Minutes
Maximum: 60 Score

General Instructions to candidates:

  • There is a ‘cool off time’ of 15 minutes in addition to the writing time of 2 hrs.
  • Use the ‘cool off time’ to get familiar with the questions and to plan your answers.
  • Read questions carefully before you answering.
  • Read the instructions carefully.
  • Calculations, figures, and graphs should be shown in the answer sheet itself.
  • Malayalam version of the questions is also provided.
  • Give equations wherever necessary.
  • Electronic devices except non-programmable calculators are not allowed in the Examination Hall.

(Questions 1 to 7): Carry one score each. Answer all questions. (Scores: 7 × 1 = 7)

Question 1.
What is the co-ordination number of particles present in FCC crystal structure?
Answer:
12

Question 2.
Identify the order of reaction if the unit of rate constant is mol L-1s-1.
Answer:
Zero order

Question 3.
What is the structure of chromate ion ((CrO4)2-)?
Answer:
Tetrahedral
OR
Plus Two Chemistry Previous Year Question Paper March 2018, 1

Question 4.
Name the test used to identify primary amines using CHCl3 and ethanolic KOH.
Answer:
Carbylamine test/reaction or Isocyanide test

Question 5.
Which among the given vitamins is water soluble?
a) A
b) B
c) D
d) E
Answer:
b) B

Question 6.
What is the crosslinked polymer obtained by the polymerisation of phenol and formaldehyde?
Answer:
Bakelite

Question 7.
……….. is an artificial sweetener which is unstable at cooking temperature.
Answer:
Aspartame

(Questions 8 to 20): Answer any ten. Each question carries two scores. (Scores: 10 × 2 = 20)

Question 8.
a) Based on the nature of intermolecular forces, classify the following solids:
i) SiO2
ii) Ice
b) ZnO turns yellow on heating. Why?
Answer:
a) i) SiO2 – Covalent/Network solid
ii) Ice – Molecular slod/Hydrogen bonded molecular solid.

b) This is due to metal excess defect caused by the presence of extra cations at interstitial sites. ZnO is white in colour at room temperature. On heating it loses oxygen as,
Plus Two Chemistry Previous Year Question Paper March 2018, 2
The excess Zn2+ ions move to interstitial sites and electrons to neighbouring interstitial sites which imparts yellow colour to the crystal.

Question 9.
A solution contains 15g urea (molar mass = 60 g mol-1) per litre of solution in water has the same osmotic pressure as a solution of glucose (molar mass =180 g mol-1) in water. Calculate the mass of glucose present in one litre of its solution.
Answer:
πV = nRT
or π = \(\frac{n R T}{V}\)
Let π1 be the osmotic pressure of urea and π2 that of glucose. Then,
Plus Two Chemistry Previous Year Question Paper March 2018, 3
Here the two solutions are isotonic i.e., have same osmotic pressure.
Therefore, π1 = π2
Plus Two Chemistry Previous Year Question Paper March 2018, 4

Question 10.
Define minimum boiling azeotropes with example.
Answer:
Azeotropes are binary mixtures having the same composition in liquid and vapour phase and boil at a constant temperature. The solutions which show large positive deviation from Raoult’s law form minimum boiling azeotrope at a specific composition.
e.g. a 95% (v/v) solution of ethanol in water.

Question 11.
Write the chemical equation of the following reactions:
a) Preparation of XeO3 from XeF6.
b) Mixing PtF6 and Xe.
Answer:
a) XeF6 + 3H2O → XeO3 + 6HF
b) Xe + PtF6 → Xe + PtF6

Question 12.
Explain how the complexes of nickel, [Ni(CN)4]2 and [Ni(CO)4] have different structures, but do not differ in their magnetic behaviour. (Ni, Atomic No: 28)
Answer:
In [Ni(CN)4]2-, Ni (3d8 4s2) is in +2 oxidation state (Ni2+ – 3d8) and it undergoes dsp2 hybridisation. So the ion has square planar geometry. But in [Ni(CO)4], Ni is in 0 oxidation state and it undergoes sp3 hybridisation. So it has tetrahedral geometry. Due to the absence of unpaired electrons both the complexes are diamagnetic.

Question 13.
Complete the reaction:
Plus Two Chemistry Previous Year Question Paper March 2018, 5
Answer:
Plus Two Chemistry Previous Year Question Paper March 2018, 6

Question 14.
During the β-elimination reaction of 2-bromopentane in an alcoholic solution of KOH results Pent-2-ene as major product and Pent-1-ene as minor product. State the rule to explain the reaction.
Answer:
Zaitsev rule/Saytzeff rule. The rule states that “in dehydrohalogenation reactions, if there is a possibility of formation of more than one alkene, the preferred product is that alkene which has the greater number of alkyl groups attached to the doubly bonded carbon atoms.”

Question 15.
Aromatic aldehydes undergo electrophilic substitution reactions. Write the nitration reaction of benzaldehyde with chemical equation.
Answer:
The -CHO group being a meta-directing group, benzaldehyde on nitration gives m-nitrobenzaldehyde.
Plus Two Chemistry Previous Year Question Paper March 2018, 7

Question 16.
Briefly describe Gatterman Koch reaction.
Answer:
Gatterman – Koch reaction: When benzene or its derivative is treated with carbon monoxide and hydrogen chloride in the presence of anhydrous aluminium chloride or cuprous chloride, it gives benzaldehyde or substituted benzaldehyde.
Plus Two Chemistry Previous Year Question Paper March 2018, 8

Question 17.
How can it convert methyl iodide to ethanamine?
Answer:
Methyl iodide when treated with KCN or NaCN gives methyl cyanide or ethane nitrile which on reduction with LiAlH4 or H2 in presence of Ni/Pt/Pd or by sodium amalgam and ethanol we get ethanamine.
Plus Two Chemistry Previous Year Question Paper March 2018, 9

Question 18.
State two differences between globular and fibrous proteins.
Answer:
Globular proteins:

  1. Polypeptide chains coil around.
  2. Spherical shape.
  3. Soluble in water.

Fibrous proteins:

  1. Polypeptide chains run parallel and held together by hydrogen and disulphide bonds.
  2. Fibre-like structure.
  3. Insoluble in water.

Question 19.
Match the following:

a) Polyacrylonitrile i) Terylene
b) 1,3-Butadien-Acrylonitrile ii) Natural Rubber
c) Ethylene glycol-Terephthalic acid iii) Buna-N
d) Cis-1, 4-polyisoprene iv) Acrilan

Answer:

a) Polyacrilonitrile iv) Acrilan
b) 1,3-butadiene-Acrylonitrile iii) Buna-N
c) Ethylene glycol-Terephthalic acid i) Terylene
d) cis-1, 4-polyisoprene ii) Natural Rubber

Question 20.
a) What are drugs?
b) Write an example for a drug classified based on its chemical structure.
Answer:
a) Drugs are chemicals of low molecular mass (~ 100 – 500u) which interact with macro molecular targets and produce a biological response.
b) Sulphonamides or Sulpha drugs like arsphenamine/salvarsan, prontosil, sulphapyridine.

(Questions 21 to 29): Answer any seven. Each question carries three scores: (Scores: 7 × 3 = 21)

Question 21.
An element crystallises as FCC with density 2.8 gm3. Its unit cell having edge length 4 × 10-8 cm. Calculate the molar mass of the element. (Given NA = 6.022 × 1023 mol-1)
Answer:
d = 2.8 g cm-3, a = 4 × 10-8 cm, z = 4 (since unit cell is FCC), NA = 6.022 × 1023 mol-1.
Density, d = \(\frac{\mathrm{zM}}{\mathrm{a}^{3} \mathrm{~N}_{\mathrm{A}}}\)
Molar mass of the element, M = \(\frac{\mathrm{da}^{3} \mathrm{~N}_{\mathrm{A}}}{\mathrm{z}}\)
Plus Two Chemistry Previous Year Question Paper March 2018, 10

Question 22.
Write the anode and cathode reactions occur in the operation of a lead storage battery. Mention the electrolyte used in the battery.
Answer:
Anode reaction: Pb(s) + SO42- (aq) → PbSO4(s) + 2e
Cathode reaction: PbO2(s) + SO42- (aq) + 4H+(aq) + 2e → PbSO4(s) + 2H2O(l)
Electrolyte: 38% solution of sulphuric acid.

Question 23.
For hydrolysis of methyl acetate in aqueous solution, the following results were observed.

t/s 0 30 60
CH3COOCH3 C/mol L-1 0.60 0.30 0.15

Show that it follows pseudo first order reaction as the concentration of water remains constant.
Answer:
As the concentration of water remains constant, practically the reaction should be pseudo first order with respect to methyl acetate.

The rate constant for a pseudo first order reaction,
Plus Two Chemistry Previous Year Question Paper March 2018, 11
From the above results it can be seen that k = k’ [H2O] is a constant and hence, it is a pseudo first order reaction.

Question 24.
a) State Hardy-Schulze rule with the help of example.
b) Why lyophilic colloids are used as protective colloids?
Answer:
a) The Hardy-Schulze rule states that the greater the valence of the flocculating ion added, the greater is its power to cause precipitation.
For example, in the coagulation of a negative sol like AS2S3, the flocculating power is in the order: Al3+ > Ba2+ > Na+
Or, in the coagulation of a positive sol like Fe2O3.xH2O, the flocculating power is in the order:
[Fe(CN)6]4- > PO43- > SO42- > Cl

b) Lyophilic colloids have a unique property of protecting lyophobic colloids. When a lyophilic sol is added to the lyophobic sol, the lyophilic particles form a layer around lyophobic particles and thus protect the lyophobic sols from electrolysis.

Question 25.
Gibbs energy of formation (ΔfG) of MgO(s) and CO(g) at 1273 K and 2273 K are given below:
fG) [MgO(s)]: -941 kJ mol-1 at 1273 K
Plus Two Chemistry Previous Year Question Paper March 2018, 12
On the basis of the above data, predict the temperature at which carbon can be used as a reducing agent for MgO(s)
Answer:
As per the Ellingham diagram, for using carbon as a reducing agent for MgO(s), the value of ΔrG for the coupled reaction, MgO(s) + C(s) → Mg(s) + CO(g) should be negative at the required temperature.
Given that, at 1273 K
i) Mg(s) + \(\frac{1}{2}\)O2(g) → MgO(s); ΔfG = -941 kJ mol-1
ii) C(s) + \(\frac{1}{2}\)O2(g) → CO(g); ΔfG = -439 kJ mol-1
On reversing equation (i) and adding with equation (ii), we get
MgO(s) + C(s) → Mg(s) + CO(g); ΔrG = (941 + – 439) kJ mol-1 = +502 kJ mol-1
Since ΔrG is positive, carbon cannot be used as a reducing agent for MgO(s) at 1273 K.
Given that, at 2273 K
iii) Mg(s) + \(\frac{1}{2}\)O2(g) → MgO(s); ΔfG = -314 kJ mol-1
iv) C(s) + \(\frac{1}{2}\)O2(g) → CO(g); ΔfG = -628 kJ mol-1
On reversing equation (iii) and adding with equation (iv), we get
MgO(s) + C(s) → Mg(s) + CO(g); ΔrG = (314 + – 628) kJ mol-1 = -314 kJ mol-1
Since ΔrG is negative, carbon can be used as a reducing agent for MgO(s) at 2273 K.

Question 26.
a) What is the formula of the phosphine?
b) How phosphine is prepared in laboratory?
Answer:
a) PH3
b) In the laboratory, phosphine is prepared by heating white phosphorus with concentrated NaOH solution in an inert atmosphere of CO2.
P4 + 3NaOH + 3H2O → PH3 + 3NaH2PO2
(Or, any other laboratory method of preparation of PH3.)

Question 27.
Assign the possible reason for following:
a) Stability of +5 oxidation state decreases and that of +3 oxidation state increases down to 15th group elements.
b) H2O is less acidic than H2S.
c) H3PO2 act as a good reducing agent while H3PO4 does not.
Answer:
a) It is due to inert pair effect. In the case of heavier elements the lowest oxidation states are more prodominent due to non-participation of the s-electrons in the formation of bonds.

b) In the case of the hydrides (H2E) of group 16 elements, down the group the H-E bond dissociation enthalpy decreases and hence acidic character increases. Due to small size of O compared to that of S, the O-H bond dissociation enthalpy is greater than the S-H bond dissociation enthalpy. Hence, H2O is less acidic than H2S.

c) The reducing property of oxoacids of phosphorus depends on the P-H bond. The acids which contain P-H bond have strong reducing properties. Thus, H3PO2 is a good reducing agent as it contains two P-H bonds. H3PO4 has no P-H bonds and hence it has no reducing property.
Plus Two Chemistry Previous Year Question Paper March 2018, 13

Question 28.
Give reasons for the following:
a) Transition metals and many of their compounds act as catalyst.
b) Scandium (Z = 21) does not exhibit variable oxidation state and yet it is regarded as a transition element.
c) Write the step involved in the preparation of Na2CrO4 from chromite ore.
Answer:
a) Transition metals and their compounds are known for their catalytic activity. This is due to their ability to adopt multiple oxidation states and to form complexes. Thus they can form with lower activation energy for the reaction. Also because the transition metal ions can change their oxidation states, they become more effective as catalysts.

In some cases transition metals provide a suitable surface for the reaction to take place. The reactants are adsorbed on the surface of the catalyst where reaction occurs. This has the effect of increasing the concentration of the reactants at the catalyst surface and also weakening of the bonds in the reacting molecules.

b) Scandium is regarded as a transition element because it has incompletely filled 3d orbitals in the ground state of the atom (3d1 4s2).

c) The chromite ore is fused with sodium carbonate or potassium carbonate in free access of air.
4FeCr2O4 + 8Na2CO+ 7O2 → 8Na2CrO4 + Fe2O+ 8CO2

Question 29.
How would you account for the following:
a) Aldehydes are more reactive than ketones towards nucleophilic addition reaction.
b) Boiling point of aldehydes are lower than alcohols.
c) Addition reaction of sodium hydrogen sulphite is useful for separation and purification of aldehydes.
Answer:
a) This is due to steric and electronic reasons.
Sterically, the presence of two relatively large substituents in ketones hinders the approach of nucleophile to carbonyl carbon that in aldehydes having only one such substituent.

Electronically, the two alkyl groups reduce the electrophilicity of the carbonyl carbon more effectively in ketones than in aldehydes.

b) The boiling points of aldehydes are lower than those of alcohols of comparable molecular masses due to the absence of intermolecular hydrogen bonding in aldehydes where as alcohols can associate through intermolecular hydrogen bonding and have higher boiling points.

c) The hydrogen sulphite addition compound is water soluble and can be converted back to the original aldehyde by treating it with dilute mineral acid or alkali. Therefore, these are useful for separation and purification of aldehydes.

(Questions 30 to 33): Answer any three. Each question carries four scores: (Score: 3 × 4 = 12)

Question 30.
a) What are primary batteries?
b) The cell potential of a mercury cell is 1.35 V, and remain constant during its life. Give reason.
c) Write the equation of the reactions involved at each electrode in a H2-O2 fuel cell.
Answer:
a) Batteries in which the reaction occurs only once and after use over a period of time they become dead and cannot be reused again.
b) This is because the overall reaction does not involve any ion in solution whose concentration can change during its life time.
c) Anode reaction: 2H2(g) + 4OH– (aq) → 4H2O(l) + 4e
Cathode reaction: O2(g) + 2H2O(l) + 4e → 4OH– (aq)

Question 31.
a) Draw the structures of geometrical isomers of [Fe(NH3)2(CN)4]
b) Write the formula of pentaammine carbonate cobalt (III) chloride.
c) Wife any two limitations of valance bond theory.
Answer:
a)
Plus Two Chemistry Previous Year Question Paper March 2018, 14

b) [Co(NH3)5(CO3)]Cl

c) i) It involves a number of assumptions.
ii) It does not give quantitative interpretation of magnetic data.
iii) It does not explain the colour exhibited by coordination compounds.
iv) It does not give a quantitative interpretation of the thermodynamic or kinetic stabilities of coordination compounds.
v) It does not make exact predictions regarding the tetrahedral and square planar structures of 4-coordinate complexes
vi) It does not distinguish between weak and strong ligands.
(Any two limitations)

Question 32.
a) Grignard reagents are important class of organometallic compounds used to prepare alcohols. Identify the compounds A and B and write the formula.
Plus Two Chemistry Previous Year Question Paper March 2018, 15
Plus Two Chemistry Previous Year Question Paper March 2018, 16
b) Write the name of products formed when salicylic acid is treated with acetic anhydride in acid medium
Answer:
a) i) Compound A : Ethanol/Ethyl alcohol – CH3CH2OH
ii) Compound B : Ethanal/Acetaldehyde – CH3CHO
b) Acetylsalicylic acid/Aspirin and Acetic acid/Ethanoic acid

Question 33.
Lucas test is used to identify primary, Secondary and tertiary alcohols.
a) Explain the process.
b) Name the reagents used in the test.
Answer:
a) Lucas test – When treated with Lucas reagent, tertiary alcohols give turbidity immediately, secondary alcohols give turbidity within 5 minutes while primary alcohols do not give turbidity at room temperature but only on heating. This test is based upon the relative reactivities of various alcohols towards Lucas reagent. The order of reactivity is, 3° alcohols > 2° alcohols > 1° alcohols. Alcohols are soluble in Lucas reagent and form a clear solution. On reaction, alkyl chlorides are formed which being insoluble result in the development of turbidity,

b) Lucas reagent is a mixture of conc. HCl and anhydrous ZnCl2.

Plus Two Physics Model Question Paper 5

Kerala Plus Two Physics Model Question Paper 5

Time: 2 Hours
Cool off time : 15 Minutes

General Instructions to candidates:

  • There is a ‘cool off time’ of 15 minutes in addition to the writing time of 2 hrs.
  • Your are not allowed to write your answers nor to discuss anything with others during the ‘cool off time’.
  • Use the ‘cool off time’ to get familiar with the questions and to plan your answers.
  • Read questions carefully before you answering.
  • All questions are compulsory and only internal choice is allowed.
  • When you select a question, all the sub-questions must be answered from the same question itself.
  • Calculations, figures and graphs should be shown in the answer sheet itself.
  • Malayalam version of the questions is also provided.
  • Give equations wherever necessary.
  • Electronic devices except non programmable calculators are not allowed in the Examination Hall.

Plus Two Physics Previous Year Question Papers and Answers 2018

Speed of light, c = 3 x 108m/s
Mass of proton, mp = 1.6726 x 10-27 kg
Mass of neutron, mn = 1.6749 x 10-27 kg
Planck’s constant, h = 6.626 x 10-34 Js
1eV=1.6 x 10-19 J

Questions 1-7 carry 1 score each. Answer any six questions.
Plus Two Physics Model Question Papers Paper 5 1.

Question 1.
Write the expression for intensity of electric field near to the surface of a charged conductor with uniform charge density σ.
Plus Two Physics Model Question Papers Paper 5 1

Question 2.
The unit of electrical resistivity is…………
Plus Two Physics Model Question Papers Paper 5 2

Question 3.
E.m.f can be induced in a coil placed in an external magnetic field by:
a. Changing the intensity of magnetic field
b. Changing the area of coil
c. Changing the orientation of the coil
d. All of the above
Plus Two Physics Model Question Papers Paper 5 3

Question 4.
Write the energy and momentum associated with a moving photon.
Plus Two Physics Model Question Papers Paper 5 4

Question 5.
The value of relative permeability of a dia magnetic material is;
Plus Two Physics Model Question Papers Paper 5 5

Question 6.
The electric field amplitude of an electromagnetic wave is 15V/m. Find the magnetic field amplitude of the wave.
Plus Two Physics Model Question Papers Paper 5 6

Question 7.
A convex lens is placed in a medium having refractive index greater than that of the lens. The lens now behaves as;
a. Converging lens
b. Diverging lens
c. Plane glass plate
d. None of the above
Plus Two Physics Model Question Papers Paper 5 7

Questions 8 to 15 carry 2 score each. Answer any 7 questions
Plus Two Physics Model Question Papers Paper 5 8.

Question 8.
a. ‘Electrostatic field is always normal to the surface of a charged conductor’. Justify the statement.
b. The value of electric potential at the surface of a charged conductor is 10 V. Find the value of intensity of electric field and potential at a point interior to it
Plus Two Physics Model Question Papers Paper 5 8
Plus Two Physics Model Question Papers Paper 5 8.1

Question 9.
A galvanometer with a coil of resistance 12 Ω shows a lull scale deflection for a current of 2.5 mA. How can it be converted into an ammeter of range 7.5 A?
Plus Two Physics Model Question Papers Paper 5 9

Question 10.
The curves shown in figure are drawn for different magnetic materials. Among the three curves, name the curve that
Plus Two Physics Model Question Papers Paper 5 10
a. Represent the material usually used for making permanent magnets.
b. Represent the material usually used for making electromagnets.
Plus Two Physics Model Question Papers Paper 5 10.1

Question 11.
A capacitor C, a variable resistance R and a bulb B are connected in series to a.c mains in the circuit as shown. The bulb glows with some brightness. How the glow of the bulb change if.
Plus Two Physics Model Question Papers Paper 5 11
a. A dielectric slab is introduced between the plates of the capacitor
b. The resistance R is increased keeping the same capacitance
Plus Two Physics Model Question Papers Paper 5 11.1

Question 12.
James Clerk Maxwell modified Ampere’s circuital theorem by introducing the concept of displacement current.
a. What do you mean by displacement current?
b. Write down the equation for displacement current.
Plus Two Physics Model Question Papers Paper 5 12

Question 13.
An object AB is kept in front of a concave mirror as shown in figure.
Plus Two Physics Model Question Papers Paper 5 13
Plus Two Physics Model Question Papers Paper 5 13.1
a. Complete the ray diagram showing the image formation of object
b. How will the position and intensity of image be affected if the lower half of the mirror’s reflecting surface is painted black?
Plus Two Physics Model Question Papers Paper 5 13.2

Question 14.
Assuming that the two diodes D1 and D2 used in the electric circuit as shown in figure are ideal. Find out the value of current flowing through 2.5 Ω resistor.
Plus Two Physics Model Question Papers Paper 5 14

Question 15.
a. Mention the function of the following used in the communication system
i. Transducer
ii. Transmitter
b. Figure shows the block diagram of a AM transmitter. Identify the boxes X and Y.
Plus Two Physics Model Question Papers Paper 5 15
Plus Two Physics Model Question Papers Paper 5 15.1

Questions 16 to 22 carry 3 scores each. Answer any 6 questions.
Plus Two Physics Model Question Papers Paper 5 16.

Question 16.
Three capacitors of capacitances 2µF, 3µF and 4µF are connected in series.
a. Find the equivalent capacitance of the combination.
b. The plates of a parallel plate capacitor have an area 20 cm² each are separated by a distance of 2.5 mm. The capacitor is charged by connecting it to a 400V supply. How much electrostatic energy is stored in the capacitor?
Plus Two Physics Model Question Papers Paper 5 16

Question 17.
A circuit using potentiometer and a battery of. negligible internal resistance is set up as shown to develop a constant potential gradient along the wire PQ. Two cells of e.m.f s E1 and E2 are connected in series as in figure in combination 1 and 2. The balance points are obtained respectively at 400 cm and 240 cm from point P. Find
Plus Two Physics Model Question Papers Paper 5 17
Plus Two Physics Model Question Papers Paper 5 17.1

Question 18.
A conducting rod PQ of length ‘ l ’ connected to a resistance ‘R’ is moved at a uniform speed ‘ V’, normal to a uniform magnetic field ‘B’
Plus Two Physics Model Question Papers Paper 5 18
a. Deduce the expression for e.m.f induced in the conductor
b. Find the magnitude and direction of current through the conductor
Plus Two Physics Model Question Papers Paper 5 18.1

Question 19.
In tuner circuits, we use the phenomenon of resonance.
a. Write the condition of resonance in series LCR circuit
b. A series LCR circuit uses L = 0. 1 H,C = 10µF and R = 100Ω. Find the value of frequency at which the amplitude of current is maximum
Plus Two Physics Model Question Papers Paper 5 19

Question 20.
The focal length of a lens has dependence on its radii of curvatures and refractive index. Derive Lens maker’s formula
Plus Two Physics Model Question Papers Paper 5 20

Question 21.
a. Write the expression for the de Broglie wavelength associated with a charged particle having charge ‘q’ and mass ‘m’, when it is accelerated by a potential of ‘V’ volts.
b. A proton and an electron have same kinetic energy. Which one has greater value of de Broglie wavelength and why?
Plus Two Physics Model Question Papers Paper 5 21

Question 22.
Find the binding energy per nucleon of 2040Ca nucleus. Given m 2040Ca = 39.962589u.
mp = 1.00783u, mn = 1.00867u
Take 1 amu = 931 MeV/c2
Plus Two Physics Model Question Papers Paper 5 22

Questions from 23-26 carry 4 score. Answer any 3 questions.
Plus Two Physics Model Question Papers Paper 5 22.

Question 23.
Gauss’s theorem is useful for finding the intensity of electric field.
a. Write the Gauss’s law in its mathematical form
b. Using the law, prove that intensity of electric field at a point due to a uniformly charged infinite plane sheet is independent of the distance from it
Plus Two Physics Model Question Papers Paper 5 23

Question 24.
a. Express Ohm’s law in terms of current density, electrical resistivity and intensity of electric field
b. Explain the variation of resistance of a semiconductor with temperature. Also draw the graph showing the variation of resistivity of silicon with temperature
Plus Two Physics Model Question Papers Paper 5 24

Question 25.
The relation between magnetic field and current is given by Biot-Savart’s law.
a. Write the expression for the magnetic field at point along the axis of a circular loop of radius ‘R’ carrying a current ‘I’.
b. From the above expression. Find the value of magnetic field at the centre of the loop
c. Sketch the magnetic field lines for current carrying circular loop
Plus Two Physics Model Question Papers Paper 5 25

Question 26.
Plus Two Physics Model Question Papers Paper 5 26
a. Identify the diode in the circuit and write the use of the resistance Rs
b. Explain how the diode helps as to stabilize the output voltage of the circuit
c. Name the type of biasing used in this diode
Plus Two Physics Model Question Papers Paper 5 26.1
Plus Two Physics Model Question Papers Paper 5 26.2

Questions 27 to 29 carry 5 scores. Answer any 27 – 29 carry 5 score. Answer any 2 questions
Plus Two Physics Model Question Papers Paper 5 27

Question 27.
In the figure, PQ is the incident ray on the equilateral glass prism ABC.
Plus Two Physics Model Question Papers Paper 5 27.1
a. Complete the ray diagram showing the passage of light and mark angle of deviation.
b. Derive an expression for the refractive index of the material of the prism
Plus Two Physics Model Question Papers Paper 5 27.2

Question 28.
According to Christian Huygens wave theory, light emanating from a source as wave fronts.
a. What is the shape of wave front emerging form a linear source?
b. Derive the mathematical expression for the bandwidth of interference bands obtained in young’s double slit experiment with the help of suitable diagram
Plus Two Physics Model Question Papers Paper 5 28

Question 29.
a. The radius of nth stationary orbit of hydrogen atoms is :
Plus Two Physics Model Question Papers Paper 5 29
Using Bohr postulates, obtain an expression for the energy of electron in the stationary states of H-atom
Plus Two Physics Model Question Papers Paper 5 29.
b. Draw the energy level diagram showing how the spectral lines corresponding to Balmer series occur due to transition between the energy levels
Plus Two Physics Model Question Papers Paper 5 29.1

Answers

Answer 1.
\(E=\frac { \sigma }{ { \varepsilon }_{ 0 } } \)

Answer 2.
Ωm

Answer 3.
(d)All the above

Answer 4.
E = hυ = bc/λ
\(P=\frac { h }{ \lambda } =\frac { h\upsilon }{ c } \)

Answer 5.
µr < 1 or µr = 0

Answer 6.
Plus Two Physics Model Question Papers Paper 5 6

Answer 7.
b. Diverging lens

Answer 8.
a. If E is not normal, there is a component a long the tangent to the surface. Then free charge on the surface experience force and will move. But E should have no tangential component in static situation.
OR
V is constant on the surface, therefore E should be normal.
b. E = 0, V = 10V

Answer 9.
G = 12V, Ig = 2.5×10-3 A
I = 7.5 A
Plus Two Physics Model Question Papers Paper 5 9
Putting the low resistance S in parallel with the galvanometer

Answer 10.
a. A
b. C

Answer 11.
Plus Two Physics Model Question Papers Paper 5 11
a. Er introduced, capacitance increases i.e,.\(\frac { 1 }{ c\omega } \) will decrease ∴ I0 will be increase so the brightness will be increased
b. R ↑, I↓ brightness ↓
resistance cause decrease in I0 ∴ bright ness will be decreased

Answer 12.
a. Displacement current is current due to time varying electric flax
Plus Two Physics Model Question Papers Paper 5 12

Answer 13.
Plus Two Physics Model Question Papers Paper 5 13
b.No change in position intensity halved/reduced

Answer 14.
D2 on reverse biasing so it does not conduct
D1 on forward biasing so it conduct
R = 3 + 2.5 = 5.5
V = 10V
I = \(\frac { V }{ R }\) = \(\frac { 10 }{ 5.5 }\) = l.82A

Answer 15.
a. i. Convert energy from one form to an other.
ii. Convert message signal to modulated signal for transmission.
b. X → modulater
Y → Power amplifier

Answer 16.
Plus Two Physics Model Question Papers Paper 5 16

Answer 17.
E α l
Plus Two Physics Model Question Papers Paper 5 17
Plus Two Physics Model Question Papers Paper 5 17.1

Answer 18.
a. The magnetic flux linked with the rectangular loop PQRS is φ = BA = Blx
According to Faraday’s law, induced emf is
Plus Two Physics Model Question Papers Paper 5 18

Answer 19.
a. V= VC
XL = XC
Plus Two Physics Model Question Papers Paper 5 19

Answer 20.
Plus Two Physics Model Question Papers Paper 5 20
Plus Two Physics Model Question Papers Paper 5 20.1

Answer 21.
Plus Two Physics Model Question Papers Paper 5 21

Answer 22.
BE = Eb = ∆mc²
= (Zmp + (A-Z) Mn – M) c²
= (20 x 1.00783 + 20 x 1.00867 – 39.962589) x
Plus Two Physics Model Question Papers Paper 5 22

Answer 23.
Plus Two Physics Model Question Papers Paper 5 23

Answer 24.
a. I = ne A Vd……….(1)
Plus Two Physics Model Question Papers Paper 5 24

Answer 25.
Plus Two Physics Model Question Papers Paper 5 25

Answer 26.
a. Zener diode
Any change in input voltage results in change in voltage drop across Rs. without any change in voltage across zener diode.
b. When Vi increases, main current increases, Iz increases with no change in load current IL. So that V0 = Vz = IL RL. When Vi decreases, Iz decreases with no change in IL.
c. Reverse biasing

Answer 27.
Plus Two Physics Model Question Papers Paper 5 27

Answer 28.
a. Cylinder
b. S1 and S2 are two narrow slits separated by a distance “d”. They are illuminated by a monochromatic source of wavelength λ . A screen is placed at a distance of D from the slits as shown in the figure
Plus Two Physics Model Question Papers Paper 1 29
Light waves from S1 and S2 reach the point P. The path difference between these two waves is S2N.
Plus Two Physics Model Question Papers Paper 1 29.1
Plus Two Physics Model Question Papers Paper 1 29.2

Answer 29.
a. For a hydrogen atom, gives the expression to determine the energies of different energy states. But then this equation requires the radius r of the electron orbit. To calculate r, Bohr’s second postulate about the angular momentum of the electron-the quantization condition is used. The angular momentum L is given by
L = mvr
Bohr’s second postulate of quantization says that the allowed values of angular momentum are integral multiples of h/2π
Plus Two Physics Model Question Papers Paper 5 29
where n is an integer, rn is the radius of nth possible orbit and un is the speed of moving electron in the nth orbit. The
allowed orbits are numbered 1,2,3………
according to the values of n, which is called the principal quantum number of the orbit.
From the relation between un and rn is
Plus Two Physics Model Question Papers Paper 5 29.1
Plus Two Physics Model Question Papers Paper 5 29.2

Plus Two Physics Previous Year Question Papers and Answers

Plus Two Physics Model Question Paper 4

Kerala Plus Two Physics Model Question Paper 4

Time: 2 Hours
Cool off time : 15 Minutes

General Instructions to candidates:

  • There is a ‘cool off time’ of 15 minutes in addition to the writing time of 2 hrs.
  • Your are not allowed to write your answers nor to discuss anything with others during the ‘cool off time’.
  • Use the ‘cool off time’ to get familiar with the questions and to plan your answers.
  • Read questions carefully before you answering.
  • All questions are compulsory and only internal choice is allowed.
  • When you select a question, all the sub-questions must be answered from the same question itself.
  • Calculations, figures and graphs should be shown in the answer sheet itself.
  • Malayalam version of the questions is also provided.
  • Give equations wherever necessary.
  • Electronic devices except non programmable calculators are not allowed in the Examination Hall.

Plus Two Physics Previous Year Question Papers and Answers 2018

Questions 1 and 2 carry 1 score each. Answer both.
Plus Two Physics Model Question Papers Paper 4 1.

Question 1.
Energy in a current carrying coil is stored in the form of
Plus Two Physics Model Question Papers Paper 4 1
a. Electric field
b. magnetic field
c. dielectric strength
d. heat

Question 2.
Who indirectly determined the mass of the electron by measuring the charge of the electron ?
Plus Two Physics Model Question Papers Paper 4 2

Answer any six questions from question number 3 to 10. Each question carries 2 scores.
Plus Two Physics Model Question Papers Paper 4 3.

Question 3.
Match the following
Plus Two Physics Model Question Papers Paper 4 3

Question 4.
A student made a circuit as shown in the figure. S1 and S2 are switches and B is a bulb.
Plus Two Physics Model Question Papers Paper 4 4
a. He argues that this circuit is equivalent to an AND gate. Do you agree with him? Justify your answer.
Plus Two Physics Model Question Papers Paper 4 4.1
b. Write down its truth table.
Plus Two Physics Model Question Papers Paper 4 4.2

Question 5.
Using the data given below, state as to which of the given lenses will you prefer to use as
i. an eye-piece and
ii. an objective to construct an astronomical telescope? Give reason for your answer.
Plus Two Physics Model Question Papers Paper 4 5

Question 6.
Electromagnetic spectrum is an orderly arrangement of electromagnetic radiations in the j ascending or descending order of frequency/wavelength
a. Arrange the following electromagnetic radiations in the ascending order of frequency:
Visible rays, Infrared rays, X-rays, micro-waves.
b. Give one application of infrared and X-rays each.
Plus Two Physics Model Question Papers Paper 4 6

Question 7.
A magnetic needle made of iron is suspended in a uniform external magnetic field. It experience a torque and the needle starts oscillating
Plus Two Physics Model Question Papers Paper 4 7
a. Write down the frequency of oscillation of the magnetic needle.
Plus Two Physics Model Question Papers Paper 4 7.1
b. If this magnetic needle is heated beyond curie temperature while it is oscillating, then its period.
Plus Two Physics Model Question Papers Paper 4 7.2
i. increases
ii. decreases
iii. remains the same
iv. becomes infinity

Question 8.
a. An equilateral glass prism is placed on a horizontal surface. A ray PQ is incident I on it. For minimum deviation:
Plus Two Physics Model Question Papers Paper 4 8
i. PQ is horizontal
ii. QR is horizontal
iii. RS is horizontal
iv. None of these
b. A thick lens gives coloured images due to
Plus Two Physics Model Question Papers Paper 4 8.1

Question 9.
The truth table of a logic gate is given below.
Plus Two Physics Model Question Papers Paper 4 9
a. Identify the gate.
b. Draw the logic symbol of the gate.
Plus Two Physics Model Question Papers Paper 4 9.1

Question 10.
a. State Gauss’ law for magnetism.
b. How this differs from Gauss’ law for electrostatics ?
Plus Two Physics Model Question Papers Paper 4 10

Answer any five question from question number 11 to 16. Each question carries 3 scores.
Plus Two Physics Model Question Papers Paper 4 11.

Question 11.
a. Which of the following symbol represents a universal gate ?
Plus Two Physics Model Question Papers Paper 4 11
b. Shown below is an experimental set up with a semiconductor diode
Plus Two Physics Model Question Papers Paper 4 11.1
i.identify the experiment
ii.draw the resulting graph.
Plus Two Physics Model Question Papers Paper 4 11.2
c. With the help of neat circuit diagram obtain an expression for voltage gain of a transistor amplifier in C-E configuration.
Plus Two Physics Model Question Papers Paper 4 11.3

Question 12.
a. The electrical analog of mass is
(i) diode
(ii) capacitance
(iii) inductance
(iv) resistance
b. A 2 m long solenoid having diameter 6 cm and 2000 turns has a secondary of 500 turns wound closely near its mid-point. Calculate the mutual inductance between the two coils.
Plus Two Physics Model Question Papers Paper 4 12

Question 13.
What is de Broglie hypothesis ?
b. Write the formula for de Broglie wave length.
c. Calculate de Broglie wavelength associated with an electron accelerated by a potential difference of 100 volts. Given mass of the electron = 9.1×10-31kg, h= 6.634×10-34 JS,1 eV= 1.6×10-19J
Plus Two Physics Model Question Papers Paper 4 13

Question 14.
Albert Einstein, the great physist proposed a clear picture to explain photoelectric effect.
a. Explain Einstein’s photo electric equation.
b. Name the quanta of light.
Plus Two Physics Model Question Papers Paper 4 14

Question 15.
a. To construct an electronic circuit, you want to select a 470 KΩ resistor with 5% tolerance.Draw a schematic diagram indicating the colour combinations that you will select.
b. As the temperature of a metallic resistor is increased; the product of its resistivity and conductivity ………….
i. Increases
ii. Decreases
iii. Remains constant
iv. May increase or decrease
c. Draw a graph showing the relation between resistivity and temperature of a super conductor.
Plus Two Physics Model Question Papers Paper 4 15

Question 16.
Earth behaves as a magnet with magnetic poles approximately near the geographic poles.
a. The order of magnitude of earth’s magnetic field in tesla is ….
b. What do you understand by ‘dynamo effect’?
c. Classify the following materials into diamagnetic and paramagnetic.
i. Lead
ii. Magnesium
iii. Tungsten
iv. Copper
Plus Two Physics Model Question Papers Paper 4 16
Plus Two Physics Model Question Papers Paper 4 16.1

Answer any four question from question number 17 to 21. Each question carries 4 scores.
Plus Two Physics Model Question Papers Paper 4 17.

Question 17.
When the magnetic flux associated with a coil changes an emf is induced in the circuit.
a. State Faraday’s law of electromagnetic induction.
b. Mention the physical significance of Lenz’s law with an example.
c. When an electrical appliance is switched off, sparking occurs. Why?
Plus Two Physics Model Question Papers Paper 4 17

Question 18.
a. Obtain an expression for the number of radioactive nuclei present at any instant in terms of the decay constant and initial number of nuclei.
b. The half life of radioactive radon is 3.8 days. Find the time during which 1/20 of radon sample will remain undecayed.
Plus Two Physics Model Question Papers Paper 4 18

Question 19.
Electric field lines are a pictorial representation of the electric field around charges.
a. State Gauss’s Law in Electrostatics.
b. Using this law derive an expression for the electric field intensity due to a uniformly charged thin spherical shell at a point.
i. Outside the shell
ii. Inside the shell
c. Suppose that you are in a cave deep within the earth. Are you safe from thunder and lightning ? Why ?
Plus Two Physics Model Question Papers Paper 4 19

Question 20.
a. State Kirchhoff’s rules for the analysis of electrical circuits.
Plus Two Physics Model Question Papers Paper 4 20
b. The circuit diagram of a potentiometer for the determination of internal resistance of a cell is shown below. Calculate the value of the internal resistance (r) of the cell.
Given R = 100 Ω, balancing length when key (K) open = 60 cm. Balancing length when key (K) closed = 58 cm.
Plus Two Physics Model Question Papers Paper 4 20.1

Question 21.
We are familiar with the semiconductors Silicon and Germanium,
a. With a necessary schematic diagram, briefly explain the characteristics of an intrinsic semiconductor.
b. Draw the energy bands of a n-type semi-conductor.
c. The following figure represents a……..
Plus Two Physics Model Question Papers Paper 4 21

Answer any three quest on from question number 22 to 25. Each question carries 5 scores.
Plus Two Physics Model Question Papers Paper 4 22.

Question 22.
Capacitor is an arrangement to increase the charge carrying capacity of a conductor
a. Each plate of a parallel plate capacitor has a charge q on it. The capacitor is now connected to a battery. Pick out the correct statement/statements.
i. The facing surfaces of the plates have equal and opposite charges.
ii. The battery supplies equal and opposite charges to the two plates.
iii. The two plates of the capacitor have equal and opposite charges.
b. The plates of a parallel plate capacitor each of area A is charged with charges +Q and -Q. Deduce the force acting between the plates of the capacitor.
c. Van de Graaff generator is a high voltage generator used to accelerate charged particles.
Draw a labelled schematic diagram of a Van De Graaff generator and state the principle behind its working.
Plus Two Physics Model Question Papers Paper 4 22
Plus Two Physics Model Question Papers Paper 4 22.1

Question 23.
Force acting on a charged particle when it moves in a combined electric and magnetic field is known as Lorentz force.
a. A charged particle is released from rest in a region of steady and uniform electric and magnetic fields; which are parallel to each other. What will be the nature of the path followed by the charged particle? Explain your answer.
b. A rectangular loop carrying a steady current is placed in a uniform magnetic field. Obtain the expression for the torque acting on the loop.
Plus Two Physics Model Question Papers Paper 4 23

Question 24.
In the potentiometer circuit shown, the balance point is at X.
Plus Two Physics Model Question Papers Paper 4 24
State with reason, where the balance point will be shifted, when
a. Resistance R is increased, keeping all parameters unchanged.
b. Resistance S is increased, keeping R constant.
c. Cell P is replaced by another cell whose e.m.f. is lower than that of cell Q.
d. You are asked to compare the emf of two cells using a potentiometer. Draw the circuit diagram for this and explain how you will determine the ratio of e.m.f.
Plus Two Physics Model Question Papers Paper 4 24.1

Question 25.
In the figure PQ is a ray incident on a prism ABC.
Plus Two Physics Model Question Papers Paper 4 25
a. Complete the ray diagram showing the passage of light. Mark angle of incidence i, angle of emergence e, angle of deviation δ and angle of refractions r1 and r2.
b. Using the diagram obtain the relation
δ = i + e – A.
c. The critical angle for diamond is 30°. What is its refractive index?
Plus Two Physics Model Question Papers Paper 4 25.1
Plus Two Physics Model Question Papers Paper 4 25.2

Answers

Answer 1.
b

Answer 2.
Thomson

Answer 3.
Plus Two Physics Model Question Papers Paper 4 3

Answer 4.
a. No. It is an OR gate
b.
Plus Two Physics Model Question Papers Paper 4 4

Answer 5.
i. For an eye piece we prefer convex lens of max power and min.aperture is lens L4.
ii. Objective must have minimum power and maximum aperture, ie., lens L1

Answer 6.
a. Microwaves, infrared, visible, x-rays
b. Infrared – Mobile phone (Bluetooth), TV, Video recorders.
X-ray – Detect the fracture of bones.

Answer 7.
Plus Two Physics Model Question Papers Paper 4 7
b. increases

Answer 8.
a. QR is horizontal
b. disperson.

Answer 9.
a. NOR gate.
Plus Two Physics Model Question Papers Paper 4 9

Answer 10.
a. Gauss’s law for magnetism states that the net magnetic flux through any closed surface is zero, i.e.,
Plus Two Physics Model Question Papers Paper 4 10
b. Gauss’s law in electrostatics states that the total flux through a closed surface is 1/ε0 times the net charge enclosed by the closed surface.
Plus Two Physics Model Question Papers Paper 4 10.1

Answer 11.
a. (ii)
Plus Two Physics Model Question Papers Paper 4 11
b. (i) For studying the V-I characteristics of a p-n junction diode.
Plus Two Physics Model Question Papers Paper 4 11.1
Plus Two Physics Model Question Papers Paper 4 11.2

Answer 12.
a. (iii) inductance
Plus Two Physics Model Question Papers Paper 4 12

Answer 13.
a. He proposed that material particles in motion should display wave-like properties.
Plus Two Physics Model Question Papers Paper 4 13

Answer 14.
a. Einstein explained photoelectric effect based on quantum theory. According to quantum theory, light contain photons having energy hυ, when a photon of energy hυ0 incident on a metal surface, electrons are liberated. A small portion of the photon energy is used for work function (φ) and remaining energy is appeared as K.E of the electrons.
By law of conservation of energy, we can write hυ = hυ0 + ½ mv²; hυ energy of incident wave ; hυ0 -» work function ½mv² -» K.E of electron
b. Photon.

Answer 15.
Plus Two Physics Model Question Papers Paper 4 15

Answer 16.
a. 10-5 T or 0.38 x 10-4 T
b. Earth magnetic field arises due to electrical currents produced by convective motion of metallic fluids in the outer core of the earth. This is called dynamo effect.
c. Lead, Copper – Dia magnetic
Magnesium, Tungsten – Para magnetic

Answer 17.
a. Faraday’s first law: According to first law an EMF is induced in a circuit.when the magnetic flux linked with it changes.
b. Lenz’s law verifies conservation of energy in the electromagnetic induction phenomenon.
When N-pole of a bar magnet is moved towards a coil, the direction of induced current in the coil will be such that the end 2 will behave as N-pole (to oppose the incoming N-pole). For this the current in the coil will be anticlockwise as seen from the right face of the coil (using right hand thumb rule).
c. Due to back emf.

Answer 18.
Plus Two Physics Model Question Papers Paper 4 18
t = 3.8 x 4.33 = 16.45 days

Answer 19.
a. Gauss’s Law states that the total flux through a closed surface is 1/ε0 times the net charge enclosed by the closed surface.
b. (i).Outside the sphere (r > R)
Plus Two Physics Model Question Papers Paper 4 19
c. Yes. This is because the electric field inside the cave will be zero. This is called electrostatic shielding.

Answer 20.
a. First law: The total current entering the junction is equal to the total current leaving the junction.
Second law: Total emf in a closed circuit is equal to the sum of the voltage drops.
Plus Two Physics Model Question Papers Paper 4 20

Answer 21.
Plus Two Physics Model Question Papers Paper 4 21
A pure semiconductor which is free from any impurity is called intrinsic semiconductor. Its conductivity controlled by the number of valence electron present in valence band. At 0K no electron is available in the conduction band semiconductor act as insulator.
Plus Two Physics Model Question Papers Paper 4 21.1
c. Diode.

Answer 22.
a. i. True ii. True iii. True
Plus Two Physics Model Question Papers Paper 4 22
c. The high electric field at the pointed ends of comb C1, ionises the air near them. The +ve charges in air are repelled and got deposited on the belt through a corona discharge.
Plus Two Physics Model Question Papers Paper 4 22.1
The charges are carried upto C2. A similar corona discharge takes place at C2 and the charges are finally transferred to the shell M. The charges spread over uniformly on the outer surface of M raising its potential to few million volts.

Answer 23.
a. The charged particle execute an accelerated straight line motion along the direction of electric field. Since the electric field and magnetic field are parallel the magnetic field will be in the direction of the motion of the charged particle and hence the force due to magnetic field will be zero.,
b. A rectangular loop carrying a steady current I and placed in a uniform magnetic field experiences a torque. It does not experience a net force.
Consider a rectangular coil ABCD, placed in a uniform magnetic field, B. Then,
τ = I ab B = NIAB sin θ or τ =M x B ,
where A is the area of the coil, N is the no of turns. Here M = I (NA) is called magnetic dipole moment. If θ = 0 or 180, τ = 0.

Answer 24.
a. Balance point will be shifted towards B.
b. No change.
c. Balance point not found on AB.
Plus Two Physics Model Question Papers Paper 4 24
Potential difference between two points of a current carrying conductor is directly proportional to the length of the wire between the two points.
A battery (B1), Rheostat and key are connected in between A and B. This circuit is called primary circuit. Positive end of E1 and E2, are connected to A and other ends are connected to a two way key. Jockey is connected to a two key through galvanometer. This circuit is called secondary circuit.
Key in primary circuit is closed and then E1 is put into the circuit and balancing length I1 is found out.
Plus Two Physics Model Question Papers Paper 4 24.1

Answer 25.
Plus Two Physics Model Question Papers Paper 4 25

Plus Two Physics Previous Year Question Papers and Answers

Plus Two Physics Model Question Paper 3

Kerala Plus Two Physics Model Question Paper 3

Time: 2 Hours
Cool off time : 15 Minutes

General Instructions to candidates:

  • There is a ‘cool off time’ of 15 minutes in addition to the writing time of 2 hrs.
  • Your are not allowed to write your answers nor to discuss anything with others during the ‘cool off time’.
  • Use the ‘cool off time’ to get familiar with the questions and to plan your answers.
  • Read questions carefully before you answering.
  • All questions are compulsory and only internal choice is allowed.
  • When you select a question, all the sub-questions must be answered from the same question itself.
  • Calculations, figures and graphs should be shown in the answer sheet itself.
  • Malayalam version of the questions is also provided.
  • Give equations wherever necessary.
  • Electronic devices except non programmable calculators are not allowed in the Examination Hall.

Plus Two Physics Previous Year Question Papers and Answers 2018

Questions 1 – 7 carry 1 score each. Answer any six questions.
Plus Two Physics Model Question Papers Paper 3 1.

Question 1.
Potentiometer measures potential more accurately because
Plus Two Physics Model Question Papers Paper 3 1
a. It measures potential in the open circuit
b. It uses sensitive galvanometer for null deflection
c. It uses high resistance potentiometer wire
d. It measures potential in the closed circuit

Question 2.
Tesla is the unit of
Plus Two Physics Model Question Papers Paper 3 2

Question 3.
Write the time constant of C-R circuit ?
Plus Two Physics Model Question Papers Paper 3 3

Question 4.
Green-house effect is the heating up of earth atmosphere due to____
a. green plants
b.infra-red rays
c. x-rays
d. ultraviolet rays
Plus Two Physics Model Question Papers Paper 3 4

Question 5.
Rainbow is formed due to a combination of
Plus Two Physics Model Question Papers Paper 3 5
a. dispersion and total internal reflection
b. refraction and absorption
c. dispersion and focusing
d. refraction and scattering

Question 6.
Ratio of intensities of two waves are given by 4:1. Then find the ratio of the amplitudes of the two waves.
Plus Two Physics Model Question Papers Paper 3 6
Plus Two Physics Model Question Papers Paper 3 6.1

Question 7.
The spectrum obtained from a sodium vapor lamp is an example of
Plus Two Physics Model Question Papers Paper 3 7
a. band spectrum
b. continuous spectrum
c. emission spectrum
d. absorption spectrum

Questions 8 to 15 carry 2 score each. Answer any 7 questions.
Plus Two Physics Model Question Papers Paper 3 8.

Question 8.
Copper is one of the suitable materials to make connecting wires due to its low resistivity.
a. What do you mean by resistivity?
b. A copper wire is in the form of a cylinder and has a resistance R. It is stretched till its thickness reduces by half of its initial size. Find its new resistance in terms of R.
Plus Two Physics Model Question Papers Paper 3 8

Question 9.
It was james Clark Maxwell who modified Ampere’s circuital theorem by introducing the concept of displacement current.
Plus Two Physics Model Question Papers Paper 3 9
a. What do you mean by displacement current?
Plus Two Physics Model Question Papers Paper 3 9.1
Plus Two Physics Model Question Papers Paper 3 9.2

Question 10.
Match the following
Plus Two Physics Model Question Papers Paper 3 10

Question 11.
A body of mass m is charged negatively. State whether the following statements are true or false.
a. During charging, there is change in mass of the body.
b. The body can be charged to 2.5 e where e is the charge of an electron.
c. While charging the body by induction new charges are created in it.
d. The force between two charged object is less when there is a medium between them (than in vacuum).
Plus Two Physics Model Question Papers Paper 3 11
Plus Two Physics Model Question Papers Paper 3 11.1

Question 12.
a. A receiver in a communication system must have
i. pick-up antenna
ii. demodulator
iii. amplifier
iv. all of these
b. Which of the following statements is wrong?
i. The attenuation of surface waves in creases with increase in frequency.
ii. The phenomenon involved in sky wave propagation is similar to total internal reflection.
iii. Space wave mode of propagation is used in satellite communication.
iv. Sky wave propagation is useful only in the range of frequencies 30 MHz to 40 MHz.
Plus Two Physics Model Question Papers Paper 3 12

Question 13.
a. The core of a transformer has the following properties :
i. core is laminated.
ii. hysterisis loop is narrow.
Explain the significance of each property.
b. What is meant by resonance in an LCR circuit ?
Plus Two Physics Model Question Papers Paper 3 13

Question 14.
a. Permanent magnets should have
i. high retentivity and low coercivity.
ii. low retentivity and high coercivity.
iii. high retentivity and high coercivity.
iv. low retentivity and low coercivity.
b. Distinguish between Para, Ferro an Dia magnetism.
Plus Two Physics Model Question Papers Paper 3 14

Question 15.
Plus Two Physics Model Question Papers Paper 3 15
a. From the above figures which one corresponds to a typical semiconductor.
b. Dimension of temperature coefficient α is……..
Plus Two Physics Model Question Papers Paper 3 15

Questions 16 to 22 carry 3 score each. Answer any 6 questions.
Plus Two Physics Model Question Papers Paper 3 15.

Question 16.
a. Work function of a metal is the
i. energy required by an electron to get absorbed in the metal surface.
ii. minimum energy required by an electron to escape from the metal surface.
iii. energy required by an electron to be retained in the metal surface.
iv. maximum energy required by an electron to escape from the metal surface.
b. Write Einstein’s Photo-Electric Equation and explain the terms in it.
c. All Photo electrons are not emitted with the same energy as the incident photons. Why?
Plus Two Physics Model Question Papers Paper 3 16

Question 17.
Kirchhoff’s rules are very useful for analysis of electrical circuits.
a. State Kirchhoff s junction rule.
b. Find the effective resistance of the circuit given below :
Plus Two Physics Model Question Papers Paper 3 17

Question 18.
A fascinating behaviour of the series RLC circuit is the phenomenon of resonance.
a. Explain Resonance in an LCR circuit.
b. Draw a graphical representation of variation of current amplitude in im with frequency to ω
c. What do you mean by sharpness of resonance? Explain it.
Plus Two Physics Model Question Papers Paper 3 18

Question 19.
a. When a low flying aircraft passes over head, we sometimes notice a slight shaking of the picture on our TV screen. Identify the optical phenomenon behind it.
b. In electromagnetic spectra, the wave length and frequencies are inversely related. A radio can tune into any station in the 7.5 MHz to 12 MHz band. Determine the corresponding wavelength band
Plus Two Physics Model Question Papers Paper 3 19

Question 20.
Various propagation modes are used in communication.
a. Mention two communication system that uses space wave propagation.
b. Why modulation is necessary in communication ?
c. If a signal of frequency cos is used to ωs modulate a carrier wave of frequency ωc
which are the frequencies contained in the modulated signal other than ωc?
Plus Two Physics Model Question Papers Paper 3 20

Question 21.
Match the following quantities using the analogy between mechanical and electrical quantities.
Plus Two Physics Model Question Papers Paper 3 21

Question 22.
In a nuclear reactor the chain reaction is carried out under controlled conditions.
a. Name the material that is used as control rods in a nuclear reactor.
b. Average energy of a neutron produced in fission of 92235U nucleus is…….
c. Write down the reactions involved in the conversion of to 92238U to 94239Pu in a nuclear reactor
Plus Two Physics Model Question Papers Paper 3 22

Questions from 23 – 26 carry 4 scores. Answer any 3 questions.
Plus Two Physics Model Question Papers Paper 3 22.

Question 23.
To emit a free electron from a metal surface a minimum amount of energy must be supplied.
a. This energy is called….
b. State three methods to supply this energy to the free electron.
c. When light of frequency 7.21×1014 Hz is incident on a metal surface, the maximum speed of the ejected electrons is 6×105 m/s. Calculate the threshold frequency for the metal.
Plus Two Physics Model Question Papers Paper 3 23

Question 24.
Seema constructed a series LCR circuit in the laboratory as shown in the diagram. She found that the voltages across the inductor and capacitor are equal when the circuit is connected to an ac source.
Plus Two Physics Model Question Papers Paper 3 24
Plus Two Physics Model Question Papers Paper 3 24.1
a. State the condition at which the voltages across L and C become equal.
Plus Two Physics Model Question Papers Paper 3 24.2
b. Obtain an expression for the frequency at which this situation occurs in a series LCR circuit.
Plus Two Physics Model Question Papers Paper 3 24.3
c. Find the voltmeter and ammeter readings in the circuit.
Plus Two Physics Model Question Papers Paper 3 24.4

Question 25.
Brewster’s law gives a relationship between angle of polarisation and refractive index of the material.
Plus Two Physics Model Question Papers Paper 3 25
a. State Brsewster’s law
Plus Two Physics Model Question Papers Paper 3 25.1
b. A ray of light is allowed to incident on the surface of a glass plate and it is found that the reflected and refracted rays are mutually perpendicular as shown in the figure. The refracted ray is deviated from its initial path by an angle of 30°
Determine the refractive index of the glass plate.
Plus Two Physics Model Question Papers Paper 3 25.1
Plus Two Physics Model Question Papers Paper 3 25.2
Plus Two Physics Model Question Papers Paper 3 25.3

Question 26.
a. A ray of light travels from a denser to a rarer medium then, the ray
i. doesn’t bend at all.
ii. bends towards the normal.
iii. bends away from the normal.
iv. goes along the normal.
b. Draw a diagram showing the path of a monochromatic light through a triangular prism.
c. Using this diagram obtain the relation
Plus Two Physics Model Question Papers Paper 3 26

Questions from 27 – 29 carry 5 scores. Answer any 2 questions.
Plus Two Physics Model Question Papers Paper 3 27.

Question 27.
a. What do you mean by barrier potential of a diode ?
b. With the help of a diagram explain the working of a full wave rectifier.
Plus Two Physics Model Question Papers Paper 3 27
Plus Two Physics Model Question Papers Paper 3 27.1

Question 28.
Various phenomena exhibited by light can be explained using the wave theory of light.
a. Name the phenomenon which proves the transverse nature of light.
b. What are the differences between interference and diffraction?
c. A plane wave front is incident on a single slit. Discuss the diffraction pattern formed by the slit. Represent the variation of intensity graphically.
Plus Two Physics Model Question Papers Paper 3 28

Question 29.
A moving charge can produce a magnetic field.
a. How does a current loop behaves like a magnetic dipole ?
b. Draw the magnetic field lines for a current loop to support your answer.
c. i. What is a cyclotron ?
ii. Write down the expression for cyclotron frequency.
Plus Two Physics Model Question Papers Paper 3 29

Answers

Answer 1.
a

Answer 2.
magnetic field

Answer 3.
CR

Answer 4.
b

Answer 5.
a

Answer 6.
Plus Two Physics Model Question Papers Paper 3 6

Answer 7.
c

Answer 8.
a. Resistivity of the material of a conductor is defined as the resistance of the conductor of unit length and of unit area of cross-section.
Plus Two Physics Model Question Papers Paper 3 8

Answer 9.
a. Displacement current is that current which comes into existence, in addition to the conduction current whenever the electric flux changes with time.
b. Ampere

Answer 10.
Plus Two Physics Model Question Papers Paper 3 10

Answer 11.
a. true
b. false
c. false
d. true

Answer 12.
a. (iv) all of these
b. (iv) Sky wave propagation is useful only in the range of frequencies 30 to 40MHz

Answer 13.
a. (i) For reducing eddy current. Eddy current heats the core and energy will be lost.
(ii) For reducing hysterisis loss.
b. In LCR, at a particular frequency XL = XC. This condition is called resonance. This frequency is called resonance frequency.

Answer 14.
a. iii. high retentivity and high coercivity.
b. Paramagnetism: Feebly magnetise along applied field.
Diamagnetism: Feebly magnetise opposite to magnetic field.
Ferromag netism: Strongly magnetise along magnetic field.

Answer 15.
a. Fig.(2)
b. α = coefficient of temp
Plus Two Physics Model Question Papers Paper 3 15

Answer 16.
a. ii. minimum energy required by an electron to escape from the metal surface.
Plus Two Physics Model Question Papers Paper 3 16
c. All the photoelectrons emitted from the metal do not have the same energy. Photoelectric current is zero when the stopping potential is sufficient to repel | even the most energetic photoelectrons, with the maximum kinetic energy (Kmax),So that Kmax = eV0

Answer 17.
a. In an electric circuit, the sum of currents | entering the junction is equal to the sum of currents leaving the junction.
b. Here 3Ω, 6Ω are parallel and 6Ω and 12Ω are parallel.
Plus Two Physics Model Question Papers Paper 3 17
Plus Two Physics Model Question Papers Paper 3 17.1

Answer 18.
a. In LCR, at a particular frequency XL = XC. This frequency is called resonance frequency.
Plus Two Physics Model Question Papers Paper 3 18
c. Sharpness of resonance is depends of quality factor or Q factor.
Q factor is equal to the ratio inductive reactance or capacitative reactance to the impedence at resonance.
Plus Two Physics Model Question Papers Paper 3 18.1

Answer 19.
a. Interference
Plus Two Physics Model Question Papers Paper 3 19

Answer 20.
a. TV Broadcast, Satellite, FM, RADAR,
b. 1. Size of the antenna: We need an antenna having a size comparable to the wave length of the signal.
2. Effective power radiated by antenna,
Plus Two Physics Model Question Papers Paper 3 20
Plus Two Physics Model Question Papers Paper 3 20.1

Answer 21.
Plus Two Physics Model Question Papers Paper 3 21

Answer 22.
Plus Two Physics Model Question Papers Paper 3 22

Answer 23.
a. Work function.
b. Thermionic emission, field emission and photo electric emission.
Plus Two Physics Model Question Papers Paper 3 23

Answer 24.
a. p.d across L and C are equal when inductive reactance = Capacitive reactance (XL = XC), i.e., at resonance.
b. At resonance, XL = XC
Plus Two Physics Model Question Papers Paper 3 24

Answer 25.
a. Tangent of angle of polarization is equal to the reflective index.
b. i + r = 90°
θ + r = 90°
θ = 90° – r = 90 – 30 = 60°
n = tan 60° = √3 = 1.732

Answer 26.
a. iii bends away from the normal
b.
Plus Two Physics Model Question Papers Paper 3 26

Answer 27.
a. During the formation of a p-n junction, holes diffuse from p to n side and electrons diffuse from n to p side. Thus there is excess of positively charged ions in n side and negatively charged ions in p side. This sets up a potential difference across the junction which is called barrier potential. It stops further diffusion.
Plus Two Physics Model Question Papers Paper 3 27
• During +ve half cycle of a.c, the end A is positive and end B is negative with respect to the centre tap. Diode D1 gets forward biased and conducts current while D2 is reverse biased and does not conduct.
• During -ve half cycle, the end Abe comes negative and end B becomes positive with respect to the centre tap. The diode D gets reverse biased and does not conduct. The diode D2 gets forward biased and conducts current.
• The output current is unidirectional and pulsating which may be filtered by connecting a capacitor parallel to load RL.

Answer 28.
a. polarization
b.
Plus Two Physics Model Question Papers Paper 3 28

Answer 29.
a. One face north and other face south.
i.e.,
Plus Two Physics Model Question Papers Paper 3 29
Plus Two Physics Model Question Papers Paper 3 29.1
Cyclotron uses the fact that the frequency of revolution of the charged particle in a magnetic field is independent of its energy. The particles move most of the time inside two semicircular disc-like metal containers, D1 and D2, which are called dees.
The whole assembly is evacuated to minimise collisions between the ions and the air molecules. A high frequency alternating voltage is applied to the dees. Positive ions or positively charged particles (e.g., proton) are released at the centre P. They move a semi-circular path in one of the dees and arrive in the gap between the dees in a time interval T/2; where T, the period of revolution, is given T = 1/ uc = 2πm/qB
uc = qB / 2πm. This frequency is called the cyclotron frequency for obvious reasons and is denoted by uc.

Plus Two Physics Previous Year Question Papers and Answers

Plus Two Physics Model Question Paper 2

Kerala Plus Two Physics Model Question Paper 2

Time: 2 Hours
Cool off time : 15 Minutes

General Instructions to candidates:

  • There is a ‘cool off time’ of 15 minutes in addition to the writing time of 2 hrs.
  • Your are not allowed to write your answers nor to discuss anything with others during the ‘cool off time’.
  • Use the ‘cool off time’ to get familiar with the questions and to plan your answers.
  • Read questions carefully before you answering.
  • All questions are compulsory and only internal choice is allowed.
  • When you select a question, all the sub-questions must be answered from the same question itself.
  • Calculations, figures and graphs should be shown in the answer sheet itself.
  • Malayalam version of the questions is also provided.
  • Give equations wherever necessary.
  • Electronic devices except non programmable calculators are not allowed in the Examination Hall.

Plus Two Physics Previous Year Question Papers and Answers 2018

Questions 1 and 2 carry 1 score each. Answer both.
Plus Two Physics Model Question Papers Paper 2 1.

Question 1.
Electromagnets are made of soft iron because soft iron has
Plus Two Physics Model Question Papers Paper 2 1
a. low retentivity and high coercive force
b. high retentivity and high coercive force
c. low retentivity and low coercive force
d. high retentivity and low coercive force

Question 2.
Draw the symbol of NOR gate
Plus Two Physics Model Question Papers Paper 2 2

Answer any six questions from question number 3 to 10. Each question carries 2 scores.
Plus Two Physics Model Question Papers Paper 2 3.

Question 3.
a. The electric field of a plane electromagnetic wave traveling in the +ve Z-direction is described by
a. Ex = E0 sin (kz + ωt)
b. E= E0 sin (kz – ωt)
c. Ex = E0 sin (2kz)
d. Ex = E0 sin (kz)
b. We feel excessive sweating on a cloudy day. Why ?
Plus Two Physics Model Question Papers Paper 2 3
Plus Two Physics Model Question Papers Paper 2 3.1

Question 4.
For many purposes, it is necessary to change an alternating voltage from one value to another. This is done with a transformer.
a. The basic principle behind a transformer is…..
b. Give an expression for the voltage and current in a transformer.
Plus Two Physics Model Question Papers Paper 2 4

Question 5.
a. Name the physical quantity which has its unit joule coulomb-1. Is it a vector or a scalar?
b. Two plane sheets of charge densities +σ and -σ are kept in air as shown in figure. What are the electric field intensities at point A and B?
Plus Two Physics Model Question Papers Paper 2 5

Question 6.
Name the electromagnetic waves used for the following applications.
a. Imaging of bones in human body.
b. Mobile phone communication.
c. Remote control of T.V.sets.
d. For sterilizing surgical instruments.
Plus Two Physics Model Question Papers Paper 2 6

Question 7.
a. Range of an electronic communication system is the
i. distance to the nearest TV station.
ii. distance to the nearest radio station.
iii. largest distance the signal can travel.
iv. largest distance between a source and destination up to which the signal is received with sufficient strength.
b. If the height of TV transmitting antenna is increased its coverage increases. Why?
Plus Two Physics Model Question Papers Paper 2 7

Question 8.
Match the following in 3 columns.
Plus Two Physics Model Question Papers Paper 2 8
Plus Two Physics Model Question Papers Paper 2 8.1

Question 9.
a. The resistance ‘R’ of a conductor depends on its length ‘I’, area of cross- section ‘A’ and the resistivity of the material ‘ρ’. The correct expression connecting R, I, A and ‘ρ’ is
Plus Two Physics Model Question Papers Paper 2 9
b. The voltage – current graphs for two resistors of the same material and same radii with lengths L1 and L2 are shown in the figure. If L1 > L2 state with reason, which of these graphs represents voltage current charge for L1?
Plus Two Physics Model Question Papers Paper 2 9.1

Question 10.
De Broglie proposed the wave nature of electrons suggesting matter waves.
Find the momentum, speed and de Broglie wavelength of an electron with kinetic energy of 120 eV.
Plus Two Physics Model Question Papers Paper 2 10

Answer any five questions from question number 11 to 16. Each question carries 3 scores.
Plus Two Physics Model Question Papers Paper 2 11.

Question 11.
a. The work function of a metal is 6 eV. If two photons each having energy 4 eV strike with the metal surface
i. will the emission be possible ?
ii. why ?
b. The waves associated with matter is called matter waves. Let λe and λp be the de- Broglie wave lengths associated with electron and proton respectively. If they are accelerated by same potential, then
Plus Two Physics Model Question Papers Paper 2 11

Question 12.
The region around a charge where its effect can be felt is called the electric field.
a. The electric field lines corresponding to an electric field is shown below.
Plus Two Physics Model Question Papers Paper 2 12
b. Some equipotential surfaces are shown in the figure. What can you say about the magnitude and direction of the electric field ?
Plus Two Physics Model Question Papers Paper 2 12.1

Question 13.
Given below is the graph between frequency (υ) of the incident light and maximum kinetic energy (Ek) of emitted photoelectrons.
Plus Two Physics Model Question Papers Paper 2 13
a. Define the terms – work functions and threshold frequency.
b. Find the values of
i. threshold frequency and
ii.work function from the graph
Plus Two Physics Model Question Papers Paper 2 13.1
Plus Two Physics Model Question Papers Paper 2 13.2

Question 14.
Consider the logic circuit using NOR gates given below:
Plus Two Physics Model Question Papers Paper 2 14
a. Write the truth table for the circuit.
b. Identify the logic operation which this circuit is performing.
Plus Two Physics Model Question Papers Paper 2 14.1

Question 15.
The phenomenon in which electric current is generated by varying magnetic field is called electromagnetic induction.
a. Which law helps to detect the direction of induced current? State the law.
b. Explain what are eddy currents.
c. Write any two uses of eddy currents.
Plus Two Physics Model Question Papers Paper 2 15

Question 16.
Biasing is provided for maintaining proper current flow across a p-n junction.
Plus Two Physics Model Question Papers Paper 2 16
a. In a biased p-n junction the net flow of holes is from ‘n’ region to ‘p’ regional
Plus Two Physics Model Question Papers Paper 2 16.1
Plus Two Physics Model Question Papers Paper 2 16.2
b. For the device shown below draw the V-I characteristics when the potential is applied between the terminals A and B.
Plus Two Physics Model Question Papers Paper 2 16.3

Answer any four questions from question number 17 to 21. Each question carries 4 scores.
Plus Two Physics Model Question Papers Paper 2 17.

Question 17.
a. Find the equivalent resistance of the circuit.
Plus Two Physics Model Question Papers Paper 2 17
b. Determine the current through the 75Ω resistance.
Plus Two Physics Model Question Papers Paper 2 17.1

Question 18.
a. A convex lens
i. is thicker at the edges than at the middle.
ii. is thicker at the middle than at the edges.
iii. diverges rays of light.
iv. is of uniform thickness everywhere.
b. With the help of a ray diagram sketch the image formation of a convex lens when the object is between C and F.
c. Derive the lens maker’s formula.
Plus Two Physics Model Question Papers Paper 2 18
Plus Two Physics Model Question Papers Paper 2 18.1

Question 19.
Remya makes the following circuit to measure the emf of a cell.
Plus Two Physics Model Question Papers Paper 2 19
She says that the voltmeter reading will give the emf of the cell.
Plus Two Physics Model Question Papers Paper 2 19.1
a. The Physics teacher says that it is not :possible to measure the emf of the cell in this way. Justify this statement.
b. Explain with a circuit diagram the method to measure the emf of a cell with a potentiometer if you are given another standard cell, (whose e.m.f. is known)
Plus Two Physics Model Question Papers Paper 2 19.2

Question 20.
a. What do you mean by Q value of a nuclear reaction?
b. Write down the expression for Q value in the case of α decay.
c. Two nuclei have mass numbers in the ratio 1 : 64. What is the ratio of their nuclear radii ?
Plus Two Physics Model Question Papers Paper 2 20
Plus Two Physics Model Question Papers Paper 2 20.1

Question 21.
A capacitor is a device used for storing charge as well as energy. It is based on a property called capacitance.
a. Write the SI unit of capacitance and define it.
b. You are given two capacitors of 2µF and 3µF. What are the maximum and minimum values of capacitance that can be obtained by combining them?
c. Obtain an expression for the electrostatic energy stored in a capacitor.
Plus Two Physics Model Question Papers Paper 2 21

Answer any three questions from question number 22 to 25. Each question carries 5 scores.
Plus Two Physics Model Question Papers Paper 2 22.

Question 22.
To emit a free electron from a metal surface a minimum amount of energy must be supplied.
a. This energy is called …….
b. State three methods to supply this energy to the free electron.
c. When light of frequency 7.21×1014 Hz is incident on a metal surface, the maximum speed of the ejected electrons is 6×105 m/ s. Calculate the threshold frequency for the metal.
Plus Two Physics Model Question Papers Paper 2 22
Plus Two Physics Model Question Papers Paper 2 22.1

Question 23.
Figure shows a graph between angle of incidence and the respective deviation of light when it is passed through an optical element.
Plus Two Physics Model Question Papers Paper 2 23
a. Name this optical element.
b. What will happen to the angle of minimum deviation D if it is completely immersed in water? Justify your answer.
c. Obtain an expression for the refractive index of the material of this optical element.
Plus Two Physics Model Question Papers Paper 2 23.1

Question 24.
The idea of ‘Electric field lines ’ is useful in pictorially mapping the electric field around charges.
a. Give any two properties of electric lines of force.
b. State Gauss’s theorem in electrostatics.
c. Using the theorem, derive an expression for electric field due to a uniformly charged spherical shell.
i. at a point outside the shell
ii. at a point inside the shell
d.
Plus Two Physics Model Question Papers Paper 2 24
A point charge of +10 μc is at a distance of 5cm directly above the centre of a square of side 10 cm as shown in the figure. What is the electric flux through the square?
Plus Two Physics Model Question Papers Paper 2 24.1

Question 25.
Gauss’s law can be used to determine the electric field due to a charge distribution, a. Below are some statements about Gauss’s law. Say whether they are true or false.
i. Gauss’s law is valid only for symmetrical charge distributions.
ii. The electric field calculated by Gauss’s law is the field due to charges inside the Gaussian surface.
b. Apply Gauss’s law to find the electric field due to an infinitely long plane sheet of charge.
c. “There can be no net charge in a region in which the electric field is uniform at all points”. Do you agree with this statement? Justify your answer.
Plus Two Physics Model Question Papers Paper 2 25
Plus Two Physics Model Question Papers Paper 2 25.1

Answers

Answer 1.
d

Answer 2.
Plus Two Physics Model Question Papers Paper 2 2

Answer 3.
a. b. E= Esin (kz – ωt)
b. The clouds prevents IR radiation from passing through and thus keeps the earth’s surface warm.

Answer 4.
a. Mutual induction
Plus Two Physics Model Question Papers Paper 2 4

Answer 5.
a. Electric potential. It is a scalar quantity.
Plus Two Physics Model Question Papers Paper 2 5

Answer 6.
a. x-rays
b. Radio waves
c. Infrared rays
d. Ultraviolet rays

Answer 7.
a. iv. largest distance between a source and destination up to which the signal is received with sufficient strength.
b. We know range, d = √2hR ; Where h is the height of the antenna. Therefore increasing height increases the range ie., coverage.

Answer 8.
Plus Two Physics Model Question Papers Paper 2 8

Answer 9.
a. \(R=\frac { pl }{ A }\)
b. B represents voltage change for L. Resistance of B is greater than that for A, since resistance increases with length.

Answer 10.
De Broglie wavelength of electrons
Plus Two Physics Model Question Papers Paper 2 10
from V², we can calculate V so we get the value of P also.

Answer 11.
a. (i) No
(ii) Free electrons are bound to the metal with some force. The minimum energy j required to liberate an electron from the metal is called work function, here the energy of one photon is less than work function. So emission will not be possible.
b. (i) λe > λp

Answer 12.
a. EA = EC > EB
b. The direction of electric field at any point on the equatorial line of the dipole will be opposite to the dipole moment. The magnitudes of electric fields are equal

Answer 13.
a. Work functions: It is the minimum amount of energy required by an electron to just escape from the metal surface of the metal.
Threshold frequency: The minimum value of the frequency of incident radiation which can emit photo electrons from a material.
b. i.Threshold frequency,v0 = 10 x 1014 Hz
ii. Work function = hv0
= 6.63 x 10-34 x 10 x 1014
= 6.63 x 10-19 J

Answer 14.
a. Truth Table
Plus Two Physics Model Question Papers Paper 2 14

Answer 15.
a. Lenz’s law. This law states that the direction of the induced e.m.f or current is such as to oppose the cause that produced it
b. The currents induced in the body of a conductor, when the magnetic flux linked with the conductor changes are called eddy currents or Focault’s currents
c. i. Electric Brakes
ii. Speedometer

Answer 16.
a. Reverse
Plus Two Physics Model Question Papers Paper 2 16

Answer 17.
Plus Two Physics Model Question Papers Paper 2 17

Answer 18.
a. ii. is thicker at the middle than at the edges.
Plus Two Physics Model Question Papers Paper 2 18
Plus Two Physics Model Question Papers Paper 2 18.1

Answer 19.
a. The voltmeter reads terminal potential difference. Emf cannot be measured in this way since the cell is connected to a resistance. Emf is the potential different across the terminals of a cell in a open circuit.
Plus Two Physics Model Question Papers Paper 2 19

Answer 20.
a. The disintegration energy or the Q- value of a nuclear reaction is the difference between the initial mass energy and the total mass energy of the decay products.
b. For α -decay
Plus Two Physics Model Question Papers Paper 2 20

Answer 21.
a. SI unit of capacitance is farad. Capacity of a conductor is said to be one farad if one coulomb of charge on it raises its potential by one volt.
b. If the capacitors are in series,
Plus Two Physics Model Question Papers Paper 2 21
Plus Two Physics Model Question Papers Paper 2 21.1

Answer 22.
a. Work function.
b. Thermionic emission, field emission and photo electric emission.
c. u = 7.21 x 1014 Hz, V = 6 x 105 m/s
Plus Two Physics Model Question Papers Paper 2 22

Answer 23.
a. Prism
b. The angle of minimum deviation decreases.
Refractive index of glass with respect to water is 1.5. But refractive index of glass with respect to water is 1.33.
Plus Two Physics Model Question Papers Paper 2 23
c.
Plus Two Physics Model Question Papers Paper 2 23.1

Answer 24.
a. i. Electric lines of force will never intersect.
ii.The tangent to the line of force at any point gives the direction of electric field at that point.
b. The electric flux through any closed surface in free is equal to 1/ε0 times the total charge enclosed by the surface.
Plus Two Physics Model Question Papers Paper 2 24
Plus Two Physics Model Question Papers Paper 2 24.1

Answer 25.
a. i. False
ii. False
Plus Two Physics Model Question Papers Paper 2 25
c. Yes, there can be no net charge in a region in which the electric field is uniform at all points. Consider a region with uniform electric field and a cubical gaussian surface in it whose face is perpendicular to electric field. Now from one face electric field enters which is considered to be +ve from the possible face it gets out which is considered to be -ve. So net flux through the gaussian surface is zero.

Plus Two Physics Previous Year Question Papers and Answers

Plus Two Physics Model Question Paper 1

Kerala State Board New Syllabus Plus Two Physics Previous Year Question Papers and Answers.

Kerala Plus Two Physics Model Question Paper 1 with Answers

Board SCERT
Class Plus Two
Subject Physics
Category Plus Two Previous Year Question Papers

Time: 2 Hours
Cool off time: 15 Minutes
Maximum: 60 Score

General Instructions to candidates:

  • There is a ‘cool off-time’ of 15 minutes in addition to the writing time of 2 hrs.
  • You are not allowed to write your answers nor to discuss anything with others during the ‘cool off time’.
  • Use the ‘cool off time’ to get familiar with the questions and to plan your answers.
  • Read questions carefully before you answering.
  • All questions are compulsory and the only internal choice is allowed.
  • When you select a question, all the sub-questions must be answered from the same question itself.
  • Calculations, figures, and graphs should be shown in the answer sheet itself.
  • Malayalam version of the questions is also provided.
  • Give equations wherever necessary.
  • Electronic devices except non-programmable calculators are not allowed in the Examination Hall.

Questions 1 – 7 Carry 1 Score each. Answer any six questions. (6 × 1 = 6)

Question 1.
Write the expression for intensity of electric field near to the surface of a charged conductor with uniform charge density σ.
Answer:
E = \(\frac{\sigma}{\varepsilon_{0}}\)

Question 2.
The unit of electrical resistivity is ……….
Answer:
Ωm (ohm meter)

Question 3.
E.m.f can be induced in a coil placed in an external magnetic field by:
a) Changing the intensity of the magnetic field
b) Changing the area of the coil
c) Changing the orientation of the coil
d) All of the above
Answer:
d) All of the above

Question 4.
Write the energy and momentum associated with a moving photon.
Answer:
E = hν, P = mc

Question 5.
The value of relative permeability of a diamagnetic material is;
a) μr > 1
b) μr = 1
c) μr < 1
d) μr = 1
Answer:
c) μr < 1

Question 6.
The electric field amplitude of an electromagnetic wave is 15V/m. Find the magnetic field amplitude of the wave.
Answer:
B = \(\frac{E}{C}=\frac{15}{3 \times 10^{8}}\) = 5 × 10-8T

Question 7.
A convex lens is placed in a medium having a refractive index greater than that of the lens. The lens now behaves as;
a) Converging lens
b) Diverging lens
c) Plane glass plate
d) None of the above
Answer:
b) Diverging lens

Questions 8 to 15 carry 2 scores each. Answer any 7 questions. (7 × 2 = 14)

Question 8.
a) ‘Electrostatic field is always normal to the surface of a charged conductor’. Justify the statement.
b) The value of elelctric potential at the surface of a charged conductor is 10V. Find the value of the intensity of the electric field and potential at a point interior to it.
Answer:
a) The surface of the charged conductor is an equipotential surface. In equipotential surface, the potential is the same and hence the electric field is always normal to the surface of a charged conductor.
b) E = 0

Question 9.
A galvanometer with a coil of resistance 12Ω shows a full-scale deflection for a current of 2.5 mA. How can it be converted into an ammeter of range 7.5 A?
Answer:
G= 12Ω, Ig= 2.5 mA = 2.5 × 10-3A, I = 7.5 A
S = \(\frac{I_{g} \times G}{I-I_{g}}=\frac{2.5 \times 10^{-3} \times 12}{7.5-2.5 \times 10^{-3}}=4 \times 10^{-3} \Omega\)

Question 10.
The curves shown in figure are drawn for different magnetic materials. Among the three curves, name the curve that,
Plus Two Physics Model Question Papers Paper 1, 1
a) Represent the material usually used for making permanent magnets.
b) Represent the material usually used for making electromagnets.
Answer:
a) A
b) C

Question 11.
A capacitor C, a variable resistance R, and a bulb B are connected in series to a.c. mains in the circuit as shown. The bulb glows with some brightness. How the glow of the bulb change if.
Plus Two Physics Model Question Papers Paper 1, 2
a) A dielectric slab is introduced between the plates of the capacitor.
b) The resistance R is increased keeping the same capacitance.
Answer:
a) The brightness of bulb increases.
(When dielectric slab is introduced, capacitance increases hence capacitive reactance decreases, then bulb glows with more brightness.)
b) The brightness of bulb decreases.

Question 12.
James Clerk Maxwell modified Ampere’s circuital theorem by introducing the concept of displacement current.
a) What do you mean by displacement current?
b) Write down the equation for displacement current.
Answer:
a) The current due to time varying electric field is called displacement current.
b) id = \(\varepsilon_{0} \frac{\mathrm{d} \varphi}{\mathrm{dt}}\)

Question 13.
An object AB is kept in front of a concave mirror as shown in figure.
Plus Two Physics Model Question Papers Paper 1, 3
a) Complete the ray diagram showing the image formation of object.
b) How will the position and intensity of image be affected if the lower half of the mirror’s reflecting surface is painted black?
Answer:
a)
Plus Two Physics Model Question Papers Paper 1, 4
b) The position of image does not charge. But the intensity decreases.

Question 14.
Assuming that the two diodes D1 and D2 used in the electric circuit as shown in figure are ideal. Find out the value of current flowing through 2.5Ω resistor.
Plus Two Physics Model Question Papers Paper 1, 5
Answer:
Since D2 is reverse biased, no current flows through 3Ω. We need to consider only the other two resistance 3Ω and 2.5Ω, which are connected in series. Total resistance, R = 3Ω + 2.5Ω = 5.5Ω
v = 10V
I = \(\frac{V}{R}=\frac{10}{5.5}\) = 1.81 A

Question 15.
a) Mention the function of the following used in the communication system
(i) Transducer
(ii) Transmitter
b) Figure shows the block diagram of an AM transmitter. Identify boxes X and Y.
Plus Two Physics Model Question Papers Paper 1, 6
Answer:
a) i) Transducer: The device that converts one form of energy into another is called transducer.
ii) Transmitter – Atransmittertransmits the information

b) x → Modulator
y → Power Amplifier

Questions 16 to 22 carry 3 scores each. Answer any 6 questions. (6 × 3 = 18)

Question 16.
Three capacitors of capacitances 2µF, 3µF and 4µF are connected in series.
a) Find the equivalent capacitance of the combination
b) The plates of a parallel plate capacitor have an area 20 cm2 each are separated by a distance of 2.5 mm. The capacitor is charged by connecting it to a 400V supply. How much electrostatic energy is stored in the capacitor?
Answer:
a) C1 = 2μF = 2 × 10-6F
C2 = 3μF = 3 × 10-6F
Plus Two Physics Model Question Papers Paper 1, 7
b) A = 20cm2 = 20 × 10-4m2
d = 2.5 mm = 2.5 × 10-3m
ε0 = 8.85 × 10-12C2/Nm2
v = 400v
Plus Two Physics Model Question Papers Paper 1, 8

Question 17.
A circuit using a potentiometer and a battery of negligible internal resistance is set up as shown to develop a constant potential gradient along the wire PQ. Two cells of e.m.fs E1 and E2 are connected in series as in figure in combination 1 and 2. The balance points are obtained respectively at 400 cm and 240 cm from point P. Find
Plus Two Physics Model Question Papers Paper 1, 9
a) The ratio between E1 and E2
b) Balancing length for the cell with emf E1 only.
Answer:
a) From the connection (1), we get
E1 + E2 = k × 400 ………(1)
k = constant
From the connection (2), we get
E2 – E1 = 240 K ……….(2)
Adding (1) and (2), we get
2E2 = 640 K
E2 = 320 K
Subtituting for E2 in equation (1)
E1 + 320 K = 400 K
E1 = 80 K
The ratio \(\frac{E_{1}}{E_{2}}=\frac{1}{4}\)
b) l1 be the balancing length of E1
E1 = kl1
80k ∝ kl1
l1 = 80 cm

Question 18.
A conducting rod PQ of length ‘l’ connected to a resistance ‘R’ is moved at a uniform speed ‘V’, normal to a uniform magnetic field ‘B’.
Plus Two Physics Model Question Papers Paper 1, 10
a) Deduce the expression for e.m.f induced in the conductor.
b) Find the magnitude and direction of current through the conductor.
Answer:
a) flux, Φ = BA
Here, Area A = ldx
dx is small displacement rod in time Δ T
Φ = B × l × dx
Plus Two Physics Model Question Papers Paper 1, 11

Question 19.
In tuner circuits, we use the phenomenon of resonance.
a) Write the condition of resonance in a series LCR circuit.
b) A series LCR circuit uses L = 0.1 H, C = 10μF and R = 100Ω. Find the value of frequency at which the amplitude of current is maximum.
Answer:
a) Lω = \(\frac{1}{\mathrm{C} \omega}\)
b) L = 0.1 H, C = 10μF = 10 × 10-6F
Plus Two Physics Model Question Papers Paper 1, 12

Question 20.
The focal length of a lens has dependent on its radii of curvatures and refractive index. Derive lens maker’s formula.

Question 21.
a) Write the expression for the de Broglie wavelength associated with a charged particle having charge ‘q’ and mass ‘m’, when it is accelerated by a potential of ‘V’ volts.
b) A proton and an electron have the same kinetic energy. Which one has a greater value of de Broglie wavelength and why?
Answer:
Plus Two Physics Model Question Papers Paper 1, 13
For electron, λe = \(\frac{h}{\sqrt{2 m_{e} K E_{e}}}\)
But KEp = KEe and mp > Me, hence λe > λp
Electron has greater de Broglie wavelength.

Question 22.
Find the binding energy per nucleon of \({ }_{20}^{40} \mathrm{Ca}\) nucleus.
Given m\({ }_{20}^{40} \mathrm{Ca}\) = 39.962589u.
mp = 1.00783u, mn = 1.00867u.
Take 1 amu = 931 MeV/c2
Answer:
Mass detect, Δm = Zmp + (A – Z) mn – M
Δm = 20 mp + 20 mn – 39.962589
= 20 × 1.00783 + 20 × 1.00867 – 39.962589
= 0.367411 u
Binding energy = 0.367411 × 931 = 342.06 MeV

Questions from 23 – 26 carry 4 scores. Answer any 3 questions. (3 × 4 = 12)

Question 23.
Gauss’s theorem is useful for finding the intensity of elelctric field.
a) Write Gauss’s law in its mathematical form
b) Using the law, prove that the intensity of the electric field at a point due to a uniformly charged infinite plane sheet is independent of the distance from it.
Answer:
\(\phi=\oint \overrightarrow{\mathrm{E}} \cdot \mathrm{d} \overrightarrow{\mathrm{s}}=\frac{\mathrm{q}}{\varepsilon_{0}}\)

Question 24.
a) Express Ohm’s law in terms of current density, electrical resistivity, and intensity of the electric field.
b) Explain the variation of resistance of a semiconductor with temperature. Also, draw the graph showing the variation of resistivity of silicon with temperature.
Answer:
a) J = σE
b) As temperature increases, the number density of electrons (η) of semiconductor increases.
ρ = \(\frac{m}{h e^{2} \tau}\)
Hence resistivity decreases with increase in temperature.
Plus Two Physics Model Question Papers Paper 1, 14

Question 25.
The relation between the magnetic field and current is given by Biot-Savart’s law.
a) Write the expression for the magnetic field at point along the axis of a circuit loop of radius ‘R’ carrying a current ‘I’.
b) From the above expression. Find the value of magnetic field at the centre of the loop.
c) Sketch the magnetic field lines for current carrying circular loop
Answer:
a) B = \(\frac{\mu_{0} \mathrm{NIR}^{2}}{2\left(\mathrm{x}^{2}+\mathrm{R}^{2}\right)^{3 / 2}}\)
I → Current
N → Number of turns
X → distance tot he point from the center of loop
b) B = \(\frac{\mu_{0} N I}{2 a}\)
c)
Plus Two Physics Model Question Papers Paper 1, 28

Question 26.
Plus Two Physics Model Question Papers Paper 1, 15
a) Identify the diode in the circuit and write the use of the resistance Rs.
b) Explain how the diode helps to stabilize the output voltage of the circuit.
c) Name the type of biasing used in this diode
Answer:
a) Zener diode, Voltage regulator
b) Solar Cell: Solar cell is a junction diode used to convert solar energy into electrical energy.

Circuit details:
Plus Two Physics Model Question Papers Paper 1, 16
Its p-region is thin and transparent and is called emitter. The n-region is thick and is called base. The output is taken across RL.

Working: When light falls on this layer, electrons from the n-region cross to the p-region and holes in the p-region cross into the n-region. Thus a voltage is developed across RL.
Solar cells are used to charge storage batteries during the daytime.
c) Reverse bias

Questions 27 to 29 carry 5 scores. Answer any 2 questions. (2 x 5 = 10)

Question 27.
In the figure, PQ is the incident ray on the equilateral glass prism ABC.
Plus Two Physics Model Question Papers Paper 1, 17
a) Complete the ray diagram showing the passage of light and mark the angle of deviation.
b) Derive an expression for the refractive index of the material of the prism.
Answer:
Plus Two Physics Model Question Papers Paper 1, 18
b)
Plus Two Physics Model Question Papers Paper 1, 21
ABC is a section of a prism. AB and AC are the refracting faces, BC is the base of the prism. ∠A is the angle of the prism.

A ray PQ incidents on the face AB at an angle i1. QR is the refracted ray inside the prism, which makes two angles r1 and r2 (inside the prism). RS is the emergent ray at angle ‘i2

The angle between the emergent ray and incident ray is the deviation ‘d’.
In the quadrilateral AQMR,
∠Q + ∠R = 180°
[since N1M and NM are normall
i.e. ∠A + ∠M = 18O° ………(1)
In the ΔQMR,
∴ r1 + r2 + ∠M = 180°……..(2)
Comparing eq (1) and eq (2)
r1 + r2 = ∠A ……….(3)
From the ΔQRT,
(i1 – r1) + (i2 – r2) = d
[since exterior angle equal sum of the opposite interior angles]
(i1 + i2) – (r1 + r2) = d
but, r1 + r2 = A
∴ (i1 + i2) – A = d
(i1 + i2) = d + A ……..(4)
It is found that for a particular angle of incidence, the deviation is found to be minimum value ‘D.
At the minimum deviation position,
i1 = i2 =i, r1 = r2 = r and d=D
Hence eq (3) can be written as,
Plus Two Physics Model Question Papers Paper 1, 22
Plus Two Physics Model Question Papers Paper 1, 23
It is found that when the angle of incidence increases deviation (d) decreases and reaches a minimum value and then increases. This minimum value of the angle of deviation is called the angle of minimum deviation.

Question 28.
According to Christian Huygens’s wave theory, light emanating from a source as wavefronts.
a) What is the shape of wavefront emerging from a linear source?
b) Derive the mathematical expression for the bandwidth of interference bands obtained in young’s double-slit experiment with the help of a suitable diagram.
Answer:
a) Cylinder
b)
Plus Two Physics Model Question Papers Paper 1, 24
S1 and S2 are two coherent sources having wavelength A. Let ‘d’ be the distance between two coherent sources. A screen is placed at a distance D from sources. ‘O’ is a point on the screen equidistant from S1 and S2.
Hence the path difference, S1O – S2O = 0
So at ‘O’ maximum brightness is obtained.
Let ‘P’ be the position of nm bright band at a distance xn from O. Draw S1A and S2B as shown in the figure.
From the right angle ΔS1AP
Plus Two Physics Model Question Papers Paper 1, 25
Plus Two Physics Model Question Papers Paper 1, 26
Plus Two Physics Model Question Papers Paper 1, 27
This is the width of the bright band. It is the same for the dark band also.

Question 29.
a) The radius of nth stationary orbit of hydrogen atom is:
rn = \(\frac{\mathrm{n}^{2} \mathrm{~h}^{2} \varepsilon_{0}}{\pi \mathrm{me}^{2}}\)
Using Bohr postulates, obtain an expression for the energy of electron in the stationary states of H-atom.
b) Draw the energy level diagram showing how the spectral lines corresponding to Balmerseries occur due to the transition between the energy levels.
Answer:
a) Consider a hydrogen atom which has one electron revolving around the nucleus having one proton. The centripetal force for an electron is provided by the electrostatic force between electron and proton
Plus Two Physics Model Question Papers Paper 1, 19
b) Energy level diagram of the hydrogen atom
Plus Two Physics Model Question Papers Paper 1, 20
Note: An electron can have any total energy above E = 0 EV. In such situations electron is free. Thus there is a continuum of energy states above E = 0 EV.

Plus Two Physics Previous Year Question Paper March 2017

Kerala State Board New Syllabus Plus Two Physics Previous Year Question Papers and Answers.

Kerala Plus Two Physics Previous Year Question Paper March 2017 with Answers

Board SCERT
Class Plus Two
Subject Physics
Category Plus Two Previous Year Question Papers

Time: 2½ Hours
Cool off time : 15 Minutes

General Instructions to Candidates

  • There is a ‘cool off time’ of 15 minutes in addition to the writing time of 2½ hrs.
  • You are not allowed to write your answers nor to discuss anything with others during the ‘cool off time’.
  • Use the ‘cool off time’ to get familiar with the questions and to plan your answers.
  • Read questions carefully before you answering.
  • All questions are compulsory and the only internal choice is allowed.
  • When you select a question, all the sub-questions must be answered from the same question itself.
  • Calculations, figures, and graphs should be shown in the answer sheet itself.
  • Malayalam version of the questions is also provided.
  • Give equations wherever necessary.
  • Electronic devices except non-programmable calculators are not allowed in the Examination Hall.

Question 1.
A concave lens always produces ………….. images
i. real
ii. virtual
iii. magnified
iv. None of these

Question 2.
A Zener diode is always operated in …….. bias.

Question 3.
Momentum of a photon with wavelength λ is……
Plus Two Physics Previous Year Question Paper March 2017 Q3

Question 4.
Write down the truth table of a NOR gate.

Question 5.
a. How many electrons constitute an electric charge of -16 µC?
(i) 1013
(ii) 1014
(iii) 1015
(iv) 1012
b. An electric dipole is a pair of equal and opposite point charges +q and -q separated by a distance r. Write an expression for its dipole moment.
c. When an electric dipole is subjected to a uniform electric field, what will happen?

Question 6.
A message signal of frequency 10kHz and peak voltage 10V is used to modulate a carrier of frequency 1 MHz and peak voltage 20V. Find the modulation index.

Question 7.
a. Resistivity of a conductor depends upon
(i) its material
(ii) its cross-sectional area
(iii) its length
(iv) All of the above
b. Calculate the current flowing through the following circuit :
Plus Two Physics Previous Year Question Paper March 2017 Q7
c. A potentiometer is a device to measure emf of a cell. Explain how the emfs of two cells can be compared using a potentiometer.

Question 8.
a. Choose the wrong option.
(i) Volt = Weber / second
(ii) Weber = Henry x Ampere
(iii) Joule = Henry x Ampere²
(iv) Volt = Weber x Second
b. The current in a coil of self-inductance 0.1 H varies from 2A to 5A in a time of 1ms. Find the induced emf across the coil.

Question 9.
a. Sound waves do not exhibit
i. interference
ii. diffraction
iii. polarization
iv. reflection
b. Describe Young’s double-slit experiment to determine the bandwidth of the interference pattern.
OR
a. The intensity of scattered light I in Rayleigh scattering is proportional to……
b. Explain the diffraction pattern obtained due to a single slit and represent graphically the variation of intensity with angle of diffraction.

Question 10.
a. Define half-life period of a radioactive nucleus. Write down the relation connecting half-life period and mean life.
b. Define 1 amu. Calculate its energy equivalent in MeV.

Question 11.
Photoelectric current does not depend on energy of the radiation, but on its intensity. Explain?

Question 12.
a. Speed of light in glass is 2 x 108 m/s. The Refractive index of glass is……
b. For an equilateral prism made of a material of refractive index √2 . find the angle of minimum deviation for a ray of monochromatic light.
c. Draw the ray diagram of a simple microscope that uses a single convex lens. Derive an expression for its linear magnification.

Question 13.
a. A dielectric slab is placed between the plates of a parallel plate capacitor. Its capacitance
i. becomes zero
ii. remains the same
iii. decreases
iv. increases
b. Derive an expression for energy stored in a capacitor.

Question 14.
a. At resonance, in an LCR circuit, the emf and current are
(i) in phase
(ii) out of phase
(iii) having a phase difference of \(\frac { \pi }{ 2 } \)
(iv) having a phase difference of \(\frac { \pi }{ 6 } \)
b. In the following circuit, find the value of V
Plus Two Physics Previous Year Question Paper March 2017 Q14
OR
a. In a circuit carrying an ideal coil with negligible resistance, the power dissipated is …….
b. In the following circuit, find the impedance.
Plus Two Physics Previous Year Question Paper March 2017 Q14.1

Question 15.
Explain hysteresis and draw hysteresis curve for a ferromagnetic substance.

Question 16.
Choose the appropriate values for X-rays from the given table.
Plus Two Physics Previous Year Question Paper March 2017 Q16

Question 17.
a. Unit of wavenumber is
i. Hz
ii. eV
iii. m
iv. m-1
b.Energy of ground state of a hydrogen atom is -13.6 eV. What is its ionization potential?

Question 18.
The current amplification factor for CB configuration of a transistor is 0.9. Find out the current amplification ion factor for CE configuration.

Question 19.
a. An electric charge q is moving with a velocity v in the direction of a magnetic field B. The magnetic force acting on the charge is
Plus Two Physics Previous Year Question Paper March 2017 Q19
b. starting from Biot – Savart law, obtain an expression for the magnetic field at an axial point of a circular coil carrying current.
OR
a. An ammeter is a current measuring device which is always connected in an electric circuit.
b. Describe a cyclotron and obtain an expression for cyclotron frequency.

Plus Two Physics Previous Year Question Paper March 2018

Kerala State Board New Syllabus Plus Two Physics Previous Year Question Papers and Answers.

Kerala Plus Two Physics Previous Year Question Paper March 2018 with Answers

 

Board SCERT
Class Plus Two
Subject Physics
Category Plus Two Previous Year Question Papers

Time: 2 Hours
Cool off time: 15 Minutes
Maximum: 60 Scores

General Instructions to candidates:

  • There is a ‘cool off-time’ of 15 minutes in addition to the writing time of 2 hrs.
  • You are not allowed to write your answers nor to discuss anything with others during the ‘cool off time’.
  • Use the ‘cool off time’ to get familiar with the questions and to plan your answers.
  • Read questions carefully before you answering.
  • All questions are compulsory and the only internal choice is allowed.
  • When you select a question, all the sub-questions must be answered from the same question itself.
  • Calculations, figures, and graphs should be shown in the answer sheet itself.
  • Malayalam version of the questions is also provided.
  • Give equations wherever necessary.
  • Electronic devices except non-programmable calculators are not allowed in the Examination Hall.

The given value of constants can be used wherever necessary.

Velocity or light in vacuum, c = 3 × 108 m/s
Plank’s constant, h = 6.64 × 10-34 Js
Charge of electron, e = 1.6 × 10-19C
Mass of the electron, m = 9.1 × 10-31 kg
Avogadro number = 6.025 × 1023 atom/mol
ε0 = 8.854  × 10-12 CN-1 m-2
μ0 = 4π × 10-7 Tm/A

Questions 1 and 2 carry 1 Score each. Answer both. (2 × 1 = 2)

Question 1.
How will you represent a resistance of 3700 Ω ± 5% using colour code?
Answer:
Orange, Violet, Red and Gold.

Question 2.
Draw the symbol of NAND gate.
Answer:
Plus Two Physics Previous Year Question Paper March 2018, 1

Answer any six questions from question number 3 to 10. Each question carries 2 Scores. (Scores: 6 × 2 = 12)

Question 3.
Two equal and opposite charges placed in air as shown in figure:
Plus Two Physics Previous Year Question Paper March 2018, 2
a) Redraw the figure and show the direction of dipole moment (P), direction of resultant electric field (E) at P.
b) Write an equation to find out the electric field at P.
Answer:
a)
Plus Two Physics Previous Year Question Paper March 2018, 3
b) \(\overline{\mathrm{E}}=\frac{1}{4 \pi \varepsilon_{0}} \frac{\overline{\mathrm{P}}}{\mathrm{r}^{3}}\)

Question 4.
Calculate the electrical capacitance of earth. The radius of earth is 6400 km.
Answer:
Capacitance of earth C = \(4 \pi \varepsilon_{0} \mathrm{R}\)
C = \(4 \pi \varepsilon_{0}\) × 6400 × 103
= 710.8 µF

Question 5.
What do you mean by drift velocity? Write the relation between drift velocity and electric current.
Answer:
The average velocity acquired by an electron under the applied electric field is called drift velocity.
I = neAVd

Question 6.
A galvanometer is connected as shown in the figure:
Plus Two Physics Previous Year Question Paper March 2018, 4
a) This combination can be used as ………..
(voltmeter/rheostat/ammeter)
b) Derive an expession to find the value of resistance S.
Answer:
a) Ammeter
b)
Plus Two Physics Previous Year Question Paper March 2018, 5
Let G be the resistance of the galvanometer, giving full deflection for a current Ig.
To convert it into an ammeter, a suitable shunt resistance ‘S’ is connected in parallel. In this arrangement Ig current flows through Galvanometer and remaining (I – Ig) current flows through shunt resistance.
Since G and S are parallel
P.d Across G = p.d across S
Ig × G = (I – Ig)s
Plus Two Physics Previous Year Question Paper March 2018, 35

Question 7.
An unpolarised light incident on a medium as shown in figure.
Plus Two Physics Previous Year Question Paper March 2018, 6
Show that the direction of reflected and refracted waves are mutually perpendicular.
Answer:
According to Brewster’s law
n = tan p = \(\frac{\sin p}{\cos p}\) ………(1)
According to Snell’s law
n = \(\frac{\sin p}{\sin x}\) …….(2)
Comparing (1) and (2), we get
\(\frac{\sin p}{\sin x}=\frac{\sin p}{\sin x}\)
sin x = cos p = sin(90 – p)
x = 90 – p, but p = r
x = 90 – r
x + r =90°
If x + r is 90°, angle between reflected and refracted ray will be 90°.

Question 8.
How many electrons, protons and neutrons are there in 16g of 8O16?
Answer:
Number of atoms in 16g of 8O16 = 6.023 × 1023
Number of protons = 8 × 6.023 × 1023
Numberof neutrons = 8 × 6.023 × 1023

Question 9.
The given block diagram shows general form of a communication system.
Plus Two Physics Previous Year Question Paper March 2018, 34
a) Identify the blocks X and Y.
b) What is the difference between attenuation and amplification?
Answer:
a) X – transmitter
Y – Receiver
b) Attenuation: The loss of strength of a signal while propagating through a medium is known as attenuation.
Amplification: It is the process of increasing the amplitude of a signal.

Question 10.
What is the population covered by the transmission, if the average population density around the tower is 1200 km-2? The antenna height is 160 m.
Answer:
Distance travelled by the wave d = \(\sqrt{2 R h}\)
Area = πd2
= π(\(\sqrt{2 R h}\))2
= π2Rh
= 3.14 × 2 × 6400 × 0.160 Km2
Population covered = Area × population density
= 3.14 × 2 × 6400 × 0.16 × 1200
= 7.72 × 106

Answer any five questions from question number 11 to 16. Each question carries 3 Scores. (Scores: 5 × 3 = 15)

Question 11.
Two spheres encloses charges as shown in figure:
Plus Two Physics Previous Year Question Paper March 2018, 7
a) Derive and expression for electric field intensity at any point on the surface S2.
b) What is the ratio of electric flux through S1 and S2?
Answer:
a) According to Guess law
Plus Two Physics Previous Year Question Paper March 2018, 8
b) Flux a charge
for first sphere Φ1 α Q
for second sphere Φ2 α 3Q
∴ \(\frac{\phi_{1}}{\phi_{2}}=\frac{Q}{3 Q}=\frac{1}{3}\)
= 1 : 3

Question 12.
The electric field lines of a positive charge is as shown in figure:
Plus Two Physics Previous Year Question Paper March 2018, 9
a) Give the sign of potential difference Vp – Vq.
b) Give the sign of work done by the field in moving a small positive charge from ‘Q’ to ‘P’.
c) What is the shape of equipotential surface near the charge?
Answer:
a) positive.
b) negative.
c) spherical.

Question 13.
a) Name the part of electromagnetic spectrum:
i) used in radar systems.
ii) produced by bombarding a metal target by a high speed electrons.
b) Electromagnetic waves are produced by ………..
(charges at rest/charges in uniform motion/ charges in accelerated motion)
c) Why only microwaves are used in microwave ovens?
Answer:
a) i) Micro waves
ii) X – rays
b) Charges in accelerated motion.
c) The frequency of micro waves matches with the rotational frequency of water molecule.

Question 14.
Two thin convex lenses of focal length f1 and f2 are placed in contact:
a) If the object is at principal axis, draw ray diagram of the image formation by this combination of lenses.
b) Obtain a general expression for effective focal length of the combination in terms of f1 and f2.
Answer:
a)
Plus Two Physics Previous Year Question Paper March 2018, 10

b) Consider two thin convex lenses of focal lengths f1 and f2 kept in contact. Let O be an object kept at a distance ‘u’ from the first lens L1, I1 is the image formed by the first lens at a distance v1.
Then from the lens formula, we can write,
\(\frac{1}{f_{1}}=\frac{1}{v_{1}}-\frac{1}{u}\) ……..(1)
This image will act as the virtual object for the second lens and the final image is formed at I (at a distance v). Then
\(\frac{1}{f_{2}}=\frac{1}{v}-\frac{1}{v_{1}}\) ……..(2)
Adding eq (1) and eq (2)
Plus Two Physics Previous Year Question Paper March 2018, 11
If the two lenses are replaced by a single lens of focal length ‘F’ the image is formed at ‘v’. Then we can write,
\(\frac{1}{F}=\frac{1}{v}-\frac{1}{u}\) …….(4)
from eq(3) and (4),
\(\frac{1}{F}=\frac{1}{f_{1}}+\frac{1}{f_{2}}\) …….(3)

Question 15.
A plane wave-front AB incident on a surface XY and undergoes refraction as shown in figure:
Plus Two Physics Previous Year Question Paper March 2018, 12
a) Prove Snell’s law of refraction by using this figure.
b) What is the change in speed of propagation and frequency of the refracted wave?
Answer:
a) AB is the incident wavefront and c1 is the velocity of the wavefront in the first medium. CD is the refracted wavefront and c2 is the velocity of the wavefront in the second medium. AC is a plane separating the two media.
Plus Two Physics Previous Year Question Paper March 2018, 13
The time taken for the ray to travel from P to R is
Plus Two Physics Previous Year Question Paper March 2018, 14
O is an arbitrary point. Hence AO is a variable.
But the time to travel a wavefront from AB to CD is constant. In order to satisfy this condition, the term containing AO in eq (2) should be zero.
Plus Two Physics Previous Year Question Paper March 2018, 15
where 1n2 is the refractive index of the second medium w.r.t. the first. This is the law of refraction.

b) Speed decreses
frequency remains same.

Question 16.
a) State radioactive decay law.
b) Prove the relation N(t) = N0e-λt.
Answer:
a) The number of nuclei undergoing decay per unit time is proportional to number of nuclei in the sample at that time.

b) According to Law of Radioactive decay,
\(\frac{\mathrm{d} \mathrm{N}}{\mathrm{d} \mathrm{t}}\) = -λN
\(\frac{\mathrm{d} \mathrm{N}}{\mathrm{d} \mathrm{t}}\) = -λdt
Integrating
In N = -λt + C …….. (1)
C is the constant of integration. To get value of C, let us assume that initially (t = 0) the number of nuclei be N0.
∴ C = In N0
Substituting for C in equation (1) we get,
In N – In N0 = -λt
In \(\frac{\mathrm{N}}{\mathrm{N}_{0}}\) = -λt
\(\frac{\mathrm{N}}{\mathrm{N}_{0}}\) e-λt
N = N0e-λt

Answer any four questions from question number 17 to 21. Each question carries 4 Scores. (4 × 4 = 16)

Question 17.
Two charged partjeles q1 and q2 are moving through a uniform magnetic field (B) as shown in figure:
Plus Two Physics Previous Year Question Paper March 2018, 16
a) What is the shape of path of q1 and q2.
b) Derive an expression for cyclotron frequency with the help of a neat diagram.
Answer:
a) q1 charges undergoes helical motion and q2 moves in a circular path.

b)
Plus Two Physics Previous Year Question Paper March 2018, 17
Let ‘v’ be the velocity of ion, q the charge of the ion and B the magnetic flux density. If the ion moves along a semicircular path of radius ‘r’, then we can write
Plus Two Physics Previous Year Question Paper March 2018, 18

Question 18.
The B-H curve of a ferromagnetic material is as shown in figure:
Plus Two Physics Previous Year Question Paper March 2018, 19
a) What do you mean by ferromagnetic material? Write any two properties of it.
b) Identify the value of retentivity and coercivity from the figure.
c) A solenoid has a core of a material with relative permeability 400. The windings of the solenoid are insulated from the core and carry a current of 2A. If the number of turns is 1000 per metre, calculate magnetic intensity (H) and magnetic field (B).
Answer:
a) Ferromagnetic substances are those which gets strongly magnetized in an external magnetic field.
Properties: High susceptibility and permeability.
b) Ob represents retentivity and Oc coercivity.
c) H = nI
= 1000 × 2
= 2 × 103 A/m
B = µsµrH
= µ0 × 400 × 2 × 103
B = IT

Question 19.
a) Derive an expression for self inductance of a solenoid.
b) What do you mean by eddy current? Write any two applications of it.
Answer:
a) Consider a solenoid (air core) of length/.number of turns N and area cross section A. let ‘n’ be the no. of turns per unit length (n = N/I)
The magnetic flux linked with the solenoid,
Φ = BAN
Φ = μ0nIAN (since B = μ0 ni)
but Φ = LI
∴ LI = μ0nIAN
L = μ0nAN
If solenoid contains a core of relative permeability
μr
Then L = μ0μrnAN

b) when magnetic flux in a metal block changes, induced currents are produced. This current is called eddy current.
Applications:

  • Magnetic beaking in trains
  • Damping
  • Induction furnace

Question 20.
The given graph shows the variation of KE with frequency of incident radiations for two surfaces A and B.
Plus Two Physics Previous Year Question Paper March 2018, 20
a) Which of the two metals has greater work function? For which of the metals will stopping potential be more for the same frequency of incident radiation?
b) The work function of caesium metal is 2.14 eV. When light of frequency 6 × 1014 Hz incident on the metal surface, what is the maximum KE of the photoelectrons and stopping potential?
Answer:
a) A
b) KE = hυ – Φ0
= 6.6 × 10-34 × 6 × 1014 – 2.14 × 1.6 × 10-19
= 0.551 × 10-19 J
= 0.345 eV
∴ Stopping potential = 0.345 V

Question 21.
a) Name different series of lines observed in hydrogen spectrum.
b) Draw energy level diagram of hydrogen atom.
Answer:
a) Laymen, Balmer, Paschen, Bracket, Pfund
b)
Plus Two Physics Previous Year Question Paper March 2018, 21

Answer any three questions from question number 22 to 25. Each question carries 5 Scores. (Scores: 3 × 5 = 15)

Question 22.
The circuit diagram of a potentiometer for determining the emf ‘E’ of a cell of negligible internal resistance is as shown in figure.
Plus Two Physics Previous Year Question Paper March 2018, 22
a) State the principle of working of a potentiometer.
b) How the balancing length AJ changes when the value of R1 decreases?
c) Derive an expression to find out internal resistance of a cell.
Answer:
a) Potential difference between two points of current carrying conductor is directly proportional to the length of the wire between two points.
b) Balancing length decreases.
c)
Plus Two Physics Previous Year Question Paper March 2018, 23
The key (K1) in the primary circuit is closed and the key is the secondary (K2) is open. Jockey is moved to get zero deflection in galvanometer. The balancing length l1 (from A) is found out.
Then we can write.
E1 ∝ l1 …….. (1)
Key K2 is put in the circuit, corresponding balancing length (l2) is found out. Let V be the applied voltage, then
V1 ∝ l1 ……..(2)
‘V’ is the voltage across resistance box.
Current through resistance box
i.e., voltage across resistance,
Plus Two Physics Previous Year Question Paper March 2018, 24

Question 23.
The phasor diagram of the alternating voltage across an unknown device X and current flowing through it are shown below.
Plus Two Physics Previous Year Question Paper March 2018, 25
a) Identify the device X.
b) Draw the graphical variation of current and voltage with ωt through this device.
c) Draw the phasor diagram which shows the relation among VR, VL, VC and I in a series LCR circuit.
Answer:
a) Resistor
b)
Plus Two Physics Previous Year Question Paper March 2018, 26
c)
Plus Two Physics Previous Year Question Paper March 2018, 27

Question 24.
A ray of light passing through a prism. If the refracted ray QR is parallel to the base BC, as shown in figure.
Plus Two Physics Previous Year Question Paper March 2018, 28
a) Derive an expression to find out deviation (d).
b) Draw a graph which shows the variation of ‘d’ with ‘i’.
c) Calculate the angle of minimum deviation if A = 60° and refractive index of the material of prism is 1.62.
Answer:
a)
Plus Two Physics Previous Year Question Paper March 2018, 29
The angle between the emergent ray and incident ray is the deviation ‘d’.
In the quadrilateral AQMR,
∠Q + ∠R = 180°
[since N1M and NM are normal]
i.e., ∠A + ∠M = 180° …….(1)
In the Δ QMR,
∴ r1 + r2 + ∠M = 180° …….(2)
Comparing eq (1) and eq (2)
r1 + r2 = ∠A …….(3)
From the Δ QRT,
(i1 – r1) + (i2 – r2) = d
(i1 + i2) – (r1 + r2) = d
but, r1 + r2 = A
∴ (i1 + i2) – A = d

Plus Two Physics Previous Year Question Paper March 2018, 30
Plus Two Physics Previous Year Question Paper March 2018, 31

Question 25.
The symbol of a diode is shown in the figure:
Plus Two Physics Previous Year Question Paper March 2018, 32
a) The diode is a ………… (rectifier diode/photo diode/Zener diode)
b) Draw the VI characteristics of above diode.
c) A zener diode with VZ = 6.0 V is used for voltage regulation. The current through the load is to be 4.0 mA and that through the zener diode is 20 mA. If the unregulated input is 10.0 V, what is the value of series resistor R?
d) What is the fundamental frequency of the ripple in a full wave rectifier circuit operating from 50 Hz mains?
Answer:
a) Rectifier diode
b)
Plus Two Physics Previous Year Question Paper March 2018, 33
c) I = IZ + IL
= 20 × 10-3 + 4 × 10-3
I = 24 × 10-3 A
Resistance R = \(\frac{V}{1}=\frac{10-6}{24 \times 10^{-3}}\)
R = 166.67 Ω
d) 100 Hz

Plus Two Maths Model Question Paper 5

Kerala Plus Two Maths Model Question Paper 5

Time : 2 1/2 Hours
Cool off time : 15 Minutes
Maximum : 80 Score

General Instructions to Candidates :

  • There is a ‘Cool off time’ of 15 minutes in addition to the writing time.
  • Use the ‘Cool off time’ to get familiar with questions and to plan your answers.
  • Read questions carefully before you answering.
  • Read the instructions carefully.
  • When you select a question, all the sub-questions must be answered from the same question itself.
  • Calculations, figures and graphs should be shown in the answer sheet itself.
  • Malayalam version of the questions is also provided.
  • Give equations wherever necessary.
  • Electronic devices except non programmable calculators are not allowed in the Examination Hall.

Plus Two Maths Previous Year Question Papers and Answers 2018 1

QUESTIONS

Question 1 to 7 carry 3 scores each. Answer any six questions only
Plus Two Maths Model Question Papers Paper 5 1

Question 1.
a. Let R be a relation on the set {1,2,3} given by R= {(1,1), (2,2), (1,2), (2,1), (2,3)}. Which among the following element to be inclu¬ded to R so that R becomes Symmetric?
i. (3,3)
ii. (3,2)
iii. (1,3)
iv. (3,1)

b. If * is defined by a * b=a-b2 and ⊕ is de-fined by a ⊕ is b=a2+b, where a and b are integers. Then find the value of (3⊕4)*5.
Plus Two Maths Model Question Papers Paper 5 2

Question 2.
Plus Two Maths Model Question Papers Paper 5 3

Question 3.
Find rate of change of area of a circle.
a. With respect to the radius, when r = 10cm
b. With respect to the time when the radius is increasing at the rate of 0.7cm/s. Given that r=5cm.
Plus Two Maths Model Question Papers Paper 5 4

Question 4.
Plus Two Maths Model Question Papers Paper 5 5

Question 5.
Find area of a circle with centre (0, 0) and radius “a” using integration.
Plus Two Maths Model Question Papers Paper 5 6

Question 6.
Consider the differential equation \(\frac { dy }{ dx } \) = \(\frac { x+y }{ x } \)
a. Write the order of the differential equation.
b. Solve above given differential equation.
Plus Two Maths Model Question Papers Paper 5 7

Question 7.
Following Table shows a brief description about manufacturing process of a company. Time required in hours per unit of the product and maximum availability of machines is also given in the table
Plus Two Maths Model Question Papers Paper 5 8
a. Write the objective function.
b. Whether it is a maximisation case or a minimisation case Justify,
c. Write the contraints.
Plus Two Maths Model Question Papers Paper 5 9

Question 8 to 17 carry 4 scores each. Answer any eight questions only
Plus Two Maths Model Question Papers Paper 5 10

Question 8.
a. Afunctionf: A → B whereA= {1,2,3} and B= {4,5,6} defined by f (1) = 5, f (2) = 6, f (3) = 4, Check whether f is a bijection. If it is bijection, write f1 as set of ordered pairs.2
b. The operation table for an operation * is given below. Given that I is the identify element, then which among the following is true regarding the elements in first column?
Plus Two Maths Model Question Papers Paper 5 11
Plus Two Maths Model Question Papers Paper 5 12

Question 9.
Plus Two Maths Model Question Papers Paper 5 13
Plus Two Maths Model Question Papers Paper 5 14

Question 10.
a. Find th relation between ‘a’ and ‘b’ so that the function defined by
Plus Two Maths Model Question Papers Paper 5 15
b. “All continuous function are not differ-entiable”. Justify this statement with an example.
Plus Two Maths Model Question Papers Paper 5 16

Question 11.
a. Find the equation of the tangent to the curve y = x2 – 2x + 7 at (2,7)
b. Find the maximum value of the function.
Plus Two Maths Model Question Papers Paper 5 17

Question 12.
Plus Two Maths Model Question Papers Paper 5 18

Question 13.
Consider the differential equation \(\frac { xdy }{ dx } \) + y = \(\frac { 1 }{ { x }^{ 2 } } \)
a. Find the integrating factor
b. Solve the above differential equation.
Plus Two Maths Model Question Papers Paper 5 19

Question 14.
Plus Two Maths Model Question Papers Paper 5 20

Question 15.
Plus Two Maths Model Question Papers Paper 5 21

Question 16.
a. Find the cartesian equation of a line passing through the origin and (5,-2,3)
b. The point P(x,y,z) lies in the First Octant and its distance from origin is 12 units. If the position vector of P makes angles 45°, 60° with x and y axes respectively, find co-ordinates of P.
Plus Two Maths Model Question Papers Paper 5 22

Question 17.
Solve graphically :
Plus Two Maths Model Question Papers Paper 5 23

Question 18 to 24 carry 6 scores each. Answer any 5 questions only
Plus Two Maths Model Question Papers Paper 5 24

Question 18.
Plus Two Maths Model Question Papers Paper 5 25
Plus Two Maths Model Question Papers Paper 5 26

Question 19.
a. Without expanding prove that
Plus Two Maths Model Question Papers Paper 5 27
b. Consider the following system of equations 2x – 3y + 5z = 11, 3x + 2y – 4z = -5, x + y-2z = -3
i. Express the system in the form Ax = B.
ii. Solve the system by matrix method a.
Plus Two Maths Model Question Papers Paper 5 28

Question 20.
Plus Two Maths Model Question Papers Paper 5 29

Question 21.
Evaluate the following.
Plus Two Maths Model Question Papers Paper 5 30
Plus Two Maths Model Question Papers Paper 5 31

Question 22.
Consider the parabolas y2 = 4x and x2 = 4y
a. Draw rough figure for the above parabolas.
b. Find the point of intersection of the two parabolas.
c. Find the area bounded by these two pa-rabolas.
Plus Two Maths Model Question Papers Paper 5 32

Question 23.
a. Find the shortest distance between the lines whos vector equations are
Plus Two Maths Model Question Papers Paper 5 33
b. If a plane meets positive x axis at a distance of 2 units from origin, positive y axis at a distance of 3 units from orign and positive z axiz at a distance of 4 units from origin. Find the equation of the plane.
c. Find the prependicular distance of (0,0,0) from the plane obtained in part (b).
Plus Two Maths Model Question Papers Paper 5 34
Plus Two Maths Model Question Papers Paper 5 35

Question 24.
a. A die is thrown twice let the event A be ‘odd number on First throw’ and B be ‘odd number on the second throw’ check whether A and B are independent.
b. Coloured balls are distributed in three boxes as shown in the following table.
Plus Two Maths Model Question Papers Paper 5 36
A box is selected at random and a ball is taken out. If the ball taken is of red colour, What is the probabability that the other ball in the box is also of red colour?
Plus Two Maths Model Question Papers Paper 5 37

ANSWERS

Answer 1.
Plus Two Maths Model Question Papers Paper 5 38

Answer 2.
Plus Two Maths Model Question Papers Paper 5 39

Answer 3.
Plus Two Maths Model Question Papers Paper 5 40

Answer 4.
Plus Two Maths Model Question Papers Paper 5 41
Plus Two Maths Model Question Papers Paper 5 42

Answer 5.
Plus Two Maths Model Question Papers Paper 5 43

Answer 6.
Plus Two Maths Model Question Papers Paper 5 44
Plus Two Maths Model Question Papers Paper 5 45

Answer 7.
Lex x = Machine G and y = Machine H
a. Objective function: Z= 20x + 30y
b. It is a maximization problem.
c. Constraints are:
3x + 4y < 10 ; 5x + 6y < 1 5 ; x > 0 ; y > 0

Answer 8.
a. A = {1,2,3}
B = {4,5,6}
f(1) = 5 ; f (2) = 6 ; f (3) = 4
Since it is one-one as well as onto, f is bijective.

∴ f = {(1,5), (2,6), (3,4)}
∴ f-1 ={(5,1), (6,2), (4,3)}

b. (ii) 1,2,3
1 * 1 = 2,
2 * 1=2
Further 3 * 2 = 3
2 * 3 = 3
∴ * is commutative.

Answer 9.
Plus Two Maths Model Question Papers Paper 5 46
Plus Two Maths Model Question Papers Paper 5 47

Answer 10.
Plus Two Maths Model Question Papers Paper 5 48
Plus Two Maths Model Question Papers Paper 5 49
Plus Two Maths Model Question Papers Paper 5 50

Answer 11.
Plus Two Maths Model Question Papers Paper 5 51
Plus Two Maths Model Question Papers Paper 5 52
Plus Two Maths Model Question Papers Paper 5 53

Answer 12.
Plus Two Maths Model Question Papers Paper 5 54
Plus Two Maths Model Question Papers Paper 5 55
Plus Two Maths Model Question Papers Paper 5 56

Answer 13.
Plus Two Maths Model Question Papers Paper 5 57
Plus Two Maths Model Question Papers Paper 5 58

Answer 14.
Plus Two Maths Model Question Papers Paper 5 59
Plus Two Maths Model Question Papers Paper 5 60

Answer 15.
Plus Two Maths Model Question Papers Paper 5 61

Answer 16.
Plus Two Maths Model Question Papers Paper 5 62

Answer 17.
Plus Two Maths Model Question Papers Paper 5 63
Plus Two Maths Model Question Papers Paper 5 64
Plus Two Maths Model Question Papers Paper 5 65
Plus Two Maths Model Question Papers Paper 5 66
Plus Two Maths Model Question Papers Paper 5 67

Answer 18.
Plus Two Maths Model Question Papers Paper 5 68
Plus Two Maths Model Question Papers Paper 5 69

Answer 19.
Plus Two Maths Model Question Papers Paper 5 70
Plus Two Maths Model Question Papers Paper 5 71
Plus Two Maths Model Question Papers Paper 5 72
Plus Two Maths Model Question Papers Paper 5 73

Answer 20.
Plus Two Maths Model Question Papers Paper 5 85
Plus Two Maths Model Question Papers Paper 5 86

Answer 21.
Plus Two Maths Model Question Papers Paper 5 74
Plus Two Maths Model Question Papers Paper 5 75
Plus Two Maths Model Question Papers Paper 5 76
Plus Two Maths Model Question Papers Paper 5 77

Answer 22.
Plus Two Maths Model Question Papers Paper 5 78
Plus Two Maths Model Question Papers Paper 5 79

Answer 23.
Plus Two Maths Model Question Papers Paper 5 80
Plus Two Maths Model Question Papers Paper 5 81
Plus Two Maths Model Question Papers Paper 5 82

Answer 24.
Plus Two Maths Model Question Papers Paper 5 83
Plus Two Maths Model Question Papers Paper 5 84

Plus Two Maths Previous Year Question Papers and Answers

Plus Two Maths Model Question Paper 4

Kerala Plus Two Maths Model Question Paper 4

Time : 2 1/2 Hours
Cool off time : 15 Minutes
Maximum : 80 Score

General Instructions to Candidates :

  • There is a ‘Cool off time’ of 15 minutes in addition to the writing time.
  • Use the ‘Cool off time’ to get familiar with questions and to plan your answers.
  • Read questions carefully before you answering.
  • Read the instructions careully.
  • When you select a question, all the sub-questions must be answered from the same question itself.
  • Calculations, figures and graphs should be shown in the answer sheet itself.
  • Malayalam version of the questions is also provided.
  • Give equations wherever necessary.
  • Electronic devices except non programmable calculators are not allowed in the Examination Hall.

Plus Two Maths Previous Year Question Papers and Answers 2018 1

QUESTIONS

Question 1 to 7 carry 3 scores each. Answer any six questions only
Plus Two Maths Model Question Papers Paper 4 1

Question 1.
a. The slope of the tangent to the curve given by
x = 1- cos θ , y = θ – sin θ at θ = \(\frac { \pi }{ 2 }\) is
(i) 0
(ii) -1
(iii) 1
(iv) Not defined

b. Find the intervals in which the function f (x) = x2 – 4x + 6 is strictly decreasing.
Plus Two Maths Model Question Papers Paper 4 2

Question 2.
Plus Two Maths Model Question Papers Paper 4 3
Plus Two Maths Model Question Papers Paper 4 4

Question 3.
Plus Two Maths Model Question Papers Paper 4 5

Question 4.
a. Write the Cartesian equation of the straight line through the point (1, 2, 3) and along the vector 3\(\widehat { i }\) + \(\widehat { j }\) + 2\(\widehat { k }\).
b. Write a general point on this straight line.
c. Find the distance from (1, 2, 3) to the plane 2x + 3y – z + 2 = 0.
Plus Two Maths Model Question Papers Paper 4 6

Question 5.
Plus Two Maths Model Question Papers Paper 4 7
Plus Two Maths Model Question Papers Paper 4 8

Question 6.
Solve the system of linear equations :
x + 2y + z = 8
2x + y – z = 1
x – y + z = 2
x + 2y + z = 8
2x + y – z = 1
Plus Two Maths Model Question Papers Paper 4 9

Question 7.
Plus Two Maths Model Question Papers Paper 4 10

Question 8 to 17 carry 4 scores each. Answer any eight questions only
Plus Two Maths Model Question Papers Paper 4 11

Question 8.
Plus Two Maths Model Question Papers Paper 4 12

Question 9.
Plus Two Maths Model Question Papers Paper 4 13

Question 10.
i. Find the equation of the plane through the points (3,-1,2), (5,2,4) and (-1,-1,6).
ii. Find the perpendicular distance from the point (6,5,9) to this plane.
Plus Two Maths Model Question Papers Paper 4 14

Question 11.
Plus Two Maths Model Question Papers Paper 4 15
a. Express the euqations of the lines into vector form.
b. Find the shortest distance between the lines.
Plus Two Maths Model Question Papers Paper 4 16

Question 12.
Plus Two Maths Model Question Papers Paper 4 17

Question 13.
a. Find the equation of a plane with intercepts 2, 3 and 4 on X, Y and Z axes respectively.
b. Find the distance of the point (-1,-2, 3) from the plane r.(2\(\widehat { i }\) – \(\widehat { j }\)) + 4 \(\widehat { k }\)  = 4.
Plus Two Maths Model Question Papers Paper 4 18

Question 14.
Plus Two Maths Model Question Papers Paper 4 19

Question 15.
Plus Two Maths Model Question Papers Paper 4 20

Question 16.
Plus Two Maths Model Question Papers Paper 4 21
Plus Two Maths Model Question Papers Paper 4 22

Question 17.
a. For two independent events A and B, which of the following pair of events need not be independent?
i. A’, B’
ii. A, B’
iii. A’, B
iv. A-B, B-A
Plus Two Maths Model Question Papers Paper 4 23

Question 18 to 24 carry 6 scores each. Answer any 5 questions only
Plus Two Maths Model Question Papers Paper 4 24

Question 18.
Consider the following L.P.P.
Maximize Z = 3x+2y
Subject to the constraints
x+2y < 10
3x+y < 15
x, y > 0
a. Draw its feasible region.
b. Find the comer points of the feasible region.
c. Find the maximum value of Z.
Plus Two Maths Model Question Papers Paper 4 25
Plus Two Maths Model Question Papers Paper 4 26

Question 19.
a. y=a cos x+b sin x is the solution of the differential equation
Plus Two Maths Model Question Papers Paper 4 27
Plus Two Maths Model Question Papers Paper 4 28

Question 20.
Plus Two Maths Model Question Papers Paper 4 29
Plus Two Maths Model Question Papers Paper 4 30

Question 21.
Plus Two Maths Model Question Papers Paper 4 31

Question 22.
Plus Two Maths Model Question Papers Paper 4 32

Question 23.
Plus Two Maths Model Question Papers Paper 4 33
Plus Two Maths Model Question Papers Paper 4 34
Plus Two Maths Model Question Papers Paper 4 35

Question 24.
a. If \(\overline { a } \),\(\overline { b } \),\(\overline { c } \),\(\overline { d } \) respectively are the position vectors representing the vertices A, B, C, D of a parallelogram, then write \(\overline { d } \) in terms of \(\overline { a } \), \(\overline { b } \) and \(\overline { c } \) .

b. Find the projection vector of \(\overline { b } \) = \(\widehat { i }\) + 2 \(\widehat { j }\) + \(\widehat { k }\) along the vector \(\overline { a } \) = 2i +j + 2k. Also write \(\overline { b } \) as the sum of a vector along \(\overline { a } \) and a vector perpendicular to \(\overline { a } \) .

c. Find the area of a parallelogram for which the vectors 2 \(\widehat { i }\) + \(\widehat { j }\) and 3 \(\widehat { i }\) + \(\widehat { j }\) + 4 \(\widehat { k }\) are adjacent sides.
Plus Two Maths Model Question Papers Paper 4 36

ANSWERS

Answer 1.
Plus Two Maths Model Question Papers Paper 4 37
The point x = 2 divides the real line into two disjoint intervals namly (-∞, 2) and (2,∞). In the interval(-∞,2), f'(x)=2x-4 < 0
∴ f is strictly decreasing in this interval.

Answer 2.
Plus Two Maths Model Question Papers Paper 4 38
Plus Two Maths Model Question Papers Paper 4 39

Answer 3.
Plus Two Maths Model Question Papers Paper 4 40

Answer 4.
Plus Two Maths Model Question Papers Paper 4 41

Answer 5.
Plus Two Maths Model Question Papers Paper 4 42
Plus Two Maths Model Question Papers Paper 4 43

Answer 6.
Plus Two Maths Model Question Papers Paper 4 44
Plus Two Maths Model Question Papers Paper 4 45

Answer 7.
Plus Two Maths Model Question Papers Paper 4 46
Plus Two Maths Model Question Papers Paper 4 47

Answer 8.
Plus Two Maths Model Question Papers Paper 4 48

Answer 9.
Plus Two Maths Model Question Papers Paper 4 49

Answer 10.
Plus Two Maths Model Question Papers Paper 4 50
Plus Two Maths Model Question Papers Paper 4 51

Answer 11.
Plus Two Maths Model Question Papers Paper 4 52

Answer 12.
Plus Two Maths Model Question Papers Paper 4 53

Answer 13.
Plus Two Maths Model Question Papers Paper 4 54
Plus Two Maths Model Question Papers Paper 4 55

Answer 14.
Plus Two Maths Model Question Papers Paper 4 56

Answer 15.
Plus Two Maths Model Question Papers Paper 4 57
Plus Two Maths Model Question Papers Paper 4 58
Plus Two Maths Model Question Papers Paper 4 59

Answer 16.
Plus Two Maths Model Question Papers Paper 4 60
Plus Two Maths Model Question Papers Paper 4 61

Answer 17.
Plus Two Maths Model Question Papers Paper 4 62

Answer 18.
Plus Two Maths Model Question Papers Paper 4 63
Plus Two Maths Model Question Papers Paper 4 64

Answer 19.
Plus Two Maths Model Question Papers Paper 4 65
Plus Two Maths Model Question Papers Paper 4 66

Answer 20.
Plus Two Maths Model Question Papers Paper 4 67

Answer 21.
Plus Two Maths Model Question Papers Paper 4 68
Plus Two Maths Model Question Papers Paper 4 69

Answer 22.
Plus Two Maths Model Question Papers Paper 4 70
Plus Two Maths Model Question Papers Paper 4 71
Plus Two Maths Model Question Papers Paper 4 72

Answer 23.
Plus Two Maths Model Question Papers Paper 4 73
Plus Two Maths Model Question Papers Paper 4 74
Plus Two Maths Model Question Papers Paper 4 75

Answer 24.
Plus Two Maths Model Question Papers Paper 4 76
Plus Two Maths Model Question Papers Paper 4 77

Plus Two Maths Previous Year Question Papers and Answers

Plus Two Maths Model Question Paper 3

Kerala Plus Two Maths Model Question Paper 3

Time : 2 1/2 Hours
Cool off time : 15 Minutes
Maximum : 80 Score

General Instructions to Candidates :

  • There is a ‘Cool off time’ of 15 minutes in addition to the writing time.
  • Use the ‘Cool off time’ to get familiar with questions and to plan your answers.
  • Read questions carefully before you answering.
  • Read the instructions careully.
  • When you select a question, all the sub-questions must be answered from the same question itself.
  • Calculations, figures and graphs should be shown in the answer sheet itself.
  • Malayalam version of the questions is also provided.
  • Give equations wherever necessary.
  • Electronic devices except non programmable calculators are not allowed in the Examination Hall.

Plus Two Maths Previous Year Question Papers and Answers 2018 1

QUESTIONS

Question 1 to 7 carry 3 scores each. Answer any six questions only
Plus Two Maths Model Question Papers Paper 3 1

Question 1.
a. Let f : R → R be a function defined by f (x) = x3 + 5. Then f1 (x) is
i. (x+5)1/3
ii. (x-5)1/3
iii. (5-x)1/3
iv. 5-x

b. Let * be a binary operation defined on Q
a*b = a-b + ab. Check whether
i. It is commutative?
ii. Is * associative ?
Plus Two Maths Model Question Papers Paper 3 2

Question 2.
Plus Two Maths Model Question Papers Paper 3 3
Plus Two Maths Model Question Papers Paper 3 4

Question 3.
Plus Two Maths Model Question Papers Paper 3 5

Question 4.
Plus Two Maths Model Question Papers Paper 3 6

Question 5.
Prove that the function f given by f (x)= log sin x is strictly increasing on ( 0,\(\frac { \pi }{ 2 } \))
Plus Two Maths Model Question Papers Paper 3 7

Question 6.
Plus Two Maths Model Question Papers Paper 3 9
Plus Two Maths Model Question Papers Paper 3 10

Question 7.
Plus Two Maths Model Question Papers Paper 3 11

Question 8 to 17 carry 4 scores each. Answer any eight questions only
Plus Two Maths Model Question Papers Paper 3 12

Question 8.
a. Show that the relation R in set of real numbers defined as R = {(a,b): a < b2} is neither reflexive nor symmetric not transitive.
b. Show that the operation * on Q, defined by a*b = a+b-ab is commutative, and ex-its and identity elements find it.
Plus Two Maths Model Question Papers Paper 3 13

Question 9.
a. The principal value of the expression cos-1 cos (680) is …………..
Plus Two Maths Model Question Papers Paper 3 14
Plus Two Maths Model Question Papers Paper 3 15

Question 10.
Plus Two Maths Model Question Papers Paper 3 16
Plus Two Maths Model Question Papers Paper 3 17

Question 11.
Plus Two Maths Model Question Papers Paper 3 18

Question 12.
Plus Two Maths Model Question Papers Paper 3 19
Plus Two Maths Model Question Papers Paper 3 20

Question 13.
Plus Two Maths Model Question Papers Paper 3 21

Question 14.
Plus Two Maths Model Question Papers Paper 3 22

Question 15.
a. Find the distance between the planes x-y + z-5 = 0 and 2x-2y + 2z = 0.
b. Write the vector equation corresponding to Cartesian equation of a line
Plus Two Maths Model Question Papers Paper 3 23
Plus Two Maths Model Question Papers Paper 3 24

Question 16.
Find the shortest distance between the lines
Plus Two Maths Model Question Papers Paper 3 25

Question 17.
Plus Two Maths Model Question Papers Paper 3 26

Question 18 to 25 carry 6 scores each. Answer any 5 questions only
Plus Two Maths Model Question Papers Paper 3 27

Question 18.
Plus Two Maths Model Question Papers Paper 3 28
Plus Two Maths Model Question Papers Paper 3 81

Question 19.
a. Use differential to approximate (0.999)1/10
b. A window is in the form of rectangle sur-mounted by a semicircular opening. The total perimeter of the windows is 1 Om. Find the dimensions of the window to admit maximum light through the whole opening.
Plus Two Maths Model Question Papers Paper 3 82

Question 20.
Find the area lying above x axis and included between the circle x2 + y2 = 8x and inside of the parabola y2 = 4x. Also draw a neat diagram.
Plus Two Maths Model Question Papers Paper 3 83

Question 21.
Evaluate:
Plus Two Maths Model Question Papers Paper 3 84

Question 22.
Minimize and maximize Z = 5x + 10 y subject to x + 2y < 120, x + y > 60, x – 2y > 0, x, y > 0
a. Draw the feasible region
b. Find the comer points
c. Find the maximum and minimum profit.
Plus Two Maths Model Question Papers Paper 3 85
Plus Two Maths Model Question Papers Paper 3 86

Question 23.
a. Find the distance of the point (-1, -5, -10) from the point of intersection of the line
Plus Two Maths Model Question Papers Paper 3 87

Question 24.
a. Two numbers are selected at random (with-out replacement) from the Pt six positive integers.

Let X denote the larger of the two numbers obtained. Find E(X) and Var(X)

b. A card from a pack of 52 cards is lost from the remaining X cards of the pack, two cards are drawn and are found to be both spades. Find the probability of the lost card being a spade.
Plus Two Maths Model Question Papers Paper 3 88
Plus Two Maths Model Question Papers Paper 3 29
Plus Two Maths Model Question Papers Paper 3 30

ANSWERS

Answer 1.
a. ii
b. a * b = a – b + ab
b*a = b- a + ba = b – a + ab
∴ * is not commutative.
(a,b) * c = d * c = d- c + dc
= a- b + ab – c + ac – bc + abc
= a- b + ab – c + ac – bc + abc
= a – b – c + ab – bc + ca + abc
a * (b * c) = a * d = a- d + ad
= a – (b – c + be) + a (b – c + be)
= a- b + c – bc + ab – ac + abc
= a – b + c ab – be + ca + abc
∴ * is not associative.

Answer 2.
Plus Two Maths Model Question Papers Paper 3 31
Plus Two Maths Model Question Papers Paper 3 32
Plus Two Maths Model Question Papers Paper 3 33

Answer 3.
We are giving that
Plus Two Maths Model Question Papers Paper 3 34
Plus Two Maths Model Question Papers Paper 3 35

Answer 4.
Plus Two Maths Model Question Papers Paper 3 36
Plus Two Maths Model Question Papers Paper 3 37

Answer 5.
Plus Two Maths Model Question Papers Paper 3 38

Answer 6.
Plus Two Maths Model Question Papers Paper 3 39

Answer 7.
Plus Two Maths Model Question Papers Paper 3 40
Plus Two Maths Model Question Papers Paper 3 41

Answer 8.
a. R = {(a,b): a < b2}
Relation R is defined in the set of real numbers.
i. Reflexive
Consider a ∈ R
If a ∈ R ⇒ a < a2 which is false (a, a) ∈ R
R is not reflexive.

ii. Symmetric
Let a,b ∈ R and
(a,b) ∈ R ⇒ a < b2 and b < a2,
which is false ⇒ (a,b) ∈ R, but (b,a) ∈ R
∴ R is not symmetric.

iii. Transitive
Let a, b, c ∈ R
Plus Two Maths Model Question Papers Paper 3 42
Plus Two Maths Model Question Papers Paper 3 43

Answer 9.
Plus Two Maths Model Question Papers Paper 3 44
Plus Two Maths Model Question Papers Paper 3 45

Answer 10.
Plus Two Maths Model Question Papers Paper 3 46
Plus Two Maths Model Question Papers Paper 3 47

Answer 11.
Plus Two Maths Model Question Papers Paper 3 48

Answer 12.
Plus Two Maths Model Question Papers Paper 3 49
Plus Two Maths Model Question Papers Paper 3 50

Answer 13.
Plus Two Maths Model Question Papers Paper 3 51
Plus Two Maths Model Question Papers Paper 3 52
Plus Two Maths Model Question Papers Paper 3 53

Answer 14.
Plus Two Maths Model Question Papers Paper 3 54
Plus Two Maths Model Question Papers Paper 3 55
Plus Two Maths Model Question Papers Paper 3 56

Answer 15.
Plus Two Maths Model Question Papers Paper 3 57

Answer 16.
Plus Two Maths Model Question Papers Paper 3 58
Plus Two Maths Model Question Papers Paper 3 59

Answer 17.
Plus Two Maths Model Question Papers Paper 3 60

Answer 18.
Plus Two Maths Model Question Papers Paper 3 61
Plus Two Maths Model Question Papers Paper 3 62
Plus Two Maths Model Question Papers Paper 3 63
Plus Two Maths Model Question Papers Paper 3 64

Answer 19.
Plus Two Maths Model Question Papers Paper 3 65
Plus Two Maths Model Question Papers Paper 3 66
Plus Two Maths Model Question Papers Paper 3 67
Plus Two Maths Model Question Papers Paper 3 68

Answer 20.
The given equation of the circle x2 + y2 = 8x can be expressed as (x – 4)2 + y2 = 16. Thus, the centre of the circle is (4,0) and radius is 4. Its intersection with the parabola y2 = 4x gives
x2 + 4x = 8x
or
x2 – 4x = 0
or
x(x-4) = 0
or
x = 0, x = 4
Thus, the point of intersection of these two curves are 0 (0,0) and P (4,4) above the x-axis.
Plus Two Maths Model Question Papers Paper 3 69
From the above the required area of the region OPQCO included between these two curves above x-axis is
= (area of the region OCPO)
+ (area of the region PCQP)

Plus Two Maths Model Question Papers Paper 3 70

Answer 21.
Plus Two Maths Model Question Papers Paper 3 71
Plus Two Maths Model Question Papers Paper 3 72
Plus Two Maths Model Question Papers Paper 3 73

Answer 22.
a. The feasible region determine! by the constraints,
x + 2y < 120, x + y > 60, x – 2y > 0,
x > 0 and y > 0
is as follows.
Plus Two Maths Model Question Papers Paper 3 74

b. The comer points of the feasible are region are A(60,0), C(60,30) and D (40,20).
The values of Z at these comer points are as follows.

Corner point Z=5x + 10y
A (60,0) 300 → Minimum
B (120,0) 600 → Maximum
C (60,30) 600 → Maximum
D (40,20) 600

a. The minimum value of Z is 300 at (60,0) and the maximum value of Z is 600 at all the points on the line segment joining (120,0) and (60,30)

Answer 23.
Plus Two Maths Model Question Papers Paper 3 76
Plus Two Maths Model Question Papers Paper 3 77
Plus Two Maths Model Question Papers Paper 3 78

Answer 24.
a. The two positive integers can be sele-cted from the fist six positive integers without replacement in 6 x 5 = 30 ways.

X represents the larger of the two numbers obtained. Therefore, X can take the value of 2,3,4,5 or 6.

For X=2, the possible observations are (1,1)and(2,1)
∴ P (x = 2) = \(\frac { 2 }{ 30 } \) = \(\frac { 1 }{ 15 } \)

For X = 3 the possible observations are (1,3), (2,3), (3,1) and (3,2).
∴ p (x = 3) = \(\frac { 4 }{ 30 } \) = \(\frac { 2 }{ 15 } \)

For x = 4 the possible observations are

(1,4), (2,4), (3,4), (4,3), (4,2) and (4,1).
∴ p (x = 4) = \(\frac { 6 }{ 30 } \) = \(\frac { 1 }{ 5 } \)

For X = 5, the possible observations are (1.5) , (2,5), (3,5), (4,5), (5,4), (5,3), (5,2) and (5,1).
∴ p (x = 5) = \(\frac { 8 }{ 30 } \) = \(\frac { 4 }{ 15 } \)

For X = 6, the possible observations are (1.6), (2,6), (3,6), (4,6), (5,6), (6,4), (6,3), (6,2) and (6,1)
∴ p (x = 6) = \(\frac { 10 }{ 30 } \) = \(\frac { 1 }{ 3 } \)

Therefore, the required probability distribution is as follows.
Plus Two Maths Model Question Papers Paper 3 79

b. Let E and E, be the respective events of choosing a spade card and a card which is not spade. Out of 52 cards, 13 cards are spade and 39 cards are not spade.
Plus Two Maths Model Question Papers Paper 3 80

Plus Two Maths Previous Year Question Papers and Answers

Plus Two Maths Model Question Papers Paper 1

Kerala State Board New Syllabus Plus Two Maths Previous Year Question Papers and Answers.

Plus Two Maths Model Question Papers Paper 1 with Answers

Board SCERT
Class Plus Two
Subject Maths
Category Plus Two Previous Year Question Papers

Time : 2 1/2 Hours
Cool off time : 15 Minutes
Maximum : 80 Score

General Instructions to Candidates:

  • There is a ‘Cool off time’ of 15 minutes in addition to the writing time.
  • Use the ‘Cool off time’ to get familiar with questions and to plan your answers.
  • Read questions carefully before you answering.
  • Read the instructions carefully.
  • When you select a question, all the sub-questions must be answered from the same question itself.
  • Calculations, figures and graphs should be shown in the answer sheet itself.
  • Malayalam version of the questions is also provided.
  • Give equations wherever necessary.
  • Electronic devices except non-programmable calculators are not allowed in the Examination Hall.

Questions 1 to 7 carry 3 Scores each. Answer any six questions only. (6 × 3 = 18)

Question 1.
i) Let R be a relation on the set {1, 2, 3} given by R = {(1, 1), (2, 2), (1, 2), (2, 1), (2, 3)}. Which among the following element to be included to R so that R becomes Symmetric?
a) (3, 3)
b) (3, 2)
c) (1, 3)
d) (3, 1)
ii) If * is defined by a*b = a – b2 and \(\oplus\) is defined by a \(\oplus\) b = a2 + b , where a and b are integers. Then find the value of (3 \(\oplus\) 4) * 5
Answer:
i) b) (3, 2)
ii) (3 \(\oplus\) 4) * 5 = (32 + 4) * 5
= 13 * 5
= 13 – 52 = -12

Question 2.
If X + Y = \(\left[\begin{array}{ll}
5 & 2 \\
0 & 9
\end{array}\right]\) and X – Y = \(\left[\begin{array}{cc}
3 & 6 \\
0 & -1
\end{array}\right]\)find 2X – 3Y.
Answer:
Plus Two Maths Model Question Papers Paper 1, 1

Question 3.
Find rate of change of area of a circle.
a) with respect to the radius, when r = 10cm
b) with respect to the time when the radius is increasing at the rate of 0.7 cm/s. Given that r = 5cm
Answer:
i) Area of a circle A = πr2
Plus Two Maths Model Question Papers Paper 1, 2

Question 4.
i) Integrate \(\int \frac{(1+\log x)^{2}}{x} d x\)
Plus Two Maths Model Question Papers Paper 1, 3
Answer:
Plus Two Maths Model Question Papers Paper 1, 4

Question 5.
Find the area of a circle with centre (0, 0) and radius ‘a’ using integration.
Answer:
Equation of the circle is x2 + y2 = a2
y = \(\sqrt{a^{2}-x^{2}}\)
Area of the circle will 4 time the area under the curve from 0 to a.
Area = \(4 \int_{0}^{a} y d x=4 \int_{0}^{a} \sqrt{a^{2}-x^{2}} d x\)
Plus Two Maths Model Question Papers Paper 1, 5

Question 6.
Consider the differential equation \(\frac{d y}{d x}=\frac{x+y}{x}\)
i) Write the order of the differential equation.
ii) Solve the above given differential equation.
Answer:
i) Order = 1.
Plus Two Maths Model Question Papers Paper 1, 6

Question 7.
The following table shows a brief description of the manufacturing process of a company. The time required in hours per unit of the product and maximum availability of machine is also given in the table:

Product The time required in hours/Unit on machine Profit per Unit in Rupees
Machine G Machine H
A 3 5 20
B 4 6 30
Maximum available time 10 hrs/day Maximum available time 15 hrs/day

i) Write the objective function.
ii) Whether it is a maximization case or a minimization case. Justify.
iii) Write the constraints.
Answer:
Let x units = Machine G and y units = Machine H
i) Objective function: Z = 20x + 30y
ii) It is a maximization problem.
iii) Constraints are:
3x + 4y ≤ 10; 5x + 6y ≤ 15; x, y ≥ 0

Questions 8 to 17 carry 4 scores each. Answer any 8 questions. (8 × 4 = 32)

Question 8.
i) A function f : A → B, where A = {1, 2, 3} and B = {4, 5, 6} defined by f(1) = 5, f(2) = 6, f(3) = 4, Check whether f is a bijection. If it is a bijection, write f-1 as set of ordered pairs.
ii) The operation table for an operation * is given below. Given that 1 is the identity element. Then which among the following is true regarding the element in the first column?

* 1 2 3
1 2 3
2 2 3
3 3 3

a) 3, 2, 2
b) 1, 2, 3
c) 1, 1, 2
d) 2, 2, 2
Check whether * is commutative.
Answer:
i) A = {1, 2, 3}; B = {4, 5, 6}
f(1) = 5, f(2) = 6, f(3) = 4
Hence f= {(1, 5),(2, 6),(3, 4)}
f-1 = {(5, 1), (6, 2), (4, 3)}
ii) b) 1, 2, 3
Since; 1*2 = 2 and 2*1 =2
3 * 2 = 3 and 2*3 = 3
Therefore * is commutative

Question 9.
i) If Sin-1 x = y, then
(a) 0 ≤ y ≤ π
b) \(-\frac{\pi}{2}\) ≤ y ≤ \(\frac{\pi}{2}\)
c) 0 < y < π
d) \(-\frac{\pi}{2}\) ≤ y ≤ \(\frac{\pi}{2}\)
ii) Find the principal value of sin-1 (\(\frac{1}{2}\))
iii) sin-1x = \(\frac{3}{4}\) find the value of cos-1 x
Answer:
i) \(-\frac{\pi}{2}\) ≤ y ≤ \(\frac{\pi}{2}\)
ii) sin-1 \(\frac{1}{2}\) = \(\frac{\pi}{6}\)
iii) sin-1x + cos-1x = \(\frac{\pi}{2}\)
\(\frac{3}{4}\) + cos-1x = \(\frac{\pi}{2}\) ⇒ cos-1x = \(\frac{\pi}{2}\) – \(\frac{3}{4}\)

Question 10.
i) Find the relation between ‘a’ and ‘b’ so that the function defined by
f(x) = \(\left\{\begin{array}{ll}
a x+1, & x \leq 3 \\
b x+3, & x>3
\end{array}\right.\) is continuous at x = 3
ii) “All continuous function are not differentiable.” Justify your answer with an example.
Answer:
i) Since f(x) is continuous at x = 3
\(\lim _{x \rightarrow 3^{-}}\) f(x) = \(\lim _{x \rightarrow 3^{+}}\) f(x) = f(3)
\(\lim _{x \rightarrow 3}\) (ax + 1) = \(\lim _{x \rightarrow 3}\) (bx + 3) = 3a +1
⇒ 3a + 1 = 3b + 3 = 3a +1
⇒ 3a + l = 3b + 3 ⇒ 3a – 3b – 2 = 0

ii) Consider the function f(x) = |x|.
Let us check the continuity and differentiability at x = 0.
Plus Two Maths Model Question Papers Paper 1, 7
Left derivative * Right derivative
Therefore not differentiable at x = 0.

Question 11.
i) Find the equation to the tangent to the curve y = x2 – 2x + 7 at (2, 7)
ii) Find the maximum value of the function?
Answer:
f(x) = sin x + cos x, 0 < x < \(\frac{\pi}{2}\)
i) y = x2 – 2x + 7 ⇒ \(\frac{d y}{d x}\) = 2x – 2
Slope at x = 2 is 2 × 2 – 2 – 2
Equation of the tangent is
(y – y1) = m(x – x1)
⇒ (y – 7) = 2(x- 2)
⇒ y – 7 = 2x – 4
⇒ 2x – y + 3 = 0

ii) c) f(x) = sin x + cos x
f'(x) = cos x – sin x
f”(x) = -sinx-cosx
For turning points;
f'(x) = cos x – sin x = 0 ⇒ tan x = 1
Plus Two Maths Model Question Papers Paper 1, 8

Question 12.
Integrate \(\int \frac{x+2}{2 x^{2}+6 x+5} d x\)
Answer:
Put x + 2 = A(4x + 6) + B
4A = 1 ⇒ A = \(\frac{1}{4}\)
Plus Two Maths Model Question Papers Paper 1, 9
Plus Two Maths Model Question Papers Paper 1, 10

Question 13.
Consider the differential equation
x \(\frac{d y}{d x}\) + y = \(\frac{1}{x^{2}}\)
i) Find the integrating factor.
ii) Solve the above differential equation.
Answer:
Plus Two Maths Model Question Papers Paper 1, 11

Question 14.
lf the vectors \(\overline{P Q}\) = -3i + 4j + 4k and \(\overline{P R}\) = -5i + 2j + 4k are the sides of a ΔPQR
i) Find the angle between \(\overline{P Q}\) and \(\overline{P R}\)
ii) Find the length of the median through the vertex P.
Answer:
Plus Two Maths Model Question Papers Paper 1, 12
ii)
Plus Two Maths Model Question Papers Paper 1, 13
With respect to the initial P the position vector of Q and R will be \(\overline{P Q}\) = -3i + 4j + 4k
and \(\overline{P R}\) = -5i + 2j + 4k respectively.
Since M is the midpoint of QR. The position vector of M will be
Plus Two Maths Model Question Papers Paper 1, 14

Question 15.
i) If \(\bar{a}\) = 5i – j – 3k and \(\bar{b}\) = i + 3j + 5k, then show that the vectors \(\bar{a}+\bar{b}, \bar{a}-\bar{b}\) are perpendicular.
ii) If \(\bar{a}\) = i – 2j + 3k, \(\bar{b}\) = 2i + 3j – 4k and
\(\bar{c}\) = i – 3j + 5k, then check whether \(\bar{a}\), \(\bar{b}\), \(\bar{c}\) are coplanar.
Answer:
Plus Two Maths Model Question Papers Paper 1, 15

Question 16.
i) Find the Cartesian equation of the line passing through origin and (5, -2, 3) ii) The point P(x, y, z) lies in the first octant and its distance from the origin is 12 units. If the position vector of P makes angles 45°, 60°with x and y axes respectively, find coordinates of P.
Answer:
Plus Two Maths Model Question Papers Paper 1, 16
Since P lies in the first octant, we take n = \(\frac{1}{2}\)
Therefore the coordinate of P is
Plus Two Maths Model Question Papers Paper 1, 17

Question 17.
Solve graphically Maximise Z = 5x + 3y Subject to the constraints 3x + 5y ≤ 15; 5x + 2y ≤ 10; x ≥ 0, y ≥ 0.
Answer:
In the figure the shaded region OABC is the fesible region. Here the region is bounded.
The corner points are
O(0, 0), A(2, 0), B(\(\frac{20}{19}, \frac{45}{19})\), C(0, 3)
Plus Two Maths Model Question Papers Paper 1, 18
Given Z = 5x + 3y

Corner points Value of Z
O Z = 0
A Z = 5(2) + 3(0) = 10
B Z = \(5\left(\frac{20}{19}\right)+3\left(\frac{45}{19}\right)=\frac{235}{19}\)
C Z = 5(0) + 3(3) = 9

Since maximum value of Z occurs at B, the soluion is z = \(\frac{235}{19}\), (\(\frac{20}{19}\), \(\frac{45}{19}\))

Questions 18 to 24 carry 6 scores each. Answer any 5 questions. (5 × 6 = 30)

Question 18.
A = \(\left[\begin{array}{ccc}
3 & 3 & -1 \\
-2 & -2 & 1 \\
-4 & -5 & 2
\end{array}\right]\)
a) Find AT
b) Express A as the sum of a symmetric and skew symmetric matrix.
ii) If AT = \(\left[\begin{array}{cc}
\cos x & \sin x \\
-\sin x & \cos x
\end{array}\right]\), verift that AT A = I
Answer:
Plus Two Maths Model Question Papers Paper 1, 19
Plus Two Maths Model Question Papers Paper 1, 20
Plus Two Maths Model Question Papers Paper 1, 21

Question 19.
i) Without expanding prove that
\(\left|\begin{array}{ccc}
x+y & y+z & z+x \\
z & x & y \\
1 & 1 & 1
\end{array}\right|\) = 0
ii) Consider the following system of equations
2x – 3y + 5z = 11; 3x + 2y – 4z = -5; x + y – 2z = -3
a) Express the system in Ax = B form.
b) Solve the system by matrix method.
Answer:
Plus Two Maths Model Question Papers Paper 1, 22
Plus Two Maths Model Question Papers Paper 1, 23

Question 20.
Find \(\frac{d y}{d x}\) of the following
i) x2 + 2xy + 2y2 = 1
ii) yx = 2x
iii) x = cos θ; y = sin θ at θ = \(\frac{\pi}{4}\)
Answer:
i) x2 + 2xy + 2y2 = 1
Differentiating w.r.to x;
Plus Two Maths Model Question Papers Paper 1, 24

ii) yx = 2x
Take log on both sides;
x log y = x log 2
Differentiating w.r.to x;
Plus Two Maths Model Question Papers Paper 1, 25

Question 21.
Evaluate the following
Plus Two Maths Model Question Papers Paper 1, 26
Answer:
Plus Two Maths Model Question Papers Paper 1, 27
Plus Two Maths Model Question Papers Paper 1, 28
Plus Two Maths Model Question Papers Paper 1, 29

Question 22.
Consider the parabolas y2 = 4x, x2 = 4y
i) Draw a rough figure for the above parabolas.
ii) Find the point of intersection of the two parabolas.
iii) Find the area bounded by these two parabolas.
Answer:
Plus Two Maths Model Question Papers Paper 1, 30
ii) solving the two conics we get the point of intersection as (0, 0) and (4, 4).
iii) Area of the enclosed region
Plus Two Maths Model Question Papers Paper 1, 31

Question 23.
i) Find the shortest distance between the lines whose vector equations are
\(\bar{r}\) = (i + 2j + 3k) + λ(i – 3j + 2k) and \(\bar{r}\) =(4i + 5j + 6k) + μ(i – 3j + 2k)
ii) If a plane meets positive x axis at a distance of 2 units from the origin, positive y axis at a distance of 3 units from the origin and positive z axis at a distance of 4 units from the origin. Find the equation of the plane.
iii) Find the perpendicular distance of (0, 0, 0) from the plane obtained in part (ii)
Answer:
i) \(\overline{a_{1}}\) = i + 2j + 3k; \(\bar{b}\) = i – 3j + 2k
\(\overline{a_{2}}\) = 4i + 5j + 6k Both lines are parallel.
Plus Two Maths Model Question Papers Paper 1, 32
Plus Two Maths Model Question Papers Paper 1, 33

Question 24.
i) A die is thrown twice let the event A be ‘odd number on first throw’ and B be ‘odd number on the second throw’ check whether A and B are independent.
ii) Coloured balls are distributed in three boxes as shown in the following table:

Box Red Black
1 2 0
II 0 2
III 1 1

A box is selected at random and a ball is taken out. If the ball taken is of red colour, What is the probability that the other ball in the box is also of red colour?
Answer:
i) n(S) = 36
A = {(1, 1), (1, 2), (1, 3), (1, 4), (1, 5), (1, 6), (3, 1), (3, 2), (3, 3), (3, 4), (3, 5), (3, 6), (5, 1), (5, 2), (5, 3), (5, 4), (5, 5), (5, 6)}
B = {(1, 1), (2, 1), (3, 1), (4, 1), (5, 1), (6, 1), (1, 3), (2, 3), (3, 3), (4, 3), (5, 3), (6, 3), (1, 5), (2, 5), (3,5), (4, 5), (5, 5), (6, 5)}
(A ∩ B) = {(1, 1), (1, 3), (1, 5), (3, 1), (3, 3), (3, 5), (5, 1), (5, 3), (5, 5)}
Plus Two Maths Model Question Papers Paper 1, 34
Therefore A and B are independent.

ii) Let B1, B2, B3 are the event of getting the boxes.
P(B1) = P(B2) = P(B3) = \(\frac{1}{3}\)
Let E be the event of getting a red ball.
Plus Two Maths Model Question Papers Paper 1, 35